Ребусы с ответами в картинках 6 класс: Ребусы для детей 6-7 лет — «Семья и Школа»

Содержание

Математические ребусы загадки кроссворды. Арифметические ребусы

Математические игры ребусы в картинках для школьников 5-7 классов

Клочкова Наталья Константиновна, учитель математики МБОУ «Бухарайская СОШ» с.Бухарай Заинский район
Описание: Данная работа может быть использована на уроках математики в 5–7 классах. Разгадывание ребусов можно предложить учащимся при проведении устного счета, можно предложить в качестве дидактического материала для домашних заданий. Эта работа может послужить как пособие для проведения внеклассных мероприятий, факультативов. Разгадывание ребусов развивает у ребенка сообразительность и учит его находить выход из сложных ситуаций, что, конечно же, пригодится в жизни. Отгадывая ребусы дети пополняют свой словарный запас, развивают внимание и образное мышление, тренируют зрительную память, учатся правильно писать и запоминать новые слова.
Цель: развитие интеллектуальных способностей, формирование логического мышления.

Задачи:
Обучающие: научить учащихся разгадывать ребусы с математической тематикой.
Развивающие: расширить кругозор учащихся в области математики.
Воспитательные: воспитать сознательное отношение к математике, как к важному предмету.
Введение:
Ребус – это головоломка в которой зашифровано слово. Это слово дается в виде рисунков с использованием букв и цифр, а также определенных фигур или предметов. Ребус – одна из самых интересных головоломок.
На этой картинке зашифровано слово КОМПЬЮТЕР.

Существуют определенные правила для разгадывания ребусов.
1.Запятая в самом начале слова говорит о том, что нужно убрать первую букву в этом слове, а запятая в конце – убрать последнюю букву в слове. Две запятых – убираем две буквы. В слове комар убираем последние две буквы АР, в слове утюг убираем первую букву У и последнюю букву Г.
2.Зачеркнутые цифры говорят о том, что буквы стоящие на этом месте убираются. В слове пять убираем вторую и третью буквы, то есть ЯТ.

Если будут зачеркнуты буквы, то они тоже убираются из слова.
3.Не зачеркнутые цифры показывают, что буквы, стоящие на месте 2 и 3 надо поменять местами. В слове утюг буквы Т и Ю меняем местами ЮТ. А теперь читаем слово полностью.
На этой картинке зашифровано слово ПЕРПЕНДИКУЛЯР.


4.Если рисунок в перевернутом виде, то загаданное с помощью картинки слово читается справа налево. Читается не слово репа, а апер. Первая буква А убирается. В слове пень последняя буква Ь убирается. Слово кит читается наоборот. В слове стул убираются первые две буквы СТ. Названия всех предметов, изображенных в ребусе, читаются только в именительном падеже.
5.«Стрелка» или знак «равно» говорят о том, что одну букву надо заменить другой. В нашем случае в слове тик букву Т надо заменить на букву Д. Теперь слово можно прочитать полностью.
На этой картинке зашифровано слово ВОСТОК.


6.Буквы, слова или картинки могут изображаться внутри других букв, над другими буквами, под и за ними. Тогда добавляются предлоги: В, НА, НАД, ПОД, ЗА. У нас в букве О содержится число СТО, поэтому получается В-О-СТО-К.
На этой картинке зашифровано слово КАРТА.


7.Цифры под картинкой указывают на то, что из данного слова нужно взять буквы, стоящие на местах под номерами 7,2,4,3,8 и составить их в том порядке, в котором расположены цифры. В слове ватрушка нужно взять буквы 7-К, 2-А, 4-Р, 3-Т, 8-А. Можно прочитать слово.
Давайте попробуем разгадать несколько ребусов из области математики.
ДОКАЗАТЕЛЬСТВО


ПЯТЬ


ЗАДАЧА


КОНУС


ВЕРШИНА


ДИАМЕТР


ЗНАМЕНАТЕЛЬ


ЛОБАЧЕВСКИЙ


МИНУС


АКСИОМА


ВЕКТОР


ВЫЧИТАНИЕ


ДВА


ДИАГОНАЛЬ


ТРЕУГОЛЬНИК


РОМБ


СТЕПЕНЬ


СЛОЖЕНИЕ


ЧИСЛО


ТОЧКА


СТЕРЕОМЕТРИЯ


Все задания оформлены яркими картинками и интересно проиллюстрированы, поэтому ребусы будут увлекать ребят. А можно попробовать и самим составить. Это будет еще интересней.

Ребусы для школьников с решением и ответами.

Математические задачи бывают самыми разнообразными по сложности, соответственно начинайте с ребенком разгадывать еще с детского садика. Математические ребусы почти всегда нравятся ребятам, поэтому Вам не нужно будет заставлять своего малыша заниматься. Мы постараемся Вам рассказать о том, какую пользу приносят математические ребусы детям, и какие именно головоломки можно предложить разгадывать школьникам определенного возраста.

Для чего нужны математические ребусы для детей?

Математика считается самой сложной наукой, которая способна доставить школьнику очень много проблем во время учения. Но ведь без обыкновенных навыков устного счета и разнообразных математических приемов невозможно просто в будущем нормально жить.

Продолжительные и достаточно сложные математические занятия, особенно с 1-го по 4-й классы, утомляют деток и не дают им возможности нормально усваивать услышанную информацию. Если Вы хотите, чтобы с Вашим ребенком такое не случилось, предложите ему изучать математику в игровой форме, к примеру, в виде математических головоломок или ребусов.

Многие школьники современного времени обожают в собственный досуг развлекаться за счет компьютерных игр либо общаться в социальных сетях с одноклассниками. Однако сегодня есть и те дети, которые не тратят собственного времени на такие игрушки, а отдают предпочтение развитию логики и сообразительности.

В настоящее время сеть Интернет заполнена разнообразными сайтами, где можно без проблем отыскать логические загадки и головоломки. Они предназначены не только для того, чтобы потратить собственное время, но еще чтобы полезно, а самое главное занимательно развлечься. Многие родители уже смогли по достоинству оценить преимущество математических головоломок, шарад, задачек, ребусов, так как их дети благодаря им смогли намного быстрее развиваться.

Благодаря математическим ребусам и задачам ребенок намного быстрее начинает правильнее рассуждать. У него формируется ум и логика.

Преимущество математических ребусов в том, что они не считаются обыкновенными математическими задачками. Они с первого знакомства заинтересовывают деток своим оригинальным изложением, возбуждают у детей желание быстрее найти разгадку на ту или иную головоломку.

Если Вы начнете с Вашим чадом регулярно находить решения к математическим ребусам, Ваш малыш уже очень скоро начнет без проблем решать более сложные задачки, которые до этого он не мог разгадать. Заинтересуйте собственного ребенка обычной математикой, и в этом Вам помогут математические ребусы.

Математические ребусы и головоломки – это загадки, имеющие различную степень сложности, составленные с применением графических элементов. Разгадывать подобные задачки – это очень увлекательно. Помимо этого, ребята более старшего возраста с огромным удовольствием самостоятельно могут составлять математические головоломки для друзей и одноклассников, что позволит им лучше тренировать собственный ум и интеллект, плюс развивать логику.

Если ребусы представлены в виде сложных загадок, детям приходится «поломать» немного голову, дабы отыскать верное решение. Во время данного увлекательного и познавательного занятия у Вашего ребенка будет формироваться нестандартные решения. В будущем такой навык пригодится Вашему чаду для того, чтобы находить возможные выходы из разнообразных ситуаций.

И самое главное, математические задачки и ребусы подарят Вашему ребенку массу положительного настроения. Если же он будет разгадывать такие головоломки с друзьями или с Вами, сможет дополнительно социализироваться и укрепить отношения.

Теперь давайте разберемся с тем, как правильно решать математические ребусы. Красочные картинки с изображением каких-то определенных предметов, цифр, знаков и букв, постоянно вызывают у детей «бешеный» интерес. Но такие картинки, как правило, кажутся им сущим хаосом. И все потому, что дети не знают, как правильно решить ребусы.



Соответственно им кажется, что такие картинки не имеют смысла. Но ведь это можно легко исправить, если внимательно изучить главные правила решения этих головоломок:

  • Названия картинок, которые зашифрованы, представлены только именительного падежа. Когда Вы будете смотреть на картинку с предметом, думайте о том, какое именно название может быть у этого изображения. Соответственно, если Вы увидите на картинке глаз, то может быть на картинке будет зашифровано «око». Никогда не останавливайтесь на одном ответе.
  • Если на картинке изображена запятая, значит у данного слова необходимо убрать какую-то определенную букву или одновременно несколько. Все будет зависеть от того, где находится запятая: перед изображением или же после него.
  • Зачастую в головоломках подобного рода встречаются буквы, которые подчеркнуты. Это решить очень легко. Вы отгадываете слово на картинке, а после этого убираете те буквы, которые подчеркнуты. Если на картинке изображены подчеркнутые числа, тогда Вам нужно убрать буквы, которые соответствуют порядковому номеру.
    Если стоят числа и буквы около не подчеркнутого изображения, тогда Вам необходимо оставить лишь данные буквы.
  • Если на картинке стоит значение Б = Р, тогда Вам нужно буквы «Б» заменить на букву «Р». Если Вы увидите вот такое равенство 2 = О, тогда в слове замените вторую букву на «О». Также на картинке может присутствовать стрелочка, к примеру, от первой буквы к третьей, тогда их просто нужно заменить друг дружкой.
  • Есть такие картинки, которые изображены в перевернутом состоянии. Тогда прочитайте слово с конца.
  • Встречаются математические ребусы, в которых есть дробь . Они легко расшифровываются: нужно вставить предлог «на». Если в знаменателе есть «2» — это означает «пол». В некоторых случаях Вы можете заметить, что во внутренней части буквы находится слог либо буква. Трактуется это так: например, если внутри буквы «О» находится «Да», тогда эта картинка означает «Вода».

Есть и другие правила, которые помогут Вам научиться разгадывать сложные головоломки либо числовые ребусы. Но с ними ребенок должен познакомиться после того, как научится разгадывать простые задачи.



Почаще проводите свое свободное время с детьми. Разгадывайте с ними ребусы, научите их находить решения к этим ребусам, так как это оказывает положительное воздействие на мозговую деятельность развивающего организма.

Математические ребусы с ответами для детей 1 класса: фото, решение, описание

Если Ваш ребенок начнет решать логические задачи с 1-го класса, у него быстрее будет развиваться сообразительность, мышление, умение делать правильные выводы и выполнять анализ. Именно подобный подход к увеличению математических возможностей имеет самую большую положительную сторону для формирования правильного мышления у деток.

Все мы знаем, что программа, составленная для школы, предполагает, как правило, лишь обучению деток решать определенные виды задач. Ученые утверждают, что важнее, чтобы первоклассник с самых первых школьных шагов смог обучаться отлично мыслить и правильно рассуждать. Они также подтвердили, что нестандартные задачки, которые необходимо решать, включив смекалку и немного мышления, очень часто ставят в затруднительную ситуацию и тех ребят, которые в школе учатся только на отлично.

Мы предлагаем Вам большое число математических ребусов для школьников. Решайте их вместе с детьми, находите совместно правильные решения, отдыхайте так, чтобы ребенку было интересно.

Цифры, которые одинаковые, обозначены на картинке одинаковыми элементами. Различные числа – разными.



Первый ребус (первоисточник смотрите )

Подумайте вместе, какое именно число фокусник решил превратить в змею?

Решение:

В первом примере змея и черепаха могут скрывать такие пары чисел: 0 – 4 либо 1 – 3. Теперь сложите эти числа. В первом случае у Вас получится 4, во втором – тоже 4.

Во втором примере ребуса подходит лишь второе сочетание чисел, так как если от 3 отнять 2 получится 1.

Ответ: за змейкой спрятана единица.



Решение:

В слове «кость» вместо «О» поставьте «И», а последнюю букву вообще уберите. Во втором слове вместо «И» поставьте «А».

Соедините эти два слова.

Ответ:

Кисточка.



Решение:

На картинке изображена лейка. Перед этим словом поставьте «К», а две последние «К» и «А» уберите.

Ответ:

Четвертый ребус:



Решение:

На картинке изображена тучка. Впереди этого слова поставьте «Р», а первую букву «Т» уберите.

Ответ:

Математические ребусы с ответами для детей 2 класса: фото, решение, описание

Во 2-ом классе программа сложнее, чем в 1-ом. Процесс обучения становится более трудоемким, соответственно Вам нужно своему чаду помочь.

Конечно же, учеба нужна, но нельзя сильно перегружать школьника. Программы, которая дается в школе, и домашнего задания, будет достаточно. Есть такие школьники, которые в школе учатся замечательно, а когда они приходят домой, начинают отказываться делать уроки.

Но Вы знаете, что детям обязательно необходимо повторять пройденный материал в школе, изучать что-то новое, улавливать новые для них слова, развивать собственное мышление и так далее. Возможно, Вы думаете, что ребенок во 2-ом классе уже стал взрослее, начинаете подавать ему много новой информации в виде дополнительных уроков, а потом удивляетесь, почему Ваши старания не дают положительных результатов.

Дело в том, что Ваш малыш устает в школе, он хочет немного поиграть и хорошенько отдохнуть. Поможет ему в этом игра, к примеру, математические ребусы. Существует большое количество таких головоломок. Но есть родители, которые ошибаются, подбирая развлекательную головоломку не по возрасту.

Не делайте и Вы этого. Внимательно изучите те варианты математических ребусов, которые предлагаем Вам мы. Они предназначены именно для школьников 2-го класса.

Решение:

На картинке изображен ключ. В этом слове уберите последние две буквы. А в конце самого слова поставьте «ЫК».



Ответ:



Решение:

На картинке изображен зонт. Уберите в слове две последние буквы. Перед словом поставьте «У» и в конце поставьте «Р».

Ответ:



Решение:

На картинке изображен лист. Вместо буквы «Л» поставьте букву «А».

Ответ:

Математические ребусы с ответами для детей 3 класса: фото, решение, описание

Ребусы, которые предназначаются для школьников 3-го класса, могут разделятся на некоторые виды. Все зависит от дисциплины в школе, к которой эти головоломки относятся. Также они могут разделяться по уровню сложности.

Учителя уже неоднократно доказывали то, что математические ребусы помогают ученику эффективнее усваивать процесс обучения. Они утверждают, что благодаря таким ребусам ребенок начинает хорошо мыслить и развивает в себе творческую способность. А еще математические ребусы помогают улучшить настроение для того, чтобы изучать новые предметы.

Очень трудно выделить те ребусы, которые подходят для ученика 3-го класса. Мы Вам хотим предложить некоторые варианты, которые Вы сможете разгадать со своим ребенком.



Решение:

На картинке изображен ромб. Уберите последние две буквы «М» и «Б». спереди слова поставьте «К», а в конце «Т».

Ответ:



Решение:

На картинке изображен дом. Уберите первую букву «Д». Впереди слова поставьте букву «Л».

Ответ:

Решение:



На картинке изображен перевернутый дом. Это означает, что слово нужно прочитать с конца. Добавьте еще в конце слова букву «А».

Ответ:

Четвертый ребус:



Четвертый ребус

Решение:

В этом варианте математического ребуса изображены буквы и цифры. Вам нужно поступить следующим образом: вместо цифры 100 напишите буквами, а потом соедините все буквы.

Ответ:

Математические ребусы с ответами для детей 4 класса: фото, решение, описание

Школьники в 4-ом классе уже начинают знакомиться с пространственным представлением. Дети изучают поверхностно геометрические фигуры и их простые свойства, начинают постепенно выполнять легкие чертежи, применяя при этом примитивные приборы для измерения. Именно в этот период времени у детей начинает формироваться основа для будущего обучения.

Школьники переходят к более сложной науке, которая очень скоро поделится на пару курсов: первый курс – это алгебра, второй – это геометрия. Зачастую, чтобы ученики немного отдохнули от тяжелого урока, учителя применяют дополнительные задания, например, математические головоломки и ребусы. Предлагаем Вам некоторые из них, которые, возможно, Вы разгадаете со своим ребенком.



Решение:

На картинке Вы видите слово и изображение предмета «нож». Вместо цифры 100 напишите слово «сто». Спереди слова «нож» уберите первую букву. Соедините все буквы.

Ответ:



Решение:

На картинке изображен гриб. Уберите спереди слова первую букву. Вместо буквы «И» поставьте букву «Ы». В конце слова поставьте «КА».

Ответ:



Решение:

На картинке изображен лист и гусь. В первом слове поменяйте местами буквы, как указано на картинке. Во втором слове уберите три первые буквы. Потом попробуйте прочитать то, что у Вас получилось.

Ответ:

Математические ребусы с ответами для детей 5 класса: фото, решение, описание

Для учеников, которые уже перешли в 5-ый класс и выше, существуют свои усложненные математические ребусы. Над ними дети должны серьезно поработать, чтобы отыскать правильный ответ. Если такого не произойдет, задачки просто не заинтересуют ребят и тогда они не принесут пользы.

Для пятиклассников предлагаем Вам вот такие ребусы:



Решение:

На картинке изображена оса и дробь. Так как у нас тут присутствует дробь, значит решение такое: под буквой «Н» находится оса. От слова «оса» отнимите последнюю букву. А далее сложите под + н + ос (последняя буква уже отсутствует).

Ответ:



Решение:

Сочетание «ЗА» находится в букве «А». Решение такое: в + а + за.

Ответ:

Математические ребусы с ответами для детей 6 класса: фото, решение, описание

В 6-ом классе детки уже становятся совсем взрослыми. Это означает, что и математические головоломки должны быть посложнее.



Решение:

На картинке изображен перевернутый гриб и оса. Поступите следующим образом: слово «гриб» прочитайте наоборот. В этом же слове вместо буквы «Г» поставьте букву «К». От слова «оса» отнимите первые две буквы. Сложите оставшиеся буквы.

Ответ:



Решение:

Здесь чтобы отыскать решение, ребенку придется немного подумать. Не говорите ему сразу ответ. Пусть Ваш школьник подумает над ответом сам, а Вы послушайте, какое именно решение он предложит Вам.

Ответ:

Математические ребусы с ответами для детей 7 класса: фото, решение, описание

Как правило, в 7-ом классе у детей начинается алгебра и геометрия. Они уже знакомы со многими геометрическими фигурами, у них лучше развито мышление, чем у школьников начальных классов. Это означает, что для таких детей нужны математические ребусы с высокой степенью сложности.



На картинке изображено сочетание букв и цифр. Вместо цифры 100 напишите слово «сто». Теперь соедините все буквы. Правда придется немного подумать.



На картинке изображена цифра 7, буква «К» и рот. «7» напишите словом «семь» и отнимите от него две последние буквы. Рот изображен перевернутым. Значит Вам нужно его прочитать наоборот с конца.



На картинке изображено перо с метром. Запятая говорит о том, что Вам нужно убрать последнюю букву от слова «перо». Все очень просто. Соедините те буквы, которые остались от слова «перо» с буквой «И» и словом «метр».

Видео: Ребус с ответами для школьников

Инструкция

Прежде чем приступить к разгадыванию сложных задач, потренируйтесь на простом примере: ВАГОН+ВАГОН=СОСТАВ. Запишите его в столбик, так будет удобнее решать. Вы имеете два неизвестных пятизначных числа, сумма которых шестизначное число, значит В+В больше 10-ти и С равно 1. Замените символы С на 1.

Сумма А+А – однозначное или двухзначное число с единицей на конце, это возможно в том случае, если сумма Г+Г больше 10 и А равно либо 0, либо 5. Попробуйте предположить, что А равно 0, тогда О равно 5-ти, что не удовлетворяет условиям задачи, т.к. в этом случае В+В=2В не может равняться 15-ти. Следовательно, А=5. Замените все символы А на 5.

Сумма О+О=2О – четное число, может быть равна 5 или 15 лишь в том случае, если сумма Н+Н – двухзначное число, т.е. Н больше 6-ти. Если О+О=5, то О=2. Это решение неверно, т.к. В+В=2В+1, т.е. О должно быть число нечетное. Значит, О равно 7-ми. Замените все О на 7.

Легко заметить, что В равно 8-ми, тогда Н=9. Замените все буквы на найденные числовые значения.

Замените в примере оставшиеся буквы на числа: Г=6 и Т=3. Вы получили верное равенство: 85679+85679=171358. Ребус отгадан.

При вычитании также начните действия с единиц. Если число того или иного разряда уменьшаемого меньше числа вычитаемого, то займите у следующего разряда 1 десяток или сотню и т.д. и произведите вычисления. Поставьте точку над числом, у которого занимали, чтобы не забыть. При выполнении действий с этим разрядом вычитайте уже из уменьшенного числа. Результат запишите под горизонтальной чертой.

Проверите правильность вычислений. Если вы складывали, тогда из полученной суммы вычтите одно из слагаемых, у вас должно получиться . Если же вы вычитали, тогда сложите полученную разность с вычитаемым, должно получиться уменьшаемое.

Обратите внимание

Обязательно разряды чисел должны находиться друг под другом.

В линейной алгебре и в геометрии понятие вектор определяется по разному. В алгебре вектор ом называется элемент вектор ного пространства. В геометрии же вектор ом называют упорядоченную пару точек евклидового пространства – направленный отрезок. Над вектор ами определены линейные операции – сложение вектор ов и умножение вектор а на некоторое число.

Инструкция

Произведением вектор а a на число? называется число?a , что |?a| = |?| * |a|. Полученный при умножении на число вектор параллелен исходному вектор у или лежит с ним на одной прямой. Если?>0, то вектор ы a и?a однонаправленными, если?вектор ы a и?a направлены в разные .

Видео по теме

Ребус – это особенная загадка, в которой искомое слово заключено в рисунках, содержащих различные буквы и цифры. На картинках вы можете встретить также и другие знаки, которые помогут прочитать слово правильно. Решение ребусов – это весьма увлекательное занятие, которое поможет вам размяться перед сложной работой. Чтобы , вы должны помнить ряд простых правил.

Инструкция

Названия любых предметов, изображенных на рисунке, читаются только в именительном падеже.

Иногда рисунок может иметь несколько названий (например, лапа или нога). А также предмет может иметь, как конкретное, так и общее название. Например, цветок является общим названием, а конкретное – или роза. Поэтому, если вы сможете правильно угадать объект, изображенный на картинке, то считайте, что самая сложная часть позади. Самый простой и популярный метод решения ребусов – рисунков по частям. То есть сначала нужно записать все названия предметов по порядку, а затем сложить из них текст.

Справа от предмета могут быть нарисованы одна или несколько перевернутых запятых – это значит, что нужно убрать одну или несколько букв в начале или конце слова соответственно.

В том случае, если над картинкой есть цифры, буквы в слове необходимо читать в определенном порядке – именно в том, в котором стоят цифры.

Математика – довольно непростая наука , однако усвоить ее азы нужно каждому. Без этих навыков и знаний в современном мире никуда.

Элементарные математические приемы и задачи закладываются в память школьников еще в младших классах. А «упустив» более легкий материал, решить сложные задания становится не под силу. Долгие и серьезные уроки математики делают детей особо неусидчивыми, а значит подавать информацию нужно в игровой форме, например, с помощью ребусов . Такие задания не нужно заставлять решать из-под палки, детки сами охотно будут браться за их разгадывание.

Главное в статье

Польза ребусов на математическую тему для развития ребенка

Ребусы на математическую тему – это те же загадки и головоломки, в которых используются рисунки и графика. Они бывают разные по уровню сложности в зависимости от возрастной категории школьников.


Правила составления математических ребусов для детей

  1. Если вы видите перед словом или картинкой запятую , то нужно убрать первую букву с этого названия . То же самое нужно сделать, если запятая стоит в конце слова. Когда около картинки две запятых, то убирается две буквы соответственно. Например, на первой картинке изображен сок — нужно убрать первую букву «С», рука — уберите слог «ка», буква «ж» так и остается, нос — слово остается целиком, пять — уберите две первые буквы. Зашифрованное слово — «окружность» .
  2. Если цифры , обозначающие последовательность букв в слове зачеркнуты, то их необходимо выбросить из него . Тоже самое касается и букв. На втором рисунке изображен цирк — уберите последнюю букву, из слова «акула» нужно убрать букву «А», готовый ответ: «циркуль».
  3. Когда рядом с картинкой стоят цифры, поменянные местами , то и в названии самого предмета нужно поменять местами буквы, которые стоят в последовательности с указанными цифрами.
  4. Если картинка изображена вверх тормашками , то отгадку нужно читать в обратном порядке: справа-налево.
  5. Для ребусов используется только именительный падеж в словах .
  6. Указатель в виде стрелки или математический знак «равно» обозначает, что нужно заменить буквы одну другой.
  7. В ребусах одно значение может быть расположено внутри другой картинки , за ней или под ней. Тогда применяйте слова: В, НА, НАД, ПОД, ЗА.
  8. Цифры, стоящие в ряд около изображения , обозначают, что нужно использовать из этого значения буквы в указанной последовательности цифр.

Вот несколько примеров математических ребусов, соответствующих приведенным правилам:

Под третьим рисунком зашифровано слово «вектор» , под четвертым — «степень» , под пятым — «два» , под шестым — «доказательство» .

Как придумать математический ребус?

Следуя общим правилам составления ребусов, попробуйте придумать для начала несложные математические задачки, используя цифры и математические термины. А затем, немного освоив простые задания, переходите к более усложненным. Вот несколько образцов ребусов по математике с ответами, которые вдохновят вас и покажут, как их нужно составлять:

Ответы: первый ребус — «диаметр» , второй — «пять» , третий — «конус» , четвертый — «задача» .


Пятая картинка — «алгебра» , шестая — «геометрия» , седьмая — «линейка» , восьмая — «уравнение» .


Девятая загадка — «диаметр» , десятая — «циркуль» , одиннадцатая — «транспортир» , двенадцатая — «конус» .



Особенности математических ребусов для начальной школы

Лучше всего приобщать ребенка к разгадыванию математических ребусов еще в детском саду, в выпускной группе. Это послужит отличной разминкой перед школой, освежит у малыша весь пройденный материал с педагогом.

Только нужно учитывать, что такие ребусы должны быть довольно легкими, и включать только те знания, которые ребенок уже усвоил и знает. Это может быть головоломка из двух-трех составляющих, ответ которой таит в себе простое математическое значение.

Эти же ребусы пригодятся для «разогрева» первоклашек. Поступление в школу – и так огромная эмоциональная нагрузка для ребенка, поэтому не стоит удручать обучение математике столь сложными ребусами. Подойдут следующие примеры:


Математические ребусы для 1 класса с ответами

Первоклассники уже хорошо знают цифры и простые математические действия, которые можно включить в ребусы. Причем для таких ребусов характерно то, что математическое значение может присутствовать как в самой загадке, так и в ее значении. А может случиться такое, что ответ совершенно не будет связан с этой точной наукой. Предложите ребенку следующие математические ребусы:

Математические ребусы для 2 класса с ответами

Для того, чтобы составить математический ребус второкласснику, нужно ориентироваться в его знаниях, то есть предлагаемая задача должна быть ему посильной. Вот что должен знать и уметь учащийся во втором классе:

  1. При решении заданий использовать в правильном порядке числа от 1 до 100, правильно озвучивая их.
  2. Решать примеры сложения и вычитания чисел, которые не превышают цифру 20.
  3. В ряде случаев применять математические действия умножения и деления.
  4. Четко знать правила использования скобок в примерах и решать их.
  5. Применять в своей лексике единицы измерения длины и объема.
  6. Вести сравнения больше-меньше цифр в пределах 100.
  7. Уметь устно прибавлять и отнимать числа в пределах 100.
  8. Решать несложные задачи с четырьмя основными арифметическими действиями, уметь увеличивать (уменьшать) число на (в) раз (единиц).
  9. С помощью линейки чертить и мерить длину отрезка.
  10. Распознавать плоские углы.
  11. Узнавать и озвучивать плоские геометрические фигуры.
  12. Уметь вычислять периметр многоугольников.






Математические ребусы для 3 класса с ответами

Чтобы разгадать посильные математические ребусы, третьеклассник на уроке математики должен:

  1. Считать и называть числа до тысячи.
  2. Выполняя основные четыре арифметические действия, называть каждую составляющую примера своим названием.
  3. Владеть таблицей умножения и оговаривать результат действия деления.
  4. Уметь решать примеры со скобками и без них.
  5. Знать единицы измерения величин и выражать их в разной интерпретации.
  6. Устно решать математические действия до значения 100.
  7. Делить многозначное число на однозначное, руководствуясь таблицей умножения.
  8. Проверять правильность расчета примеров.
  9. Выполнять задачи на одно-два действия.
  10. Придумывать задачи, обратные исходной.
  11. Уметь кратко записать задачу.
  12. Вычислять уравнения и неравенства.
  13. Чертить простые геометрические фигуры, согласно исходным данным задания, вычислять их периметр и площадь.
  14. Уметь пользоваться циркулем, чертя окружности заданных радиусов.





Математические ребусы для 4 класса с ответами

На уроках математики четвероклассник должен:

  1. Уметь решать задачи рациональным и нерациональным способом.
  2. Решать задачи, записывая ход их решения.
  3. Иметь представление вычисления объема и площади геометрических фигур, исходя из выученных формул.
  4. Чертить геометрические фигуры, обозначать их компоненты латинскими буквами.
  5. Строить и мерить углы транспортиром.
  6. Знать свойства равенства.
  7. Решать задания с количеством арифметических действий от одного до четырех.
  8. Знать свойства сторон, углов, радиусов геометрических фигур.
  9. Вычитать и прибавлять многозначные числа.
  10. Делить многозначное число на однозначное и многозначное.
  11. Иметь понятие натурального ряда.
  12. Умножать дробь на натуральное число.
  13. Правильно называть и писать дроби: числитель и знаменатель.
  14. Сравнивать дроби.




Математические ребусы для 5 класса с ответами

Программа по математике для пятиклассника схожа с предыдущим годом, только имеет более обширный характер. Недаром ведь в некоторых школах четвертый класс пропускается, а вся школьная программа за пропущенный год изучается в пятом классе.





Математические ребусы для 6 класса с ответами
  1. В шестом классе активно изучается геометрия, в частности ее теоремы.
  2. Ребенок знакомится с известными учеными в области математики и других точных наук.
  3. Школьник имеет дело с изучением геометрических фигур на плоскости, учится вычислять их объем и площадь по изученным формулам.
  4. По алгебре в ход идет решение уравнений с двумя неизвестными, неравенств.




Математические ребусы с цифрами с ответами

Цифры, изображенные в математических ребусах, могут быть двух видов:

  • Те, название или часть названия которых используется для ответа.
  • Те, которые стоят около изображения, и указывают на то, что из названия этого изображения нужно позаимствовать буквы, соответствующие последовательности стоящих цифр в ряду.


Математические загадки, ребусы, кроссворды

Хорошо тренируют умственную активность не только ребусы по математике, но еще и логические, арифметические загадки, кроссворды. Они развивают любознательность и сообразительность у детей. А игровая форма заданий помогает достигнуть высокой скорости мышления и догадки.

Для самых маленьких подойдут такие задачки:


Решите еще такие кроссворды и задания:

  • Решите примеры, линиями соедините ответ и группу детишек, соответствующую ему (первое задание).
  • Решите примеры на веслах, а затем линиями соедините каждое из них с лодками, имеющими правильный ответ (второе задание).

  • Заполните пропущенные клеточки цифрами таким образом, чтобы по горизонтали и по вертикали всегда ответ получался 15 (третье задание).
  • Заполните пропуски и решите примеры (четвертое задание).

Разгадайте кроссворды:

Вот более сложные ребусы:



Как решать математические ребусы с буквами?

Решение математических ребусов с буквами

Все слова состоят из букв, поэтому множество ребусов содержат в своей структуре буквы. Руководствуясь основными принципами решения ребусов, вы с легкостью осилите математические ребусы с буквами.




Математические ребусы и головоломки

Такие загадки и головоломки будут интересны не только школьникам, но и их родителям:




Самые легкие математические ребусы

Пусть школьник потренируется для начала на простых математических ребусах. К примеру, на таких:


Сложные математические ребусы

Попробуйте предоставить вашему сорванцу вот такие головоломки, которые позволят сконцентрировать смекалку и потренировать интеллект. Это задание предположительно для учеников пятых классов.

В нашей статье приведены примеры математических ребусов с ответами разных уровней сложности, зависящих от возраста школьника. Изучив основные правила разгадывания ребусов, попробуйте составить интересные задания своим деткам. Такого рода занятия помогут ребенку активизировать свои интеллектуальные способности, выработают усидчивость и концентрацию внимания, а также закрепят пройденный материал по математике. Это увлекательное занятие поможет сплотить родных (товарищей), и создать дружескую атмосферу в семье и школьном коллективе.

Ребус – уникальное изобретение человечества, помогающее воспитывать у людей остроту ума, сообразительность, смекалку. Взрослые иногда любят побаловаться решением таких задачек в свободное время, но больше всего удовольствия ребусы доставляют для детей. Чтобы совместить приятное и полезное, предлагаем вам разгадывать ребусы с цифрами для детей, которые даются на нашем сайте с ответами.

Ребусы направлены на логическое развитие ребенка.

Как их решать?

Математические ребусы не являются задачками, к которым мы привыкли в школе, хотя некоторые элементы подобных действий они все же могут содержать. Давайте вспомним, как выглядит традиционный ребус.

Берется какое-нибудь слово для зашифровки. Далее оно делится на части и зашифровывается каждая из частей. Разгадав каждую часть ребуса в отдельности, необходимо сложить слово.

Математические ребусы могут быть как лингвистического, так и числового характера. Например, в задачке путем математических действий можно вычислить необходимую цифру. Если же математические ребусы с числами для детей зашифрованы словами, тогда задача упрощается.

Подборка материалов по теме


Ответы к этому ребусу: стриж, семья, сорока, столб.

Как можно их использовать?

Решать ребусы можно на уроках с детьми младшего школьного возраста, а также дошкольниками в детском саду или эстетическом центре, если они уже знают цифры и умеют в них ориентироваться. В школе можно подключать к работе ребусы с римскими числами, хотя разгадывать их детям пока будет труднее.

Конечно, строить математические занятия полностью на ребусах нельзя. Но урок можно значительно разнообразить, если после нескольких трудных заданий предложить для детей веселый ребус. Если занятия проходят в детском центре или садике, то математические ребусы для детей можно предлагать ежедневно, между играми или другими видами деятельности. Конечно, они должны быть привязаны к изучению цифр, так как дети в этом возрасте еще плохо ориентируются в числах.

Математические ребусы можно давать ребятам на дом, конечно, с тем учетом, что дома им помогут родители. В школе на открытом уроке, если учитель прибегнет к такого рода заданиям, его наверняка ждет успех.

Как же разгадывать математические ребусы? Приведем несколько примеров.

Итак, первая часть слова в ребусе зашифрована в виде слова «очки», в котором нужно убрать первую и третью буквы. Так мы получаем «чи». Далее от слова «слон» отнимаем последнюю букву. Получаем слово «число».

Еще один ребус. Первая часть слова – это нота, находящаяся посередине первой линии на нотном стане («ми»). Вторая часть слова – это «нос», в котором вторая буква равна «у». Если сложить все вместе, то получится «минус».

Итак, ребус не сложный, и понять принцип его построения младшие школьники тоже могут. Когда дети освоятся с ребусами, можно предложить им самим придумать математические ребусы. Ребята обожают такие задания. Когда все придумают хотя бы по одной-две задачи, попросите остальных отгадать. Для этого малыши должны нарисовать картинки к своим ребусам на листах бумаги или на доске.

Еще один вариант использования ребусов – это подготовить конкурс работ детей. Это можно сделать в неделю математики или при подготовке к празднику. Работы с ребусами повесьте на видное место, например, в холле или актовом зале. Для родителей будет очень интересно посмотреть детские работы и попробовать их разгадать. Ребусы с ответами лучше не вешать, чтобы не лишать зрителей интриги.

Видео по теме

Выводы

Ребусы – очень полезные задания для детей, особенно, если они способны научить новому. Математические задачки не только позволяют повторить материал по числам, но и развить смекалку и сообразительность .

Дети – очень мобильные и любопытные существа. Ребусы способны пробудить их фантазию и острый ум, который наверняка найдет решение проблемы. Подбрасывайте ребятам больше пищи для ума, стимулируйте процесс мышления, творческие способности. Пусть математика тесно переплетается с филологией и логикой, ведь взаимодействие предметов позволяет с детства ощутить связь различных дисциплин, что так необходимо для формирования целостной картины мира.

Математические ребусы с цифрами для детей. Дидактический материал по математике. Цифра «7 Наглядные и видео пособия

Ребус – логическая игра, в которой надо разгадать ответ по картинке. На последней изображаются предметы, животные и растения, буквы и цифры. Имеет значение их взаимное расположение. Даже для непосед ребусы могут стать увлекательным занятием, если подать его в игровой форме. Например, можно предложить научить ребенка, как разгадывать шпионские шифровки.

И от самых простых картинок-головоломок для дошкольного возраста добраться до сравнительно сложных. Уверяем: если ваше чадо увлечется и научиться включать логическое мышление, со временем уже вы будете учиться у него, как решать загадки в картинках.

Ребусы придуманы на огромное множество тем. Главное, чтобы каждое слово, буква и предмет, служащие ответом к картинке, были уже знакомы малышу.

Как решать ребусы для детей с буквами в картинках?

Если вы заинтересовались ребусами, то, скорее всего, знаете преимущества этих логических задачек. Они развивают память, сообразительность, скорость мышления, умение ориентироваться в ситуации и применять уже полученные знания.

Чтобы научить ребенка 6-7 лет, как правильно решать задания, для начала начала объясните ему правила. Не нужно настаивать, чтобы он запомнил все сразу. Скорее всего, вы и сами их все не знаете. Лучше за день объяснять одно-два и подкреплять тематическими заданиями. Последние можно распечатывать (удобнее для занятий на улице) или показывать с монитора. На следующих занятиях тоже лучше не предлагать слишком много материала. Важно объяснить чаду, что сначала ему нужно правильно идентифицировать и назвать предмет, изображенный на картинке. И только потом в отношении этого слова применять правила.

Итак, почитаем основные правила! В частности, определимся, что означает запятая, перечеркивание, перевернутый предмет и другие тонкости в картинках.

  • Что значит запятая в начале или конце ребуса?
    Запятая внизу или вверху перед картинкой значит, что от названия изображенного предмета надо отбросить одну букву в начале. Соответственно, видим две запятые – отбрасываем две первые буквы. Эти значки встречаются очень часто.
  • Что значит перевернутая запятая в начале или конце?
    Правила для перевернутых запятых, аналогичны правилам для обычных запятых (смотри предыдущий пункт).
  • Что означают зачеркнутые и дописанные буквы?
    Зачеркнутая буква на картинке значит, что из названия нарисованного предмета ее нужно исключить (и добавить другую, если она указана). Дописанная слева или справа от картинки – надо добавить ее к слову в начале и в конце.
  • Что означают цифры в ребусах?
    Цифры могут иметь два значения. Они стоят над словом? Чтобы отгадать ответ, надо переставить буквы с места на место в обозначенном порядке. Название цифры может быть частью слова (часто используют «сто», «пять»). Зачеркнутая цифра значит, что из слова нужно исключить букву с таким порядковым номером. Следует помнить, что некоторые цифры, так же как и предметы, могут иметь несколько названий (единица – «кол», «раз», «один»).
  • Что означает плюс, знак «равно»?
    Если между словами (символами) стоит знак «плюс», значит, их надо добавить друг к дружке. Иногда «+» значит предлог «к», нужное выбирается по смыслу. Знак «равно» (например, А=К) говорит о том, что все буквы «А» в слове следует заменить на буквы «К».
  • Вертикальная или горизонтальная черта в заданиях?
    Горизонтальная черта значит одновременно «под», «по», «над» и «на» в зависимости от контекста. Используется с буквами или картинками, когда одна часть нарисована ниже черты, другая – выше. Иногда обозначает дробь (половину чего-либо, то есть «пол-»).
  • Расположение букв на картинке и предлоги
    Важно посмотреть на взаиморасположение букв. Если они размещены одна в другой, значит, к их названиям добавляется предлог «в». Одна буква нарисована за другой – имеется в виду предлог «за» или «перед».
  • Предмет на картинке нарисован вверх ногами ? Чтобы получить ответ, надо прочитать слово наоборот. Короткие слова дети 6-7 лет вполне могут перевернуть в уме. Правда, количество таких заданий довольно ограничено.

Чаще всего в ребусах используется одновременно несколько правил. Считается, что в 6-7 лет дети уже знакомы с буквами, четко знают их названия. Если младший школьник еще не сталкивался с запятыми, научить его новому значку не составит особой сложности.

Примеры ребусов в картинках для детей 6-7 лет с ответами

Дети 6-7 лет и меньше гораздо лучше воспринимают материал в привязке к какому-либо запоминающемуся событию. Ребусы про животных будут разгадываться с восторгом, если предложить их ребенку на следующий день после посещения зоопарка. Девочке-первоклашке, которая горит желанием поступать в музыкальную школу, будут интересны музыкальные ребусы. А ребенку, впечатленному планетарием мальчику, понравятся картинки про космос.

Про животных и о птицах

Давая детям задание про птиц или животных, убедитесь, что он уже сталкивался с такими названиями животных, а также понимает все, что изображено на картинке.

Ребусы о семье, о маме

Кто для ребенка милее всех, если не мамочка! А кого он с радостью встречает каждый раз, кроме мамы и папы? Деткам очень понравится узнавать и угадывать в зашифрованных картинках бабушку, дедушку, сестричку и других родных. Распечатайте или нарисуйте картинки поярче и начинайте развлекаться, заодно обучая ребенка!

О спорте, о здоровье

Ребусы о труде, здоровье, спорте, профессиях и многие другие можно использовать в качестве тематических игровых пособий. В выпускной группе детсада, первых классах школы или дома намечается занятия либо беседа на одну из тем? Загадка в виде картинки позволит усвоить материал лучше, чем обычный безликий рассказ. Малышей заинтересует нестандартная подача материала.

Ребусы по сказкам

Сказки со знакомыми героями, современные или классические мультфильмы – неисчерпаемый кладезь вдохновения. Если ребенок не очень интересуется логическими загадками, можно попробовать увлечь его отгадыванием любимых персонажей. Загадок на эту тему гораздо больше, чем приведено в качестве примера. Зная интересы и любимые сказки вашего ребенка, вы можете создать ребусы в виде аппликаций самостоятельно.

Знакомимся с числом и цифрой «7» , используем для знакомства различный дидактический материал .

Дидактический материал по математике.

Цифра «7»

Веселые стихи

Цифра семь – подъемный кран –Заглянула в гости к нам.Семь цветов с собой взяла,

В небе радугу зажгла.

Семь ночей и дней в неделе.

Семь вещей у нас в портфеле:

Промокашка и тетрадь.

Ручка, чтобы ей писать,

И резинка, чтобы пятна

Подчищала аккуратно,

И пенал, и карандаш,

И букварь – приятель наш.

Я легко смогу отнять

От семи и три, и пять.

Но отнять у Саньки

Не могу я санки.

В. Ланцетти

Над лесами, над рекойСемицветный мост дугой.Если б мог я встать на мост,

Я б рукой достал до звезд.

В деревушке семь избушек,

Семь крылечек, семь старушек,

Семь щенков, семь дымков,

Семь драчливых петухов

На семи плетнях сидят,

Друг на друга не глядят.

Распустили семь хвостов.

Каждый хвост семи цветов.

А. Стройло

Семь – точно острая коса,

Коси, коса, пока роса.

Г. Виеру

У нее одна нога,

Да и та без сапога!

Загадки

Кнутом не гонят,Овсом не кормят;Когда пашет –

Семь плугов тянет.

(Трактор.)

Что за птицы пролетаютПо семерке в каждой стае?Вереницею летят,

Не воротятся назад.

(Дни недели.)

Нас отара, семь баранов,Защищает от буранов.

(Шуба.)

Приказало солнце: «Стой,

Семицветный мост крутой!».

(Радуга.)

Семь ребят на лесенке

Заиграли песенки.

(Ноты.)

Братцев этих ровно семь.Вам они известны всем.Каждую неделю кругом

Ходят братцы друг за другом.

Попрощается последний –

Появляется передний.

(Дни недели.)

Веселые задачки

Дружно муравьи живутИ без дела не снуют.Два несут травинку,

Два несут былинку,

Три несут иголки.

Сколько их под елкой?

Пять ворон на крышу сели,

Две еще к ним прилетели.

Отвечайте быстро, смело,

Сколько всех их прилетело?

Как-то вечером к медведю

На пирог пришли соседи:

Еж, барсук, енот, косой,

Волк с плутовкою-лисой.

А медведь никак не мог

Разделить на всех пирог.

Помоги ему скорей,

Посчитай-ка всех зверей.

Слон, слониха, два слоненкаШли толпой на водопой.А навстречу три тигренка

С водопоя шли домой.

Сосчитайте поскорей,

Сколько встретилось друзей?

Дядя ёжик в сад зашел,

Десять спелых груш нашел.

Три из них он дал ежатам,

Остальные же – зайчатам.

(Сколько груш он дал зайчатам?)

На опушке в день весенний

Под кудрявою листвой

К Белоснежке в день рожденья

Собрался народ лесной:

Гном Тихоня, Безбородый,

Мудрый, Рыжий и Ворчун,

Забияка и Молчун.

Сколько было там народа?

В. Усачева

  Наша Маша рано встала,Кукол всех пересчитала:

Две Матрешки на окошке,

Две Аринки на перинке,

Две Танюшки на подушке,

А Петрушка в колпачке

На дубовом сундучке.

(Сколько всех?)

Белка, ежик и енот,

Волк, лиса, малышка крот

Были дружные соседи.

На пирог пришли к медведю.

Вы, ребята, не зевайте:

Сколько всех зверей, считайте

  Мама-белка для детишекСобрала десяток шишек.

Сразу все не отдала,

По одной всего дала.

Старшему – еловую,

Среднему – сосновую,

Младшему – кедровую.

(Сколько шишек осталось у белки?)

К медвежонку в день рожденья

Собрался народ лесной:

Ежик, волк, енот, барсук,

Заяц, лось, лисенок – друг.

Посчитайте поскорей,

Сколько было всех гостей?

  Бабушка МилаЖивотных любила,

Каждое утро ласкала, кормила:

Кошку, корову, овцу, поросенка,

Сколько животных у бабули

в семейке?

Сидят рыбаки, стерегут поплавки.

Рыбак Корней поймал трех окуней.

Рыбак Евсей – четырех карасей.

Сколько рыб рыбаки

Натаскали из реки?

  Бабушка Надя в деревне живет,Животных имеет, а счет не ведет.

Я буду, ребята, их называть,

Корова, теленок, два сереньких гуся,

Овца, поросенок и кошка Катуся.

(Сколько животных у бабушки?)

В. Дроздова

Летела стая тетеревов,

Села на рощицу;

По двое на дерево сядут –

Одно дерево лишнее.

По одному сядут –

Один тетерев лишний.

(Ответ: 4 тетерева и 3 дерева.)

Пословицы и поговорки

  • Семь бед – один ответ.
  • Семеро одного не ждут.
  • Семь раз отмерь, один раз отрежь.
  • Один с сошкой, семеро с ложкой.
  • Семь пятниц на неделе.
  • У семи нянек дитя без глазу.
  • Лук от семи недуг.

Скороговорки

  • Сидели, свистели семь свиристелей.
  • В семеро саней по семеро в сани уселись сами.
  • У Степана есть сметана, простокваша да творог, семь копеек – туесок.

Физкультминутка

Сколько елочек зеленых,

Столько выполним поклонов (7).

Сколько здесь у нас кружков,

Столько сделаем прыжков (7).

Сколько вязаных клубков.

Столько сделаем хлопков (7).

Графические задачи

З а д а н и е 1. Раздели утят.

Семь утят в пруду все время ссорятся. Как разделить их тремя линиями, чтобы прекратить ссоры?

З а д а н и е 2. Найди круги.

Найдите в каждой фигуре по 7 кругов.

З а д а н и е 3. Помоги Белоснежке найти путь.

Белоснежка решила угостить семерых гномов, которые работали в лесу. Как пройти к гномам, не заходя к каждому дважды?

Ребус

7 я

Цифра 7 очень необычная в истории русской культуры. О ней слагались сказки, пословицы, поговорки, стихи¸ загадки. Современные авторы придумывают новые истории и даже ребусы, чтобы детям интереснее было изучать эту цифру. В методиках преподавания математики для учителя, который идет на урок в 1 класс, применяются фото, картинка, презентация. Их можно скачать у нас на сайте бесплатно.

Фото-картинки

Словесные пособия

На уроках для детей, которые ходят в 1 класс, можно применять словесные пособия для изучения математики. В их качестве могут служить пословицы, поговорки, скороговорки, стихи, загадки, где упоминается число 7.

Пословицы, поговорки и загадки

Пословицы, поговорки, загадки – жанры устного народного творчества. Цифра 7 в них – необычное число. Оно всегда было связано с мотивом гармонии вселенной. Например, такие пословицы, как «Семь раз отмерь – один раз отрежь» говорят о том, что цифра 7 почиталась как самодостаточное число. Повторить семь раз какое-то действие, значит достигнуть в нем удачи. Загадки на число 7 связаны в основном с днями недели: их тоже у нас семь. Скачать загадки можно на сайте.

Конечно, посвящать много времени на уроках математики тому, что число 7 представляет собой некий идеал, и углубляться в историю нет времени. Но воспитывать детей в национальном духе просто необходимо. Для этого применяйте пословицы и поговорки. Это назидательные жанры фольклора, содержащие всю народную мудрость. Если вы будете на уроках уместно рассказывать пословицы и поговорки, дети будут воспринимать это как должное, а затем и сами начнут цитировать. Пословицы и поговорки, посвященные цифре 7, тоже будут связаны с восприятием этого числа как гармоничного, выражающего согласие человека и окружающего мира. Скачать пословицы и поговорки можно на нашем сайте.

Стихи

В какой бы класс ни ходил ребенок, ему будет полезно послушать стихи о цифре 7. Это число присутствует у многих современных авторов, но лучшими можно назвать стихи С.Маршака. Стихи на уроке математики не только способствуют большей заинтересованности детей. Стихи в умелых руках развивают детскую память, прививают любовь к родному слову, а ведь именно с помощью слов мы обращаемся к любой науке. Учитель, который готовит открытый урок в начальный класс, можно использовать стихи о цифре 7, чтобы разнообразить занятие, привлечь внимание детей и гостей. В детских развивающих центрах можно учить с ребятами наизусть несложные стихи про семерку. Чем больше вы предлагаете детям, тем больше затем от них получите. Скачать стихи современных авторов про цифру 7 можно на нашем сайте.

Интересно детям будет разгадывать ребусы. Именно ребусы развивают логическое мышление, смекалку. Не имея развитой сообразительности, разгадать ребусы будет тяжело. Ребусы про цифру 7 могут заключать в себе зашифрованное слово или внешний вид самой цифры. Скачать интересные ребусы для детей можно здесь.

Интересным для ребят также может быть вопрос: на что похожа цифра «семь»? Давайте подумаем, что могут ответить дети. Она похожа на кочергу, на старуху, на клюшку и т.д. А что бы вы, взрослые, ответили? На что похожа цифра «семь»?

Как правильно писать цифру?

Наглядные и видео пособия

В качестве наглядного пособия, в какой бы класс ни ходил ребенок, может выступить фото с предметами или животными, картинки, где нужно что-то или кого-то посчитать. Можно сделать юмористическое фото цифры 7 с помощью компьютерных программ, совместив число с интересными предметами или явлениями. Можно предложить каждому из детей сделать фото (или продемонстрировать уже имеющиеся) тех предметов, которые очень похожи на число 7. Увидите, с каким удовольствием дети воспримут эту затею. Также фото можно использовать, если учителем, который собирается на урок в начальный класс, готовится презентация. Скачать фото и презентацию также можно на сайте.

Презентации

  1. Пословицы, звгадки, поговорки и стихи о цифре

Презентация просто необходима для полноценного урока и развития ребят. Большинство детей имеет зрительную память, и презентация способствует максимальному усвоению материала. Однако построена она должна быть с умом. Презентация представляет собой не просто набор слайдов: они должны быть методически выстроены. Это означает, что вы должны уметь в любой момент суметь объяснить ваш выбор слайдов. Чтобы презентация у вас наверняка получилась, мы предлагаем вам воспользоваться презентацией, посвященной числу 7, которую можно легко скачать у нас на сайте.

Видео материалы

Раскраски

Математические игры ребусы в картинках для школьников 5-7 классов

Клочкова Наталья Константиновна, учитель математики МБОУ «Бухарайская СОШ» с.Бухарай Заинский район
Описание: Данная работа может быть использована на уроках математики в 5–7 классах. Разгадывание ребусов можно предложить учащимся при проведении устного счета, можно предложить в качестве дидактического материала для домашних заданий. Эта работа может послужить как пособие для проведения внеклассных мероприятий, факультативов. Разгадывание ребусов развивает у ребенка сообразительность и учит его находить выход из сложных ситуаций, что, конечно же, пригодится в жизни. Отгадывая ребусы дети пополняют свой словарный запас, развивают внимание и образное мышление, тренируют зрительную память, учатся правильно писать и запоминать новые слова.
Цель: развитие интеллектуальных способностей, формирование логического мышления.
Задачи:
Обучающие: научить учащихся разгадывать ребусы с математической тематикой.
Развивающие: расширить кругозор учащихся в области математики.
Воспитательные: воспитать сознательное отношение к математике, как к важному предмету.
Введение:
Ребус – это головоломка в которой зашифровано слово. Это слово дается в виде рисунков с использованием букв и цифр, а также определенных фигур или предметов. Ребус – одна из самых интересных головоломок.
На этой картинке зашифровано слово КОМПЬЮТЕР.

Существуют определенные правила для разгадывания ребусов.
1.Запятая в самом начале слова говорит о том, что нужно убрать первую букву в этом слове, а запятая в конце – убрать последнюю букву в слове. Две запятых – убираем две буквы. В слове комар убираем последние две буквы АР, в слове утюг убираем первую букву У и последнюю букву Г.
2.Зачеркнутые цифры говорят о том, что буквы стоящие на этом месте убираются. В слове пять убираем вторую и третью буквы, то есть ЯТ. Если будут зачеркнуты буквы, то они тоже убираются из слова.
3.Не зачеркнутые цифры показывают, что буквы, стоящие на месте 2 и 3 надо поменять местами. В слове утюг буквы Т и Ю меняем местами ЮТ. А теперь читаем слово полностью.
На этой картинке зашифровано слово ПЕРПЕНДИКУЛЯР.


4.Если рисунок в перевернутом виде, то загаданное с помощью картинки слово читается справа налево. Читается не слово репа, а апер. Первая буква А убирается. В слове пень последняя буква Ь убирается. Слово кит читается наоборот. В слове стул убираются первые две буквы СТ. Названия всех предметов, изображенных в ребусе, читаются только в именительном падеже.
5.«Стрелка» или знак «равно» говорят о том, что одну букву надо заменить другой. В нашем случае в слове тик букву Т надо заменить на букву Д. Теперь слово можно прочитать полностью.
На этой картинке зашифровано слово ВОСТОК.


6.Буквы, слова или картинки могут изображаться внутри других букв, над другими буквами, под и за ними. Тогда добавляются предлоги: В, НА, НАД, ПОД, ЗА. У нас в букве О содержится число СТО, поэтому получается В-О-СТО-К.
На этой картинке зашифровано слово КАРТА.


7.Цифры под картинкой указывают на то, что из данного слова нужно взять буквы, стоящие на местах под номерами 7,2,4,3,8 и составить их в том порядке, в котором расположены цифры. В слове ватрушка нужно взять буквы 7-К, 2-А, 4-Р, 3-Т, 8-А. Можно прочитать слово.
Давайте попробуем разгадать несколько ребусов из области математики.
ДОКАЗАТЕЛЬСТВО


ПЯТЬ


ЗАДАЧА


КОНУС


ВЕРШИНА


ДИАМЕТР


ЗНАМЕНАТЕЛЬ


ЛОБАЧЕВСКИЙ


МИНУС


АКСИОМА


ВЕКТОР


ВЫЧИТАНИЕ


ДВА


ДИАГОНАЛЬ


ТРЕУГОЛЬНИК


РОМБ


СТЕПЕНЬ


СЛОЖЕНИЕ


ЧИСЛО


ТОЧКА


СТЕРЕОМЕТРИЯ


Все задания оформлены яркими картинками и интересно проиллюстрированы, поэтому ребусы будут увлекать ребят. А можно попробовать и самим составить. Это будет еще интересней.

Ребусы для школьников с решением и ответами.

Математические задачи бывают самыми разнообразными по сложности, соответственно начинайте с ребенком разгадывать еще с детского садика. Математические ребусы почти всегда нравятся ребятам, поэтому Вам не нужно будет заставлять своего малыша заниматься. Мы постараемся Вам рассказать о том, какую пользу приносят математические ребусы детям, и какие именно головоломки можно предложить разгадывать школьникам определенного возраста.

Для чего нужны математические ребусы для детей?

Математика считается самой сложной наукой, которая способна доставить школьнику очень много проблем во время учения. Но ведь без обыкновенных навыков устного счета и разнообразных математических приемов невозможно просто в будущем нормально жить.

Продолжительные и достаточно сложные математические занятия, особенно с 1-го по 4-й классы, утомляют деток и не дают им возможности нормально усваивать услышанную информацию. Если Вы хотите, чтобы с Вашим ребенком такое не случилось, предложите ему изучать математику в игровой форме, к примеру, в виде математических головоломок или ребусов.

Многие школьники современного времени обожают в собственный досуг развлекаться за счет компьютерных игр либо общаться в социальных сетях с одноклассниками. Однако сегодня есть и те дети, которые не тратят собственного времени на такие игрушки, а отдают предпочтение развитию логики и сообразительности.

В настоящее время сеть Интернет заполнена разнообразными сайтами, где можно без проблем отыскать логические загадки и головоломки. Они предназначены не только для того, чтобы потратить собственное время, но еще чтобы полезно, а самое главное занимательно развлечься. Многие родители уже смогли по достоинству оценить преимущество математических головоломок, шарад, задачек, ребусов, так как их дети благодаря им смогли намного быстрее развиваться.

Благодаря математическим ребусам и задачам ребенок намного быстрее начинает правильнее рассуждать. У него формируется ум и логика.

Преимущество математических ребусов в том, что они не считаются обыкновенными математическими задачками. Они с первого знакомства заинтересовывают деток своим оригинальным изложением, возбуждают у детей желание быстрее найти разгадку на ту или иную головоломку.

Если Вы начнете с Вашим чадом регулярно находить решения к математическим ребусам, Ваш малыш уже очень скоро начнет без проблем решать более сложные задачки, которые до этого он не мог разгадать. Заинтересуйте собственного ребенка обычной математикой, и в этом Вам помогут математические ребусы.

Математические ребусы и головоломки – это загадки, имеющие различную степень сложности, составленные с применением графических элементов. Разгадывать подобные задачки – это очень увлекательно. Помимо этого, ребята более старшего возраста с огромным удовольствием самостоятельно могут составлять математические головоломки для друзей и одноклассников, что позволит им лучше тренировать собственный ум и интеллект, плюс развивать логику.

Если ребусы представлены в виде сложных загадок, детям приходится «поломать» немного голову, дабы отыскать верное решение. Во время данного увлекательного и познавательного занятия у Вашего ребенка будет формироваться нестандартные решения. В будущем такой навык пригодится Вашему чаду для того, чтобы находить возможные выходы из разнообразных ситуаций.

И самое главное, математические задачки и ребусы подарят Вашему ребенку массу положительного настроения. Если же он будет разгадывать такие головоломки с друзьями или с Вами, сможет дополнительно социализироваться и укрепить отношения.

Теперь давайте разберемся с тем, как правильно решать математические ребусы. Красочные картинки с изображением каких-то определенных предметов, цифр, знаков и букв, постоянно вызывают у детей «бешеный» интерес. Но такие картинки, как правило, кажутся им сущим хаосом. И все потому, что дети не знают, как правильно решить ребусы.



Соответственно им кажется, что такие картинки не имеют смысла. Но ведь это можно легко исправить, если внимательно изучить главные правила решения этих головоломок:

  • Названия картинок, которые зашифрованы, представлены только именительного падежа. Когда Вы будете смотреть на картинку с предметом, думайте о том, какое именно название может быть у этого изображения. Соответственно, если Вы увидите на картинке глаз, то может быть на картинке будет зашифровано «око». Никогда не останавливайтесь на одном ответе.
  • Если на картинке изображена запятая, значит у данного слова необходимо убрать какую-то определенную букву или одновременно несколько. Все будет зависеть от того, где находится запятая: перед изображением или же после него.
  • Зачастую в головоломках подобного рода встречаются буквы, которые подчеркнуты. Это решить очень легко. Вы отгадываете слово на картинке, а после этого убираете те буквы, которые подчеркнуты. Если на картинке изображены подчеркнутые числа, тогда Вам нужно убрать буквы, которые соответствуют порядковому номеру. Если стоят числа и буквы около не подчеркнутого изображения, тогда Вам необходимо оставить лишь данные буквы.
  • Если на картинке стоит значение Б = Р, тогда Вам нужно буквы «Б» заменить на букву «Р». Если Вы увидите вот такое равенство 2 = О, тогда в слове замените вторую букву на «О». Также на картинке может присутствовать стрелочка, к примеру, от первой буквы к третьей, тогда их просто нужно заменить друг дружкой.
  • Есть такие картинки, которые изображены в перевернутом состоянии. Тогда прочитайте слово с конца.
  • Встречаются математические ребусы, в которых есть дробь . Они легко расшифровываются: нужно вставить предлог «на». Если в знаменателе есть «2» — это означает «пол». В некоторых случаях Вы можете заметить, что во внутренней части буквы находится слог либо буква. Трактуется это так: например, если внутри буквы «О» находится «Да», тогда эта картинка означает «Вода».

Есть и другие правила, которые помогут Вам научиться разгадывать сложные головоломки либо числовые ребусы. Но с ними ребенок должен познакомиться после того, как научится разгадывать простые задачи.



Почаще проводите свое свободное время с детьми. Разгадывайте с ними ребусы, научите их находить решения к этим ребусам, так как это оказывает положительное воздействие на мозговую деятельность развивающего организма.

Математические ребусы с ответами для детей 1 класса: фото, решение, описание

Если Ваш ребенок начнет решать логические задачи с 1-го класса, у него быстрее будет развиваться сообразительность, мышление, умение делать правильные выводы и выполнять анализ. Именно подобный подход к увеличению математических возможностей имеет самую большую положительную сторону для формирования правильного мышления у деток.

Все мы знаем, что программа, составленная для школы, предполагает, как правило, лишь обучению деток решать определенные виды задач. Ученые утверждают, что важнее, чтобы первоклассник с самых первых школьных шагов смог обучаться отлично мыслить и правильно рассуждать. Они также подтвердили, что нестандартные задачки, которые необходимо решать, включив смекалку и немного мышления, очень часто ставят в затруднительную ситуацию и тех ребят, которые в школе учатся только на отлично.

Мы предлагаем Вам большое число математических ребусов для школьников. Решайте их вместе с детьми, находите совместно правильные решения, отдыхайте так, чтобы ребенку было интересно.

Цифры, которые одинаковые, обозначены на картинке одинаковыми элементами. Различные числа – разными.



Первый ребус (первоисточник смотрите )

Подумайте вместе, какое именно число фокусник решил превратить в змею?

Решение:

В первом примере змея и черепаха могут скрывать такие пары чисел: 0 – 4 либо 1 – 3. Теперь сложите эти числа. В первом случае у Вас получится 4, во втором – тоже 4.

Во втором примере ребуса подходит лишь второе сочетание чисел, так как если от 3 отнять 2 получится 1.

Ответ: за змейкой спрятана единица.



Решение:

В слове «кость» вместо «О» поставьте «И», а последнюю букву вообще уберите. Во втором слове вместо «И» поставьте «А».

Соедините эти два слова.

Ответ:

Кисточка.



Решение:

На картинке изображена лейка. Перед этим словом поставьте «К», а две последние «К» и «А» уберите.

Ответ:

Четвертый ребус:



Решение:

На картинке изображена тучка. Впереди этого слова поставьте «Р», а первую букву «Т» уберите.

Ответ:

Математические ребусы с ответами для детей 2 класса: фото, решение, описание

Во 2-ом классе программа сложнее, чем в 1-ом. Процесс обучения становится более трудоемким, соответственно Вам нужно своему чаду помочь.

Конечно же, учеба нужна, но нельзя сильно перегружать школьника. Программы, которая дается в школе, и домашнего задания, будет достаточно. Есть такие школьники, которые в школе учатся замечательно, а когда они приходят домой, начинают отказываться делать уроки.

Но Вы знаете, что детям обязательно необходимо повторять пройденный материал в школе, изучать что-то новое, улавливать новые для них слова, развивать собственное мышление и так далее. Возможно, Вы думаете, что ребенок во 2-ом классе уже стал взрослее, начинаете подавать ему много новой информации в виде дополнительных уроков, а потом удивляетесь, почему Ваши старания не дают положительных результатов.

Дело в том, что Ваш малыш устает в школе, он хочет немного поиграть и хорошенько отдохнуть. Поможет ему в этом игра, к примеру, математические ребусы. Существует большое количество таких головоломок. Но есть родители, которые ошибаются, подбирая развлекательную головоломку не по возрасту.

Не делайте и Вы этого. Внимательно изучите те варианты математических ребусов, которые предлагаем Вам мы. Они предназначены именно для школьников 2-го класса.

Решение:

На картинке изображен ключ. В этом слове уберите последние две буквы. А в конце самого слова поставьте «ЫК».



Ответ:



Решение:

На картинке изображен зонт. Уберите в слове две последние буквы. Перед словом поставьте «У» и в конце поставьте «Р».

Ответ:



Решение:

На картинке изображен лист. Вместо буквы «Л» поставьте букву «А».

Ответ:

Математические ребусы с ответами для детей 3 класса: фото, решение, описание

Ребусы, которые предназначаются для школьников 3-го класса, могут разделятся на некоторые виды. Все зависит от дисциплины в школе, к которой эти головоломки относятся. Также они могут разделяться по уровню сложности.

Учителя уже неоднократно доказывали то, что математические ребусы помогают ученику эффективнее усваивать процесс обучения. Они утверждают, что благодаря таким ребусам ребенок начинает хорошо мыслить и развивает в себе творческую способность. А еще математические ребусы помогают улучшить настроение для того, чтобы изучать новые предметы.

Очень трудно выделить те ребусы, которые подходят для ученика 3-го класса. Мы Вам хотим предложить некоторые варианты, которые Вы сможете разгадать со своим ребенком.



Решение:

На картинке изображен ромб. Уберите последние две буквы «М» и «Б». спереди слова поставьте «К», а в конце «Т».

Ответ:



Решение:

На картинке изображен дом. Уберите первую букву «Д». Впереди слова поставьте букву «Л».

Ответ:

Решение:



На картинке изображен перевернутый дом. Это означает, что слово нужно прочитать с конца. Добавьте еще в конце слова букву «А».

Ответ:

Четвертый ребус:



Четвертый ребус

Решение:

В этом варианте математического ребуса изображены буквы и цифры. Вам нужно поступить следующим образом: вместо цифры 100 напишите буквами, а потом соедините все буквы.

Ответ:

Математические ребусы с ответами для детей 4 класса: фото, решение, описание

Школьники в 4-ом классе уже начинают знакомиться с пространственным представлением. Дети изучают поверхностно геометрические фигуры и их простые свойства, начинают постепенно выполнять легкие чертежи, применяя при этом примитивные приборы для измерения. Именно в этот период времени у детей начинает формироваться основа для будущего обучения.

Школьники переходят к более сложной науке, которая очень скоро поделится на пару курсов: первый курс – это алгебра, второй – это геометрия. Зачастую, чтобы ученики немного отдохнули от тяжелого урока, учителя применяют дополнительные задания, например, математические головоломки и ребусы. Предлагаем Вам некоторые из них, которые, возможно, Вы разгадаете со своим ребенком.



Решение:

На картинке Вы видите слово и изображение предмета «нож». Вместо цифры 100 напишите слово «сто». Спереди слова «нож» уберите первую букву. Соедините все буквы.

Ответ:



Решение:

На картинке изображен гриб. Уберите спереди слова первую букву. Вместо буквы «И» поставьте букву «Ы». В конце слова поставьте «КА».

Ответ:



Решение:

На картинке изображен лист и гусь. В первом слове поменяйте местами буквы, как указано на картинке. Во втором слове уберите три первые буквы. Потом попробуйте прочитать то, что у Вас получилось.

Ответ:

Математические ребусы с ответами для детей 5 класса: фото, решение, описание

Для учеников, которые уже перешли в 5-ый класс и выше, существуют свои усложненные математические ребусы. Над ними дети должны серьезно поработать, чтобы отыскать правильный ответ. Если такого не произойдет, задачки просто не заинтересуют ребят и тогда они не принесут пользы.

Для пятиклассников предлагаем Вам вот такие ребусы:



Решение:

На картинке изображена оса и дробь. Так как у нас тут присутствует дробь, значит решение такое: под буквой «Н» находится оса. От слова «оса» отнимите последнюю букву. А далее сложите под + н + ос (последняя буква уже отсутствует).

Ответ:



Решение:

Сочетание «ЗА» находится в букве «А». Решение такое: в + а + за.

Ответ:

Математические ребусы с ответами для детей 6 класса: фото, решение, описание

В 6-ом классе детки уже становятся совсем взрослыми. Это означает, что и математические головоломки должны быть посложнее.



Решение:

На картинке изображен перевернутый гриб и оса. Поступите следующим образом: слово «гриб» прочитайте наоборот. В этом же слове вместо буквы «Г» поставьте букву «К». От слова «оса» отнимите первые две буквы. Сложите оставшиеся буквы.

Ответ:



Решение:

Здесь чтобы отыскать решение, ребенку придется немного подумать. Не говорите ему сразу ответ. Пусть Ваш школьник подумает над ответом сам, а Вы послушайте, какое именно решение он предложит Вам.

Ответ:

Математические ребусы с ответами для детей 7 класса: фото, решение, описание

Как правило, в 7-ом классе у детей начинается алгебра и геометрия. Они уже знакомы со многими геометрическими фигурами, у них лучше развито мышление, чем у школьников начальных классов. Это означает, что для таких детей нужны математические ребусы с высокой степенью сложности.



На картинке изображено сочетание букв и цифр. Вместо цифры 100 напишите слово «сто». Теперь соедините все буквы. Правда придется немного подумать.



На картинке изображена цифра 7, буква «К» и рот. «7» напишите словом «семь» и отнимите от него две последние буквы. Рот изображен перевернутым. Значит Вам нужно его прочитать наоборот с конца.



На картинке изображено перо с метром. Запятая говорит о том, что Вам нужно убрать последнюю букву от слова «перо». Все очень просто. Соедините те буквы, которые остались от слова «перо» с буквой «И» и словом «метр».

Видео: Ребус с ответами для школьников

%d0%b8%d0%bd%d1%84%d0%be%d1%80%d0%bc%d0%b0%d1%82%d0%b8%d0%ba%d0%b0+6+%d0%ba%d0%bb%d0%b0%d1%81%d1%81+%d1%80%d0%b5%d0%b1%d1%83%d1%81

4.9 475 %d0%b8%d0%bd%d1%84%d0%be%d1%80%d0%bc%d0%b0%d1%82%d0%b8%d0%ba%d0%b0+6+%d0%ba%d0%bb%d0%b0%d1%81%d1%81+%d1%80%d0%b5%d0%b1%d1%83%d1%81

Hex,Octal,HTML Decoder – DDecode

… D0%B9+%D0%BF%D0%BE+%D0%B2%D1%81%D0%B5%D0%BC+%D1% 81%D1%82%D1%80%D0%B0%D0%BD%D0%B0%D0%BC+%D0%B8+%D0 …

  ddecode.com

Как выкачать верстку из письма? — Хабр Q&A

… браузер показать свойства письма выходит вот такая чепуха=D0=94=D0= BE=D0=B1=D1=80=D1=8B=D0=B9 =D0=B4=D0=B5=D0=BD=D1=8C!

  qna.habr.com

Santa Cruz, CA – Testimonials – Harbor Veterinary Hospital

… <a href=https://muzaza.ru/mp3/%D0%B8%D0%B7+%D1%84%D0%B8%D0% BB%D1%8C%D0% … .ru/?music=%D1%85%D0%B8%D1%82%D1%8B%2090 %20%D1%85>музыка-90-х</a> … %BA%D0%BE%D0%BC%D0%B8%D1%81 %D1%81%D0%B0%D1%80+remix>комиссар ремиксы</a> … Posted : 2/6/2021

  www.harborvet.com

Профессиональная Этика Учебное Пособие Для Бакалавров …

http://noskovazoia.smallheart.ru/?gmdl&keyword=%D0%9F%D1%80%D0%BE% D1%84%D0%B5%D1%81%D1%81%D0%B8%D0%BE%D0%BD%D0%B0% …

  www.google.com

Скачать бесплатно формирование математических …

http://liudmilakoo.softbeststore.ru/?gmdl&keyword=%D0%A1%D0%BA%D0%B0 %D1%87%D0%B0%D1%82%D1%8C+%D0%B1%D0%B5%D1%81%D0%BF …

  www.google.com

http://video.yandex.ua/#search?text=%D0%9F%D1%80%D0%B5 …

http://video.yandex.ua/#search?text=%D0%9F%D1%80%D0%B5%D1%81%D1 %81-%D0%BA%D0%BE%D0%BD%D1%84%D0%B5%D1%80%D0%B5%D0 …

  www.facebook.com

Просмотр исходного текста страницы Программы для …

_%D0%A2%D0%B5%D0%BA%D1%81%D1%82%D1%8B_%D0%BF%D1%80 %D0%BE%D0%B3%D1%80%D0%B0%D0%BC%D0%BC#.D0.9E.D0.BF.

  pascalabc.net

Результаты поиска “%D0%BA%D0%B0%D1%84%D0%B5%D0 …

Результаты поиска » %D0%BA%D0%B0%D1%84%D0%B5%D0%B4%D1%80 %D0%B0%20%D0%B3%D0%BE%D1%81%D1%83%D0%B4%D0%B0%D1%  …

  igsu.ranepa.ru

https://www.dw.com/ru/%D0%BE%D0%BE%D0%BD-%D0%B7%D0 …

… %BB%D0%B8-%D1%82%D0%B5%D1%80%D1%80%D0%BE%D1%80%D0 % … 0.5 https://www.dw.com/ru/%D1%81%D0%B0%D0%BC%D1%8B%D0%B5 -% … %82%D1%81%D1%82%D0%B0%D0%B2%D0%BA%D1%83/a- 46532895 … 84-%D1%81%D1%82%D1%80%D0%B0%D0%BD%D0%B8%D1 %86-%D0 …

  www.dw.com

Результаты поиска | %D0%A1%D0%BF%D0%B8%D1%81%D0 …

… %D0%BC%D0%B5%D1%80%D0%BE%D0%BF%D1%80%D0%B8%D1%8F %D1%82%D0%B8%D0%B9 %D0%BD%D0%B0 …

  www.travelhotels.co.il

Поиск Yandex ничего не нашел

Самые сложные ребусы для взрослых. Увлекательные задачки, головоломки и ребусы для взрослых и детей (11 фото)

Ребус – логическая игра, в которой надо разгадать ответ по картинке. На последней изображаются предметы, животные и растения, буквы и цифры. Имеет значение их взаимное расположение. Даже для непосед ребусы могут стать увлекательным занятием, если подать его в игровой форме. Например, можно предложить научить ребенка, как разгадывать шпионские шифровки.

И от самых простых картинок-головоломок для дошкольного возраста добраться до сравнительно сложных. Уверяем: если ваше чадо увлечется и научиться включать логическое мышление, со временем уже вы будете учиться у него, как решать загадки в картинках.

Ребусы придуманы на огромное множество тем. Главное, чтобы каждое слово, буква и предмет, служащие ответом к картинке, были уже знакомы малышу.

Как решать ребусы для детей с буквами в картинках?

Если вы заинтересовались ребусами, то, скорее всего, знаете преимущества этих логических задачек. Они развивают память, сообразительность, скорость мышления, умение ориентироваться в ситуации и применять уже полученные знания.

Чтобы научить ребенка 6-7 лет, как правильно решать задания, для начала начала объясните ему правила. Не нужно настаивать, чтобы он запомнил все сразу. Скорее всего, вы и сами их все не знаете. Лучше за день объяснять одно-два и подкреплять тематическими заданиями. Последние можно распечатывать (удобнее для занятий на улице) или показывать с монитора. На следующих занятиях тоже лучше не предлагать слишком много материала. Важно объяснить чаду, что сначала ему нужно правильно идентифицировать и назвать предмет, изображенный на картинке. И только потом в отношении этого слова применять правила.

Итак, почитаем основные правила! В частности, определимся, что означает запятая, перечеркивание, перевернутый предмет и другие тонкости в картинках.

  • Что значит запятая в начале или конце ребуса?
    Запятая внизу или вверху перед картинкой значит, что от названия изображенного предмета надо отбросить одну букву в начале. Соответственно, видим две запятые – отбрасываем две первые буквы. Эти значки встречаются очень часто.
  • Что значит перевернутая запятая в начале или конце?
    Правила для перевернутых запятых, аналогичны правилам для обычных запятых (смотри предыдущий пункт).
  • Что означают зачеркнутые и дописанные буквы?
    Зачеркнутая буква на картинке значит, что из названия нарисованного предмета ее нужно исключить (и добавить другую, если она указана). Дописанная слева или справа от картинки – надо добавить ее к слову в начале и в конце.
  • Что означают цифры в ребусах?
    Цифры могут иметь два значения. Они стоят над словом? Чтобы отгадать ответ, надо переставить буквы с места на место в обозначенном порядке. Название цифры может быть частью слова (часто используют «сто», «пять»). Зачеркнутая цифра значит, что из слова нужно исключить букву с таким порядковым номером. Следует помнить, что некоторые цифры, так же как и предметы, могут иметь несколько названий (единица – «кол», «раз», «один»).
  • Что означает плюс, знак «равно»?
    Если между словами (символами) стоит знак «плюс», значит, их надо добавить друг к дружке. Иногда «+» значит предлог «к», нужное выбирается по смыслу. Знак «равно» (например, А=К) говорит о том, что все буквы «А» в слове следует заменить на буквы «К».
  • Вертикальная или горизонтальная черта в заданиях?
    Горизонтальная черта значит одновременно «под», «по», «над» и «на» в зависимости от контекста. Используется с буквами или картинками, когда одна часть нарисована ниже черты, другая – выше. Иногда обозначает дробь (половину чего-либо, то есть «пол-»).
  • Расположение букв на картинке и предлоги
    Важно посмотреть на взаиморасположение букв. Если они размещены одна в другой, значит, к их названиям добавляется предлог «в». Одна буква нарисована за другой – имеется в виду предлог «за» или «перед».
  • Предмет на картинке нарисован вверх ногами ? Чтобы получить ответ, надо прочитать слово наоборот. Короткие слова дети 6-7 лет вполне могут перевернуть в уме. Правда, количество таких заданий довольно ограничено.

Чаще всего в ребусах используется одновременно несколько правил. Считается, что в 6-7 лет дети уже знакомы с буквами, четко знают их названия. Если младший школьник еще не сталкивался с запятыми, научить его новому значку не составит особой сложности.

Примеры ребусов в картинках для детей 6-7 лет с ответами

Дети 6-7 лет и меньше гораздо лучше воспринимают материал в привязке к какому-либо запоминающемуся событию. Ребусы про животных будут разгадываться с восторгом, если предложить их ребенку на следующий день после посещения зоопарка. Девочке-первоклашке, которая горит желанием поступать в музыкальную школу, будут интересны музыкальные ребусы. А ребенку, впечатленному планетарием мальчику, понравятся картинки про космос.

Про животных и о птицах

Давая детям задание про птиц или животных, убедитесь, что он уже сталкивался с такими названиями животных, а также понимает все, что изображено на картинке.

Ребусы о семье, о маме

Кто для ребенка милее всех, если не мамочка! А кого он с радостью встречает каждый раз, кроме мамы и папы? Деткам очень понравится узнавать и угадывать в зашифрованных картинках бабушку, дедушку, сестричку и других родных. Распечатайте или нарисуйте картинки поярче и начинайте развлекаться, заодно обучая ребенка!

О спорте, о здоровье

Ребусы о труде, здоровье, спорте, профессиях и многие другие можно использовать в качестве тематических игровых пособий. В выпускной группе детсада, первых классах школы или дома намечается занятия либо беседа на одну из тем? Загадка в виде картинки позволит усвоить материал лучше, чем обычный безликий рассказ. Малышей заинтересует нестандартная подача материала.

Ребусы по сказкам

Сказки со знакомыми героями, современные или классические мультфильмы – неисчерпаемый кладезь вдохновения. Если ребенок не очень интересуется логическими загадками, можно попробовать увлечь его отгадыванием любимых персонажей. Загадок на эту тему гораздо больше, чем приведено в качестве примера. Зная интересы и любимые сказки вашего ребенка, вы можете создать ребусы в виде аппликаций самостоятельно.

Для того чтобы найти ответ на представленные задачки, вам не потребуются трехэтажные формулы, инженерный калькулятор и длинные расчеты. Логика, фантазия и способность смотреть на вещи под другим углом позволят вам, подобно Архимеду, воскликнуть «Эврика» 10 раз за время прочтения этого поста.


Смотреть все фото в галерее

Переправа, переправа


Это очень древняя загадка, известная многим народам. Задачка про волка, козу и капусту имеет множество вариаций, в одной из них рассказывается о двух супружеских ревнивых парах, а в мультсериале «Симпсоны» в серии из 20 сезона Гомеру надо было перевезти через реку малышку Мэгги, пса и банку крысиного яда.
Условия варианта с животными и капустой: крестьянину надо перевезти в лодке через реку волка, козу и капусту. Лодка вмещает только двух пассажиров (капуста тоже считается за пассажира). Понятно, что волка с козой, а козу с капустой оставлять одних нельзя. Как крестьянину перевезти нажитое непосильным трудом на другой берег?

Показать ответ

Первой надо перевезти козу, оставив волка грустить в компании капустки. Вернуться за хищником или за овощем, перевезти на другой берег и забрать с собой козу. Парнокопытное оставить в исходной точке, взять то, что осталось, переплыть на другой берег и вернуться за козой. Как волк, коза и капуста будут мирно существовать на другом берегу — уже совсем другая история.

Парковка


Говорят, что такую задачку задают китайским первоклашкам, точнее поступающим в первый класс детям, и дают на ее решение 20 секунд. Назовите номер парковочного места, которое закрывает красный .

Показать ответ

Если иметь перед собой эту задачку на листке бумаги, а не на экране компьютера, то решение может прийти быстрее, потому что головоломку надо просто перевернуть. Нажмите на клавиатуре ctrl+alt+вниз и увидите, какую цифру закрывает (чтобы вернуть монитор в исходное состояние, нажмите ctrl+alt+вверх). Уложились в 20 секунд?

Еврейский вопрос


Этот вопрос когда-то давно был в игре «Что? Где? Когда?». Звучал он примерно так: один американский журналист в статье про отношения США и Израиля утверждал, что США буквально располагаются в самом центре Иерусалима и доказывал это при помощи заголовка. Каким же образом, по мнению американского журналиста, США находятся в центре Иерусалима?

Показать ответ

Для того чтобы понять логику журналиста, вам надо написать название священного города на английском. JerUSAlem озаглавил статью американец. 3:0 в пользу знатоков.

Равенство


На эту задачку дается тридцать секунд. Надо доказать, почему это равенство верно. Время пошло.

Показать ответ

Здесь на помощь опять придет ctrl+alt+вниз. Получается, что eight=8, то есть восемь=8.

Ламповая задачка


Представьте себе две комнаты, разделенные стеной с непрозрачной дверью. Вы в комнате с тремя выключателями, а в соседней с потолка свешиваются три лампочки, каждой из которых соответствует один из выключателей. Вам надо выяснить, какой лампочке какой рубильник соответствует, при этом в комнату с лампочками можно зайти только один раз. Свет из-под двери не пробивается, в щелочку смотреть нельзя, окон и дыр в стене нет.

Показать ответ

Надо сделать так: нажимаете вкл. на двух выключателях. Ждете.
Выключаете один из них. Идете в комнату с лампочками. Одна горит, две другие нет, но одна из неработающих должна быть еще горячей. Осторожно трогаете лампочки и разгадываете загадку.

Уж замуж невтерпеж


Загадка-сказка про замужество. У царя три дочери. В королевство приезжает красавец принц и просит срочно выдать за него одну из девушек, неважно какую. Царь, вместо того, чтобы спросить у дочерей, кто из них хочет выйти замуж, устраивает что-то вроде конкурса. У кого из дочерей быстрее закипит кастрюля с водой, та и будет с принцем жить-поживать и добра наживать. Условия равны: кастрюли одинакового объема, воды налито поровну, температура нагревания тоже у всех одна и та же. И еще важное условие: младшая дочь замуж пока не собирается и выигрывать в турнире не очень хочет, средняя хочет поскорее свинтить от царственного папаши, а старшая просто мечтает об этом самом принце и души в нем не чает. У кого из дочерей в итоге кастрюля закипела быстрее?

Показать ответ

Вожделеющая принца старшая дочь и мечтающая покинуть родительский дом средняя все время открывали крышки своих кастрюль, тем самым понижая температуру воды. А вот младшая преспокойно сидела на стуле, красила ногти на ногах, насвистывала веселую мелодию, не трогала кастрюльку и в итоге вышла замуж.

Классику надо знать


Раз уже вспомнили трех сестер, то вот вам такой вопрос. Что за загадочная «реникса» украшала тетрадь ученика одной семинарии, что это значит?

Показать ответ

В чеховских «Трех сестрах» муж Маши рассказывает: «В какой-то семинарии учитель написал на сочинении «чепуха», а ученик прочел «реникса» — думал, по-латыни написано». Так что «реникса» — это чепуха, но не по-латыни, а русскими прописными буквами.

Загадочная смерть


В жизнеописании Гомера неизвестного автора, которого принято называть Псевдо-Плутархом, говорится о том, как пожилой поэт спросил у богов о месте своего рождения. Боги сообщили поэту, что родился он на острове Иос и посоветовали остерегаться загадочных слов мальчишек. Так вот сказитель приехал на Иос. Однажды он сидел на камнях на берегу моря и увидел (у этого фейкового Плутарха он не слепой) юных рыбаков, проходящих мимо. Гомер поинтересовался уловом. Ничего не поймавшие ребята ответили ему: «Все, что мы поймали, мы оставили позади, все, что мы не поймали, мы носим с нами». По версии Псевдо-Плутарха легендарный сказитель умер от уныния, так и не найдя расшифровки загадочных слов рыбаков. Что же имели ввиду маленькие жители острова Иос?

Показать ответ

Рыболовы имели ввиду вшей. В отсутствии клева они ловили на себе вшей и выбрасывали их в море. Те, которых поймать не удалось, остались жить в их хитонах.

Кто тяжелее?


Эти три кубика сделаны из одинакового материала, отверстия в них одинакового диаметра, являются сквозными и перпендикулярными граням. Ваша задача определить самый тяжелый и самый легкий кубик.

Показать ответ

Самый тяжелый — второй, самый легкий — первый. Чем больше отверстий пересекается внутри кубика, тем он тяжелее.

Черно-белый ребус


В финале хитрый ребус. Какое слово здесь зашифровано?

Показать ответ

Это «колесо» образуют сорок букв «А». Ответ: Сорока

В этой статье мы рассмотрим самые интересные головоломки, предназначенные для детей и при этом подвластные не каждому взрослому. Они успели поставить в ступор не одного интернет-пользователя и обрели огромную популярность в Интернете, как и шуточные тесты с ответами , – а как быстро с ними справитесь вы? Правильные ответы ждут вас в конце статьи!

Куда едет автобус?

Если говорить про самые популярные детские задачи в Интернете, то это одна из них. Перед вами изображение автобуса. В какую же сторону он направляется?

Сколько здесь точек?

Еще задачи на внимательность для самых зорких пользователей: сколько черных точек на пересечениях линий вы видите?

Какой кружок больше?

А теперь будем разгадывать интересные графические головоломки. Сможете ответить, какой из желтых кругов, изображенных на рисунке, больше в размерах?

Двигаем спички

Следующие детские головоломки тоже часто дают решать первоклашкам: в них требуется двигать спички определенным образом, чтобы получить заданную фигуру.

Найдите панду!

Интернет взорвали и следующие графические головоломки художников, которые в сложные картинки поместили изображение панды и предлагали другим пользователям найти ее. Они спрятали панду в толпу штурмовиков из «Звездных войн», в сборище металлистов, и даже пытались скрыть ее среди несметного количества массажных столов. Проверьте свою внимательность!

Японский IQ-тест

А вот какой тест на определение IQ придумали японцы. На берегу стоит мужчина с двумя сыновьями, мать с двумя дочерьми и полицейский с преступником. Перед ними плот, на котором им нужно перебраться на другой берег. Попробуйте подумать, как их можно туда перевезти, учитывая такие интересные условия:

  • На плоту могут поместиться одновременно только двое, а совсем без людей он плыть не может.
  • Дети могут передвигаться на плоту только со взрослыми. Но сыновья не могут одни оставаться с матерью девочек, а дочки – с отцом мальчиков.
  • А преступник не может оставаться наедине с остальными без присмотра полицейского.

Нашли ответ? Если нет, то прохождение этого любопытного теста смотрите в видео:

Правильные ответы

У этой головоломки могут быть два правильных ответа. Первый – автобус едет влево, поскольку на другой стороне, невидимой для зрителя, расположены двери, через которые пассажиры попадают внутрь. Этот ответ справедлив для наших дорог с правосторонним движением. Но для стран, где дорожное движение левостороннее, правильным ответом будет – вправо.

На картинке изображены парковочные места, и автомобиль занимает одно из них. Если вы перевернете рисунок, то поймете, что изначально видели числа вверх ногами. Поэтому число под машиной – 87. Сколько бы вы ни пытались вычислить здесь какой-нибудь хитроумный полином, такие интересные головоломки рассчитаны не на алгебраическую логику, а скорее на смекалку.

Недостающее значение = 2. Чтобы разгадывать подобные детские головоломки, нужно ставить себя на место детишек. Разве умеют малыши решать сложные уравнения, считать арифметические прогрессии? Зато они замечают, что значения в столбиках зависят от количества кружочков в каждом наборе цифр. Возьмем, например, ряд 6855: в цифре 6 есть один кружок, а в цифре 8 целых два, поэтому на выходе получаем значение 1+2 =3, то есть 6855=3. А в ряду 2581 двумя кружками обладает только цифра 8, поэтому решение – 2.

Всего на рисунке изображено 12 точек. Но наш мозг устроен таким образом, что не позволяет увидеть их все одновременно, поэтому за один раз мы можем заметить лишь три-четыре черные точки.

Кружки абсолютно одинаковые! Такие простые головоломки построены на зрительной иллюзии. Синие кружки в левой части рисунка большие и находятся на некотором расстоянии от желтого. Круги же в правой части маленькие и стоят плотно к желтому кружку, поэтому нам и кажется, что он больше, чем первый.

А вот как решаются интересные детские головоломки со спичками:


Разоблачаем панду:

Ребусы для взрослых являются интересными загадками в картинках. Ребусы представляют собой такую игру, в которой слова зашифрованы при помощи рисунков, а иногда и целые фразы или высказывания. Наряду с рисунками на картинке могут значиться буквы, цифры знаки, символы. Ребусы, конечно же, также делятся по уровню сложности, и для начала надо решать только самые простые головоломки, чтобы понять их принцип, а уж потом можно браться за расшифровку целых предложений. В любом случае, не имеет значения, в каком возрасте вы решили научиться разгадыванию ребусов, это очень полезное занятие, чтобы с пользой провести свой досуг.

Давайте сначала взглянем на те ребусы , которые вы можете нарисовать даже самостоятельно на бумаге и предложить разгадать их членам вашей семьи. Это ребусы с пространственными предлогами, то есть при расшифровке вы можете добавить к слову «в», «над», «на», «под», «за», «по», «от», «перед», «у», «с». Например, вы видите, что на картинке на переднем плане изображены две буквы «КА», они нарисованы объемными и яркими, а на втором плане изображена буква «З», вы должны сразу понять, что «З» стоит за «КА», и исходя из этого подставить правильный пространственный предлог и расшифровать слово, то есть в данном случае было зашифровано слово «ЗА-КА-З».

Следующий ребус отлично подойдет для тех, кто уже закончил старшую школу, потому что в нем будет упоминание про такой математический символ, как «число Пи», который выглядит как прописная «П». На картинке изображена большая буква «О», внутри овала которой стоит буква «Пи». Наверняка, если ребенок подойдет к своему отцу и спросит, какое же слово тут зашифровано, то он сможет догадаться что это «Пи-В-О».

Более сложные ребусы с ответами содержат не только буквы, но и изображения и знаки, например, ребус состоит из большой буквы «О», в центре которой изображены розы, а после буквы «О» стоят две запятые и написаны еще три буквы «БЕЙ». Безусловно, здесь используется пространственный предлог, ведь в букве «О» находятся розы. После первой части ребуса, который можно расшифровать, как «В-О-РОЗЫ», стоят две запятые, которые указывают на то, что из данного слова надо выбросить последние две буквы, то есть у нас получается слово «В-О-РО» и еще добавляем «БЕЙ», и становится понятно, что было зашифровано слово «Воробей», чем вам не ребусы с ответами по русскому языку .

Конечно же, самые интересные загадки-головоломки для взрослых будет содержать в себе только буквы и в зависимости от их расположения друг относительно друга и будет определено слово. Так может быть зашифровано слово «Подошва»: на рисунке в верхней части написана буква «О», под ней проведена черта и ниже написана буква «А» внутри которой написана «Ш».

Также можно составить и другие , например, слова «Почта», это когда по большой букве «Ч» нарисованы «та». А слово «голова» может быть зашифровано тоже очень просто, достаточно в большой букве «А» написать «голо».

Развивать память и сообразительность вам помогут разные виды головоломок для взрослых – задачи на логику, сложные, смешные или заковыристые вопросы, разнообразные интеллектуальные математические игры.­

Игры и головоломки для взрослых на логику

Для решения разнообразных логических головоломок не требуется высокий уровень образования, начать их решать – полезное занятие абсолютно для всех.­ Разнообразные загадки с подвохом для взрослых тренируют нестандартное мышление, что поможет в повседневной жизни быстрее находить рациональные решения в непростых житейских ситуациях.

Загадки для взрослых­

Такой тип заданий позволит сразу же проверить правильность найденного вами решения. Чем хороши эти короткие загадки? Можно тематически, по ответам, сделать подборку для определенного праздника или застолья, чтобы развлечь гостей. В зависимости от состава приглашенных, хорошо подойдут загадки с подвохом, задания, которые рассмешат ваших друзей, или математические задания.

С подвохом

В заданиях с подвохом зачастую сам вопрос на первый взгляд выглядит нелогичным, например: на каком языке говорят молча? Когда же оглашается ответ, то первая реакция человека проявляется как несогласие с ним. На первый взгляд, вопрос и подобранная отгадка соединены непривычно и с двойным подтекстом. Но немного подумав, не можешь не согласиться, что такое дерзкое решение правильное и очень логичное (ответ: на языке жестов).

Смешные

Играть в разгадывание смешных загадок – одно удовольствие. Пока ваши гости будут высказывать варианты ответов на заковыристые вопросы, всю компанию гарантированно будут сотрясать взрывы хохота.­

Математические

В таких загадках надо угадать заданную цифру, либо высчитать результат, полагаясь не так на арифметику, как на сообразительность. Ответ, который выглядит очевидным и как будто лежит на поверхности, часто является неверным.

Интеллектуальные игры

Задачи на логику для взрослых – это многоходовые комбинации для тренировки мышления. Чтобы правильно их решить, надо продумывать действия на несколько шагов вперед. Такие задания относительно трудные, часто они бывают в виде оригинальных картинок, где нужно переставить или дорисовать некоторые элементы.

▶▷▶ ребусы по алгебре за 7 класс картинки с ответами

▶▷▶ ребусы по алгебре за 7 класс картинки с ответами
ИнтерфейсРусский/Английский
Тип лицензияFree
Кол-во просмотров257
Кол-во загрузок132 раз
Обновление:27-12-2018

ребусы по алгебре за 7 класс картинки с ответами – Yahoo Search Results Yahoo Web Search Sign in Mail Go to Mail” data-nosubject=”[No Subject]” data-timestamp=’short’ Help Account Info Yahoo Home Settings Home News Mail Finance Tumblr Weather Sports Messenger Settings Want more to discover? Make Yahoo Your Home Page See breaking news more every time you open your browser Add it now No Thanks Yahoo Search query Web Images Video News Local Answers Shopping Recipes Sports Finance Dictionary More Anytime Past day Past week Past month Anytime Get beautiful photos on every new browser window Download головоломки по математике 7 класс с ответами в картинках кампусятарф/blog/27250html Cached Ребусы по математике 14 ч назад математические ребусы головоломки задачи в картинках с ответами 6 класс Ребусы по математике с ответами · Подробнее Комментарии ( 132) Ребусы «Алгебра» – IgraZaru Игры, ребусы, загадки wwwigrazaru/page-1-1-25html Cached Ребусы по ПДД (2 коллекция) Архив с ребусами и ответами в формате zip можно скачать (1,78 Mb) ребусы с ответами для 3 класса картинки bravelut8blogspotcom/2013/05/3_14html Cached социальные автобусы подольск бульвар дмитрия донского музыкальные кроссворды и ребусы для детей загадки, ребусы ,задания по английскому языку для 2 класса ту-туru автобусы ребусы в картинках с ответами онлайн мерс Ребусы по математике – vremyazabavru vremyazabavru/zanimatelno/rebusi/rebusi-slova/82-rebusi Cached Ребусы по математике Я зашифровал слова по математике в ребусы Попробуйте их решить не заглядывая в ответы Ребусы для алгебры 7 класс Математические ребусы (5-6 классы kinderbooksru/klass/rebusy-dlya-algebry- 7 -klass Cached Нужны загадки, ребусы ,кросворды по алгебре нужны чтобы всё это было связанно с алгеброй за 7 или 8 класс !!!Замечательный кроссворд для 6- 7 класса по истории По горизонтали: 1 Математические ребусы – womanadviceru womanadviceru/matematicheskie-rebusy Cached Математические ребусы для учеников 5-9 класса с ответами Для учеников средней школы, особенно 8-9 класса, ребусы по математике уже должны быть довольно сложными – такими, чтобы ребятам пришлось серьезно потрудиться нетерпеливые бусы: ребусы с ответами для детей 9-11 лет bravelut8blogspotcom/2013/05/9-11html Cached картинки с ребусами с сообщение про ребусы ; мария семёнова бусый волк 3-я книга; ребусы по геометрии за 9-ый класс ; ребусы игры по русскому языку; тур от кемерово до шерегеша автобусы кроссворд на тему математика 7 класс фото / Блог им uponys кампусятарф/blog/105160html Cached Стенгазета по математике 5 класс – занимательно: «Математика – царица наук!» кроссворды по математике скачать с ответами 5 класс кроссворд из 15 слов с ответами ребусы в ребус по азбуке Найти бесплатно за 6 класс математические задачи ребусы storifycom/djibzirrona/nayti-besplatno-za-6 Cached Метаграммы в картинках для детей 6 7 летДемидова Н С Комплект кроссвордов по математике 5 11 классОгаркова М А Кроссворд Математические ребусы с ответами для 4 класса Математические ребусы для 1 класса с ответами – Официальный сайт sitesgooglecom/site/losucohou/matemat перейдите по ЭТОЙ ссылке самостоятельно Математические ребусы для 1 класса с ответами  Главная страница волкова Математика и конструирование: пособие Promotional Results For You Free Download | Mozilla Firefox ® Web Browser wwwmozillaorg Download Firefox – the faster, smarter, easier way to browse the web and all of Yahoo 1 2 3 4 5 Next 2,910 results Settings Help Suggestions Privacy (Updated) Terms (Updated) Advertise About ads About this page Powered by Bing™

  • а за неправильный списывали 10 очков Читать ещё Занимательная математика Математические ребусы с решением и ответами Математические ребусы Задача Эйнштейна На школьной викторине участникам предложили 20 вопросов За правильный ответ ученику ставилось 12 очков
  • если он ответил на все вопросы и набрал 86 очков? Задача 2 Разместите на трех грузовиках 7 полных бочек
  • сообразительность

деление и умножение) Разгадать такой ребус Читать ещё Математические головоломки

что их нужно вычеркнуть Скрыть 3 Математические ребусы womanadviceru › matematicheskie-rebusy Сохранённая копия Показать ещё с сайта Пожаловаться Информация о сайте Математические ребусы для учеников 5-9 класса с ответами Для учеников средней школы

  • особенно 8-9 класса
  • ребусы по математике уже должны быть довольно сложными – такими
  • задания по английскому языку для 2 класса ту-туru автобусы ребусы в картинках с ответами онлайн мерс Ребусы по математике – vremyazabavru vremyazabavru/zanimatelno/rebusi/rebusi-slova/82-rebusi Cached Ребусы по математике Я зашифровал слова по математике в ребусы Попробуйте их решить не заглядывая в ответы Ребусы для алгебры 7 класс Математические ребусы (5-6 классы kinderbooksru/klass/rebusy-dlya-algebry- 7 -klass Cached Нужны загадки

Яндекс Яндекс Найти Поиск Поиск Картинки Видео Карты Маркет Новости ТВ онлайн Знатоки Коллекции Музыка Переводчик Диск Почта Все Ещё Дополнительная информация о запросе Показаны результаты для Нижнего Новгорода Москва 1 Математические ребусы с картинками и ответами ped-kopilkaru › Блоги › 5-7 класс Сохранённая копия Показать ещё с сайта Пожаловаться Информация о сайте Математические ребусы с картинками и ответами , 5- 7 класс Математические игры ребусы в картинках для школьников 5- 7 7 Цифры под картинкой указывают на то, что из данного слова нужно взять буквы, стоящие на местах под номерами 7 ,2,4,3,8 и составить их в том порядке, в котором расположены цифры Читать ещё Математические ребусы с картинками и ответами , 5- 7 класс Математические игры ребусы в картинках для школьников 5- 7 классов Автор: Клочкова Наталья Константиновна, учитель математики МБОУ «Бухарайская СОШ» сБухарай Заинский район Описание: Данная работа может быть использована на уроках математики в 5– 7 классах 7 Цифры под картинкой указывают на то, что из данного слова нужно взять буквы, стоящие на местах под номерами 7 ,2,4,3,8 и составить их в том порядке, в котором расположены цифры В слове ватрушка нужно взять буквы 7 -К, 2-А, 4-Р, 3-Т, 8-А Можно прочитать слово Скрыть 2 Математические ребусы – Математика – 7 класс multiurokru › Обо мне › …-riebusy-18html Сохранённая копия Показать ещё с сайта Пожаловаться Информация о сайте Ребус – вид загадки, в которой разгадываемые слова даны в виде рисунков в сочетании с буквами или цифрами Правила разгадывания ребусов : 1 Запятая слева от слова означает количество букв, которые нужно удалить с начала слова , Было «лягушка» стало «ягушка» 2 Перевернутая запятая справа означает Читать ещё Ребус – вид загадки, в которой разгадываемые слова даны в виде рисунков в сочетании с буквами или цифрами Правила разгадывания ребусов : 1 Запятая слева от слова означает количество букв, которые нужно удалить с начала слова , Было «лягушка» стало «ягушка» 2 Перевернутая запятая справа означает количество букв, которые нужно удалить с конца слова Было «ПАЛЬМА» стало «ПАЛЬ» 3 Если над словом перечеркнутые буквы, то это означает, что их нужно вычеркнуть Скрыть 3 Математические ребусы womanadviceru › matematicheskie-rebusy Сохранённая копия Показать ещё с сайта Пожаловаться Информация о сайте Математические ребусы для учеников 5-9 класса с ответами Для учеников средней школы, особенно 8-9 класса , ребусы по математике уже должны быть довольно сложными – такими, чтобы ребятам пришлось серьезно потрудиться Читать ещё Математические ребусы для учеников 5-9 класса с ответами Для учеников средней школы, особенно 8-9 класса , ребусы по математике уже должны быть довольно сложными – такими, чтобы ребятам пришлось серьезно потрудиться, чтобы их расшифровать В противном случае подобные задачки не смогут заинтересовать и надолго увлечь школьников, а значит, будут абсолютно бесполезны В частности, для занятий с учениками 6- 7 класса и старше можно использовать такие математические ребусы с ответами , как: Статьи по теме: Как составить ребус ? Хороший, сложный ребус с трудом разгадывает взрослый, что и говорить о пол Скрыть 4 Математические ребусы в картинках с ответами pesochnizzaru › Математические ребусы Сохранённая копия Показать ещё с сайта Пожаловаться Информация о сайте Очень красивые математические ребусы в картинках с ответами для детей и учителей математики Разнообразить скучный урок помогут занимательные математические ребусы в картинках 5 Ребусы по математике VremyaZabavru › zanimatelno/rebusi/rebusi-slova/82… Перевести Сохранённая копия Показать ещё с сайта Пожаловаться Информация о сайте Ребусы по математике: интересные ребусы связанные с математикой попробуй реши! 6 Ребусы « Алгебра » – IgraZaru Игры, ребусы , загадки igrazaru › page-1-1-25html Сохранённая копия Показать ещё с сайта Пожаловаться Информация о сайте Ребусы « Алгебра » 04062011 17:12 Данные ребусы можно использовать на уроках математики как элемент занимательной деятельности 4-6 класс 8 марта безопасность биологические верность видеоролик воображения высказывание глагол дети дошкольного возраста дорожные живописи животных Читать ещё Ребусы « Алгебра » 04062011 17:12 Данные ребусы можно использовать на уроках математики как элемент занимательной деятельности 4-6 класс 8 марта безопасность биологические верность видеоролик воображения высказывание глагол дети дошкольного возраста дорожные живописи животных интернет ископаемые котиков кружок лето музыкальные мультфильмы неделя недоразвитие речи новоселы ориентирование отряда по значению пожарники познавательный интерес поле последовательность предпринимательства разум решение неравенств русские рыбак свойства слово старшая группа страна спорта счет терминология термины информатики технология урок-игра фгос физ-ра химии шекспир шоколад Скрыть 7 Ребусы по алгебре за 7 класс картинки с ответами — смотрите картинки ЯндексКартинки › ребусы по алгебре за 7 класс картинки с ответами Пожаловаться Информация о сайте Смотреть все результаты поиска на сервисе ЯндексКартинки 8 Математические игры ребусы в картинках для infourokru › matematicheskie-igri-rebusi-v…klassov… Сохранённая копия Показать ещё с сайта Пожаловаться Информация о сайте Подробнее о сайте Ребус – это головоломка в которой зашифровано слово Это слово дается в виде рисунков с использованием букв и цифр Существуют определенные правила для разгадывания ребусов 6Буквы, слова или картинки могут изображаться внутри других букв, над другими буквами, под и за ними Тогда добавляются Читать ещё Ребус – это головоломка в которой зашифровано слово Это слово дается в виде рисунков с использованием букв и цифр, а также определенных фигур или предметов Ребус — одна из самых интересных головоломок На этой картинке зашифровано слово КОМПЬЮТЕР Существуют определенные правила для разгадывания ребусов 6Буквы, слова или картинки могут изображаться внутри других букв, над другими буквами, под и за ними Тогда добавляются предлоги: В, НА, НАД, ПОД, ЗА У нас в букве О содержится число СТО, поэтому получается В-О-СТО-К На этой картинке зашифровано слово КАРТА Скрыть 9 Ребусы с цифрами и ответами steshkaru › Ребусы с цифрами и ответами Сохранённая копия Показать ещё с сайта Пожаловаться Информация о сайте Математические ребусы с цифрами для детей Принцип, на котором построены ребусы Как решать ребусные задачи Ребус – уникальное изобретение человечества, помогающее воспитывать у людей остроту ума, сообразительность, смекалку Взрослые иногда любят побаловаться решением таких Читать ещё Математические ребусы с цифрами для детей Принцип, на котором построены ребусы Как решать ребусные задачи Ситуации на уроках, где можно применить ребусы для детей Ребус – уникальное изобретение человечества, помогающее воспитывать у людей остроту ума, сообразительность, смекалку Взрослые иногда любят побаловаться решением таких задачек в свободное время, но больше всего удовольствия ребусы доставляют для детей Чтобы совместить приятное и полезное, предлагаем вам разгадывать ребусы с цифрами для детей, которые даются на нашем сайте с ответами Ребусы направлены на логическое развитие ребенка Как их решать? Скрыть 10 Ребусы по математике seninvg07narodru › Ребус на уроке Сохранённая копия Показать ещё с сайта Пожаловаться Информация о сайте Серия рисунков – ребусов разрабатывалась к урокам математики 5 класса Однако ребусы можно применять на занятиях в других классах Ребусы по математике интерактивные программы на уроке ребусы по математике и геометрии с ответами korebusru › indexphp?option=com_contenttask=view… Сохранённая копия Показать ещё с сайта Пожаловаться Информация о сайте В ребусах зашифрованы термины по математике и, в частности, ребусы по геометрии, от начальных классов до 7 класса школы Ответы на все ребусы приведены на отдельной картинке в конце Читать ещё В ребусах зашифрованы термины по математике и, в частности, ребусы по геометрии, от начальных классов до 7 класса школы По большей части ребусы составлены мной, за исключением нескольких давно известных Сначала идут ребусы для 1, 2, 3 классов , далее для средних классов школы Нажатием на маленькую картинку каждый ребус увеличивается в режиме полного просмотра Ответы на все ребусы приведены на отдельной картинке в конце Ребусы по математике: Ребусы по математике Ребусы по математике для начальных классов Ребусы по математике для 1 класса Ребусы по математике для 1 класса Ребусы по матем Скрыть Ребусы по информатике с ответами для детей umochkiru › Ребусы › Ребусы по информатике Сохранённая копия Показать ещё с сайта Пожаловаться Информация о сайте Отличная подборка ребусов по информатике для детей 1- 7 классов в которых зашифрованы различные термины и девайсы На все ребусы вы также сможете найти ответы под картинкой №1 Посмотреть правильный ответ Читать ещё Отличная подборка ребусов по информатике для детей 1- 7 классов в которых зашифрованы различные термины и девайсы На все ребусы вы также сможете найти ответы под картинкой №1 Посмотреть правильный ответ ДИСКОВОД №2 Посмотреть правильный ответ ДИСПЛЕЙ №3 Посмотреть правильный ответ ДРАЙВЕР №4 Посмотреть правильный ответ ГРАФИКА №5 Посмотреть правильный ответ ИНТЕРНЕТ №6 Посмотреть правильный ответ КЛАВИАТУРА № 7 Посмотреть правильный ответ КЛАВИША №8 Посмотреть правильный ответ КОМПЬЮТЕР №9 Посмотреть правильный ответ КУРСОР №10 Посмотреть правильный ответ МОДЕМ №1 Скрыть Ребусы Алгебра – презентация по Алгебре ppt4webru › Алгебра › rebusy-algebrahtml Сохранённая копия Показать ещё с сайта Пожаловаться Информация о сайте Презентация на тему Ребусы Алгебра к уроку по Алгебре Ребусы Алгебра Бельская ОА, учитель математики МОУ «Иланская СОШ № 1»Красноярского края В оформлении ребусов использован материал Яндекс Картинки – Скачать эту презентацию Читать ещё Презентация на тему Ребусы Алгебра к уроку по Алгебре Ребусы Алгебра Бельская ОА, учитель математики МОУ «Иланская СОШ № 1»Красноярского края № слайда 2 В оформлении ребусов использован материал Яндекс Картинки – Скачать эту презентацию Чтобы скачать материал, введите свой email, укажите, кто Вы, и нажмите кнопку Скрыть Математические ребусы 5egena5ru › zanimatelnaya-matematika1html Сохранённая копия Показать ещё с сайта Пожаловаться Информация о сайте Занимательная математика Математические ребусы с решением и ответами На школьной викторине участникам предложили 20 вопросов За правильный ответ ученику ставилось 12 очков, а за неправильный списывали 10 очков Читать ещё Занимательная математика Математические ребусы с решением и ответами Математические ребусы Задача Эйнштейна На школьной викторине участникам предложили 20 вопросов За правильный ответ ученику ставилось 12 очков, а за неправильный списывали 10 очков Сколько правильных ответов дал один из учеников, если он ответил на все вопросы и набрал 86 очков? Задача 2 Разместите на трех грузовиках 7 полных бочек, 7 бочек, наполненных наполовину, и 7 пустых бочек так, чтобы на всех грузовиках был одинаковый по массе груз Задача 3 На столе лежат карандаши Двое играющих берут по очереди 1, 2 или 3 карандаша Проигрывает тот, кто берёт последний карандаш Скрыть Математические ребусы : числовые, арифметические logiclikecom › Занимательная математика › matematicheskie-rebusy Сохранённая копия Показать ещё с сайта Пожаловаться Информация о сайте Математические головоломки, ребусы с ответами для 1-4 класса Лучшие ребусы на логику Математический ребус – это занимательная шифровка с картинками и цифрами на выполнение арифметических действий (сложение, вычитание, деление и умножение) Разгадать такой ребус Читать ещё Математические головоломки, ребусы с ответами для 1-4 класса Лучшие ребусы на логику Математический ребус – это занимательная шифровка с картинками и цифрами на выполнение арифметических действий (сложение, вычитание, деление и умножение) Разгадать такой ребус – расставить между цифрами математические знаки так, чтобы равенстве стало верным Популярные варианты головоломки — неполные примеры, неравенства, таблицы, в которых все или часть цифр заменили буквами, изображениями , звездочками или пропусками Какие еще ребусы и головоломки можно решать на ЛогикЛайк? Арифметические ребусы в таблицах Скрыть Ребусы для детей, детские ребусы в картинках , словами filipocru › guess/rebus Сохранённая копия Показать ещё с сайта Пожаловаться Информация о сайте В этой рубрике вы найдете много ребусов для детей в картинках , словами, из букв и цифр К каждому ребусу есть ответ , который ты можешь посмотреть, если не смог разгадать Вместе с « ребусы по алгебре за 7 класс картинки с ответами » ищут: ребусы по алгебре 7 класс с ответами ребусы по алгебре ребусы по алгебре 7 класс ребусы по алгебре 7 класс с ответами в картинках ребусы по алгебре 8 класс с ответами в картинках ребусы по алгебре 8 класс ребусы по алгебре 9 класс с ответами в картинках ребусы по алгебре и геометрии ребусы по алгебре 7 класс с ответами и вопросами ребусы по алгебре 8 класс с ответами 1 2 3 4 5 дальше Bing Google Mailru Ребусы по алгебре за 7 класс картинки с ответами — смотрите картинки ЯндексКартинки › ребусы по алгебре за 7 класс картинки с ответами Пожаловаться Информация о сайте 20 декабря 20 декабря 20 декабря 20 декабря 20 декабря Смотреть все картинки Нашёлся 201 млн результатов Дать объявление Регистрация Войти Войдите через соцcеть Попробовать ещё раз Включить Москва Настройки Клавиатура Помощь Обратная связь Для бизнеса Директ Метрика Касса Телефония Для души Музыка Погода ТВ онлайн Коллекции Яндекс О компании Вакансии Блог Контакты Мобильный поиск © 1997–2018 ООО «Яндекс» Лицензия на поиск Статистика Поиск защищён технологией Protect Алиса в ЯндексБраузере Выключит компьютер по голосовой команде 0+ Установить

Кроссворды, ребусы, чайнворды по русскому языку

Кроссворды, чайнворды, ребусы

Кроссворд может быть предложен классу в начале урока с целью актуализации знаний или постановки проблемы нового урока.

Кроссворд, предложенный в конце урока, может стать своеобразным подведением итогов работы на уроке.

Незаменимы кроссворды, чайнворды и другие головоломки в тех случаях, когда детям нужно дать своеобразную минутку отдыха: переключение внимания, возможность посмотреть на языковые явления под другим углом зрения – хорошая возможность поддержать умственную активность учащихся на уроке.

Ниже предложен небольшой набор головоломок и кроссвордов, который может быть полезен учителям.

Знаете ли вы пословицы?

1. Слово – …, а молчание – золото. (Серебро.)

2. Авось да как-нибудь до добра не …(доведут).

3. Доброе … и кошке приятно. (Слово.)

4. Делу …, потехе – час. (Время.)

5. Где тонко, там и … (рвется).

6. Копейка … бережет. (Рубль.)

7. Один с сошкой, а … с ложкой. (Семеро.)

Кроссворд «Слова на шипящий согласный звук»

Разгадка этого кроссворда поможет ученикам вспомнить правописание слов различных частей речи, которые заканчиваются на шипящий согласный. Все слова, которые учащиеся впишут в клетки кроссворда, оканчиваются на шипящий, но лишь некоторые из этих слов имеют конечный ь.

По горизонтали:

1. Имя существительное мужского рода, II склонения, обозначающее непромокаемое пальто.

3. Краткая форма прилагательного жгучий в мужском роде.

4. Имя существительное женского рода, I склонения, однокоренное со словом тучи, в родительном падеже множественного числа.

6. Частица.

8. Существительное рикша в родительном падеже множественного числа.

10. Глагол несовершенного вида без приставки, однокоренной со словом сжигать.

13. Глагол несовершенного вида, омоним к глаголу быть, в настоящем времени, во 2-м лице единственного числа повелительного наклонения.

14. Имя существительное женского рода, I склонения, которое обозначает места для зрителей в театре, в родительном падеже множественного числа.

По вертикали:

1. Краткая форма прилагательного, равнозначного причастному обороту имеющий сходство.

2. Имя существительное женского рода, III склонения, обозначающее едкое химическое соединение, которое окрашивает лакмусовую бумажку в синий цвет.

4. Имя существительное женского рода, III склонения, синоним к слову пробоина.

5. Имя существительное женского рода, III склонения, синоним к слову нелепость.

7. Краткая форма прилагательного пригожий.

9. Второе лицо единственного числа настоящего времени от глагола шить.

11. Наречие образа действия, однокоренное с глаголом скакать.

12. Имя существительное женского рода, III склонения, антоним к слову сладость.

14. Имя существительное женского рода, III склонения, антоним к слову правда.

15. Одно из трех наречий на шипящий согласный, в которых мягкий знак не пишется.

Ответы:

По горизонтали: 1 – плащ; 3 – жгуч; 4 – туч; 6 – иль; 8 – рикш; 10 – жечь; 13 – ешь; 14 – лож.

По вертикали: 1 – похож; 2 – щелочь; 4 – течь; 5 – чушь; 7 – пригож; 9 – шьешь; 11 – вскачь; 12 – горечь; 14 – ложь; 15 – уж.

 Используя шифр, как можно быстрее найдите слова, которые скрываются за этими цифрами:

421

9810

1086

983

285

4611

3467

10439

108910

34911

10471

9473

35121

25131

28517

109121

54687

281101

Шифр:

АВГИЛМНОРТЯ

1 2 3 4 5 6 7 8 9 10 11

Ответы:

Ива

рот

том

рог

вол

имя

гимн

тигр

торг

гиря

тина

ринг

глава

влага

волан

трава

лимон

врата

Примечание. По аналогии с этим заданием учитель может зашифровать название части речи, изучаемой в данный момент или морфологические признаки, которые будут изучаться на уроке.

Решите следующие задачи:

 Отделите плод от растения.

СОШИСНШАКА

Комментарий: зачеркнув буквы слова «шишка» (плод), получаем название растения – слово «сосна». Так же решаются остальные задачи.

 Растворите белое в прозрачном.

СВОАДХАРА

Ответ: вода; сахар.

 Уберите горячее – останется холодное.

КИАЙПЯСТОБЕКРГ

Ответ: кипяток, айсберг.

 Найдите друга и врага Буратино.

МАБАЛЗЬВИИЛИНОА

Ответ: Мальвина, Базилио.

 Зачеркните ядовитое – оставьте съедобное.

ПООПГАНЕНОККА

Ответ: поганка, опенок.

 Как можно быстрее из каждой строчки выберите лишь те буквы, которые не повторяются, и составьте из них пожелание.

АОБООУАДА –________________________

ИПЬРТПЕИР – _________________________

ЭПВФЭСФПЕ – ________________________

ВЮЛГВДЛАЮ – ________________________

ЖЗСУУЧЖАЗ – _________________________

ОАСАОТАЛО – _________________________

ГЦИДВГЦЫД! – _________________________

Ответ: Будьте всегда счастливы!

 Слово – не воробей: вылетит – не поймаешь. Но вы все-таки постарайтесь поймать слова, которые улетели со страницы.

Над городом низко повисли снеговые __________________. Вечером началась ______________. Снег повалил большими ___________________. Холодный ветер выл, как дикий ___________________. В конце пустынной и глухой __________ вдруг показалась какая-то девочка. Она была худа и бедно _______________. Она продвигалась медленно вперед, валенки свалились с ног и __________________ ей идти. На ней было плохое ___________________ с узкими рукавами, а на плечах ___________________.

 Перед вами несколько «крылатых выражений», в которых пропущены одинаковые слова. Отгадайте их. Составьте с ними предложения.

Из… вон плохо. Мастер на все …

Сидеть сложа … Золотые …

 Составьте анаграммы.

Например: Карета – ракета.

Урон – (руно).

Арап – (пара).

Мошкара – (ромашка).

Осколок – (колосок).

Соха – (хаос).

Ниша – (наши).

 Проанализировав образец, вставьте нужное слово:

багор (роса) тесак

гараж (….) табак

фляга (альт) жесть

косяк (….) мираж

книга (аист) салат

пирог (….) омлет

лоток (клад) лодка

олимп (….) катер

пирог (поле) слеза

рынок (….) осада

канва (внук) улика

холст (….) олень

гамма (гимн) диван

перец (….) клоун

аргон (роза) гроза

атлет (….) норма

порыв (вода) осада

откос (….) отдых

кочан (туча) штука

хвост (….) отрез

Искомые слова: жаба, кожа, грот, порт, роса, слон, плен, тема, стыд, трос.

 Подберите слово, которое служило бы окончанием первого и началом второго (количество точек соответствует количеству букв):

у (…) ова

с (…) окно

г (…) ожа

у (…) ь

ме (…) олад

по (…) арь

пи (…) ок

го (…) кот

пе (…) ол

са (…) он

ам (…) ан

та (…) ат

апо (…) б

кес (…) ет

вам (…) амида

сна (…) овой

бал (…) еда

каб (…) ошко

обы (…) ка

дик (…) ец

беге (…) ор

свир (…) ник

контр (…) ив

ковер (…) лета

Искомые слова: гол, тол, рог, клад; шок, ток, лот, бой; сок, рай, бар, пир; стол, сон, пир, ряд; бес, лук, чай, образ; мот, ель, акт, кот.

Словесные тесты

Найдите общие окончания сгруппированных столбиками слов:

Искомые окончания указанных слов: -ОЛ; -ОМ; -АЙКА; -ОТ; -ЕСТЬ; -ИНА; -ЕНЬ; -ОЛЬ.

Чайнворд «Местоимение»

Почти все слова в этом чайнворде – местоимения. Отгадайте, какие это слова.

1. Личное местоимение 3-го лица мужского рода. (Он.)

2. Личное местоимение 1-го лица множественного числа в дательном падеже. (Нам.)

3. Назовите часть речи, которая употребляется вместо имени существительного, прилагательного или числительного. (Местоимение.)

4. Личное местоимение 3-го лица единственного числа мужского рода родительного падежа. (Его.)

5. Местоимение, которое изменяется по родам. (Он.)

6. Личное местоимение 1-го лица множественного числа родительного падежа. (Нас.)

7. Указательное местоимение с количественным значением. (Столько.)

8. Название разряда местоимений, сходных с вопросительным в дательном падеже единственного числа мужского рода. (Относительному.)

9. Название разряда местоимений, к которому относится местоимение «тот» в творительном падеже множественного числа. (Указательные.)

 Если вы впишете изображенные здесь предметы в паутину правильно, то в обозначенном поясе прочитаете географическое название. Назовите его.

Ответы: 1 – поезд; 2 – ручка; 3 – индюк; 4 – белка; 5 – акула; 6 – лампа; 7 – такси; 8 – игла; 9 – кошка; 10 – арбуз. (Прибалтика.)

 Названия всех нарисованных предметов впишите в клеточки от края к центру. Если вы подберете все слова правильно, то в третьем ряду прочитаете названия трех континентов.

Ответы: 1 – поезд; 2 – диван; 3 – парта; 4 – овощи; 5 – репка; 6 – флаги; 7 – краны; 8 – козел; 9 – птица; 10 – рояль; 11 – шкафы; 12 – кофта; 13 – марка; 14 – книга; 15 – кукла; 16 – глаза. (Европа, Азия, Африка.)

 Если вы впишете в схему правильно следующие слова, то в обозначенных рядах прочитаете имена 2 девочек и 1 мальчика.

1. Н…прав… . 2. К…ток. 3. М…л…дой. 4. Лош…дь. 5. …лта. 6. Вет…р. 7. Кин… . 8. Мама.

Ответы: I. – Наташа, II. – Володя, III. – Марина.

 Впишите названия предметов в клеточки, и вы увидите, что некоторые буквы связаны между собой линиями. Найдите начало этой головоломки и, двигаясь по линиям от буквы к букве, прочитайте пословицу.

Ответы: 1 – галстук; 2 – окно; 3 – учебник; 4 – тетрадь; 5 – пенал; 6 – глобус; 7 – линейка; 8 – тысяча; 9 – компас; 10 – портфель.

(Кончил дело – гуляй смело.)

Кроссворд «Раз – рас»

Заполните клетки словами (именами существительными, прилагательными и глаголами) с приставками РАЗ- (левая сторона) и РАС- (правая). В правой стороне во всех словах должно быть стечение согласных на стыке приставки и корня.

1. Движение с целью набрать скорость. 2. Место раздвоения железнодорожного пути. 3. Сбор сведений о противнике. 4. Слишком свободный, фамильярный. 5. Разделить линиями на графы. 6. Разногласия, споры. 7. Раздвоение (ствола, дерева, дороги и т. п.). 8. Расставание, жизнь вдали от близких. 9. Беспорядочный, небрежный в делах человек. 10. Освобождение от груза. 11. Беседа. 12. Полное поражение противника. 13. Часть текста, выделенная заглавием, номером и т. п. 14. Соленая вода для засолки овощей. 15. Молодые растения, выращенные для пересадки. 16. Здравый смысл, способность правильно мыслить. 17. Продление срока уплаты, разрешение на уплату по частям. 18. Невнимательный, постоянно забывающий или путающий что-либо. 19. Время перед восходом солнца. 20. Вид казни. 21. Быстро разрубить чем-либо острым. 22. Снять седло с оседланного животного. 23. Питомник. 24. Посылка каких-либо предметов, документов, писем в разные места. 25. Небольшое художественное произведение в прозе. 26. Затраты, издержки.

Ответы: 1 – разбег; 2 – разъезд; 3 – разведка; 4 – развязный; 5 – разграфить; 6 – раздоры; 7 – развилка; 8 – разлука; 9 – разгильдяй; 10 – разгрузка; 11 – разговор; 12 – разгром; 13 – раздел; 14 – рассол; 15 – рассада; 16 – рассудок; 17 – рассрочка; 18 – рассеянный; 19 – рассвет; 20 – расстрел; 21 – рассечь; 22 – расседлать; 23 – рассадник; 24 – рассылка; 25 – рассказ; 26 – расход.

Чайнворд «Имя существительное»

Цель: повторить формообразование некоторых существительных.

Все слова в этом чайнворде (в виде буквы «С») – существительное. Какие же это слова?

1. Существительное 1-го склонения, обозначающее неопрятного человека. (Неряха.)

2. Существительное женского рода, 1-го склонения, синоним выражения вооруженные силы в родительном падеже единственного числа. (Армий.)

3. Существительное мужского рода, 2-го склонения, синоним слова истукан в творительном падеже единственного числа. (Идолом.)

4. Существительное мужского рода 2-го склонения, с уменьшительно-ласкательным суффиксом, образованного от существительного медведь. (Медвежонок.)

5. Существительное мужского рода 2-го склонения, с суффиксом –щик, обозначающее профессию и образованное от слова камень. (Каменщик.)

6. Существительное женского рода, 1-го склонения, являющееся названием домашней птицы, в дательном падеже единственного числа. (Курице.)

7. Существительное женского рода, 3-го склонения, являющееся названием породы деревьев, в дательном падеже множественного числа. (Елям.)

8. Существительное мужского рода, 2-го склонения, обозначающее сооружение для переезда и перехода через реку, в родительном падеже множественного числа. (Мостов.)

9. Существительное среднего рода, в котором при склонении появляется суффикс –ен, в родительном падеже множественного числа. (Времен.)

бесплатная математика тайна Координатная диаграмма Mystery Picture. Скачайте ЗДЕСЬ! 12 ноя 2018 · Представляем Esti-Mysteries! Каждое изображение предлагает учащимся задуматься, какое число представлено на изображении. Среди наиболее эффективных печатных листов – график умножения. ru / Используйте эту загадочную игру, чтобы побудить детей использовать различные математические навыки для решения головоломки. Неправильные ответы окрашены в фиолетовый цвет. Удерживайте своих учеников увлекательными занятиями по математике в конце учебного года или во время летней школы! Эта загрузка включает в себя 4 загадочные картинки, каждая из которых использует различные математические навыки – диаграмма сотен, диаграмма 120, умножение, построение координатных графиков.Помимо рождественских мероприятий, можно найти бесплатные задания по математике с рождественскими загадочными картинками в различных формах, и они также варьируются от снежного кома до переезда в зону отдыха, а в случае, если вы живете в теплых районах, также походов на пляж. Это очень важный инструмент, чтобы научить ребенка пользоваться таблицей умножения. Coloring Squared будет часто пытаться дать вам новую страницу раскраски математических фактов. Эта образовательная игра-головоломка теперь является заброшенной, и ее действие разворачивается в аниме / манге, математике / логике, фэнтези, медитации / дзен, умственном обучении и элементах головоломки.21 мая 2019 г. · 10 супер веселых математических загадок и головоломок для детей от 10 лет (включая ответы!) Сообщение Автор: Энтони Персико Некоторые из них существуют уже много веков, а другие совершенно новые, но все они были выбраны в качестве самые веселые и увлекательные математические загадки для тех, кто любит проверять свои навыки решения проблем и получать удовольствие от решения математических головоломок. Дело о шагающем скелете – это 9-страничная электронная книга, которая содержит 6-страничный рассказ с иллюстрациями и 5-ти словесными задачами, подходящими для 1-3 классов (сложение и вычитание одиночных цифр, но пара многоступенчатых) .Реальный. После долгих, веселых и полезных выходных с учениками 9-го класса они смогут развить свои навыки счета и грамотности, а также общую уверенность в себе, командообразование и социальные навыки. 05 января 2020 г. · БЕСПЛАТНЫЕ задания Esti-Mystery для классов K-2: этот набор «загадок куба unifix» не требует сверхсложных математических навыков в рамках подсказок. Эта бесплатная математическая загадка «Дело о супер плохом супергерое» – отличный способ добавить азарта и веселья к вашим урокам математики! Студенты будут работать над различными математическими вопросами, чтобы выяснить, какой из супергероев стал супер плохим! 23 апр.2020 г. · Загрузите бесплатные упражнения с изображениями «Цифровая математика».Математика «Тайны убийства» – отличный способ вовлечь учащихся в процесс обучения, а также начать изучать их навыки рассуждения и решения проблем. Бесплатная распечатка загадочного цвета по номеру. Пересмотр чисел – Эта математическая тайна убийства вовлекает студентов, практикующих и применяющих свою базовую алгебру, число и 28 октября 2020 г. · Бесплатные рабочие листы по математике для промежуточной школы Рабочий лист Благодарения | Бесплатные печатные листы с изображениями Mystery Mystery, исходное изображение: www. Нажмите на морскую фигуру и введите свой ответ на цифровой клавиатуре.Поэкспериментируйте с этими дизайнами и адаптируйте их к своим потребностям. Последняя неделя (или месяц) в школе может быть тяжелой! Иногда бывает сложно удержать внимание детей и вообще хоть как-то учиться! Я люблю использовать забавные и увлекательные занятия, которые нравятся детям, но которые все же имеют некоторую образовательную ценность. Вы разгадали все загадки нашего шаблона «Math Mystery Escape Room» и снова вернулись? Добавьте эту редактируемую инфографику в свою презентацию, чтобы было больше разнообразия. Одним из самых последних бума в играх для всех возрастов стали квесты.учительспайучитель. Рабочие листы для второго класса. •. Я собрал некоторые из моих бестселлеров Math Mystery Picture Freeware Lilly Wu & the Terra Cotta Mystery для Mac OS v. Идеально подходит для использования Бросьте вызов ученикам 6-го класса в этой чудесной мистической игре и узнайте, кто найдет ангела, у которого есть был перемещен с вершины дерева. Math Mystery: Case of the Super Bad Superhero Video Hook. Математические пазлы Cuemath для детей – это эффективный способ помочь ребенку увлечься математикой и развить к ней интерес.Студентам понравится этот рабочий лист, поскольку он позволяет им использовать свои знания об умножении, чтобы найти скрытые вдохновляющие цитаты. Напишите нам по адресу: [адрес электронной почты защищен] Электронная почта Мы надеемся, что детям понравились эти бесплатные раскраски с математическими фактами. После первоначального задания от 27 июля 2016 г. · Задания-раскраски для 2-го класса по математике. M & m Math Рабочие листы. Рабочие листы для печати – прекрасный ресурс как для мам, так и для пап и детей. Вовлеките своих учеников в увлекательную и мотивирующую математическую задачу в нашем математическом проекте для второго класса «Тайна пропавших пряничных домиков».Отлично подходит для использования во время летней школы, детского сада или дома с вашими детьми, когда они математика Mystery Escape Room Шаблоны инфографики Бесплатная тема Google Slides и шаблон PowerPoint. 30 декабря 2019 г. · Тайна оценки дает детям возможность попрактиковаться и отточить свои навыки оценки, которые развивают чувство числа. Вопросы основаны на бесплатных рабочих листах математических головоломок в формате pdf для печати, рабочих листах математических головоломок для практики и улучшения различных математических навыков, сложения, вычитания, соотношений, дробей, деления, умножения, для детского сада, 1, 2, 3, 4, 5 класса, 6 классов.Поделитесь с нами своими отзывами о страницах, которые вы использовали и которые вам понравились. Эти рабочие тетради идеально подходят как для детей, так и для взрослых. Или скажите нам, что вы хотели бы видеть в одном из наших следующих заданий или книжек-раскрасок. В конце студентам нужно будет позвонить… Mystery Media Math Worksheets. 120 Chart Mystery Picture. Эта бесплатная загрузка включает в себя четыре различных математических загадочных картинки, которые охватывают самые разные навыки для ваших элементарных детей! Включает: Таинственная картинка с сотнями диаграмм. Рабочие листы для третьего класса.Ниже приведены несколько загадок убийств, которые я разгадал и использовал с моими учениками разных способностей и возрастов. Скрытые изображения включают в себя изображения животных, детей, транспортных средств, фруктов, овощей, монстров и многое другое! Бесплатные PDF-файлы Math Mystery для третьеклассников. com Электронная почта Мы надеемся, что детям понравились эти бесплатные раскраски с математическими фактами. 30 мая 2017 г. · Бесплатные летние математические загадки {для классов K-5} Все бесплатные подарки. Раскраска по номерам. Поощряйте вашего ребенка выполнять упражнения с их собственной скоростью, когда вы их используете.clubdetirologrono. Есть только одна проблема: она не знает, кто это придумал! Эта математическая игра охватывает сбор данных и блок-схемы, измерения в сантиметрах, добавление двузначных чисел и числовые связи к 20. Saxon Math 5 4 Тесты и рабочие листы. Рабочие листы с загадочными картинками требуют, чтобы учащиеся ответили на основные факты и раскрасили их в соответствии с кодом. Вашим ученикам это понравится, так как это позволит им использовать свои математические знания (умножение, базовую алгебру, шансы / эвенты) в египетском математическом приключении, где им понадобится тема Math Mystery Escape Room Presentation Бесплатная тема Google Slides и шаблон PowerPoint.Улики нужны, чтобы сузить список возможных подозреваемых, мест, лечения и сценариев до тех пор, пока не останется только один. Лабиринты, головоломки, числовые шаблоны, задачи со словами, упражнения и многое другое! Загадочные числа! 3–5 классы. Основные навыки – понимание разницы между четным и нечетным, сложение и больше / меньше. Чтобы прочитать подробный пост о том, как назначить эти бесплатные цифровые математические Mystery Multiplication – Advanced. Совершенно новый для перезапуска Используйте эту загадочную игру, чтобы побудить детей использовать различные математические навыки для решения головоломки.Пишите нам по адресу: coloringsquared @ yahoo. M & math рабочие листы. Используя наш ресурс “Проект” Пряничный домик “, учащимся будет представлен увлекательный сценарий, в котором они должны будут найти недостающие записи ежегодного конкурса имбирных пряников в Cookie Town. 09 декабря 2020 г. · Используйте бесплатные рабочие листы по математике с рождественскими загадочными картинками, чтобы продолжать учиться во время празднования Рождества. Рабочие листы Mystery Media по математике. Щелкните Получить книги и найдите свои любимые книги в онлайн-библиотеке. От рабочих листов с загадочными медиа-графиками до видеороликов с загадочными медиа-головоломками можно быстро найти образовательные ресурсы, проверенные учителями.Позвольте своим детям практиковаться и изучать другие математические навыки, посетив наш раздел забавных математических игр Здесь. Бросьте вызов своим ученикам 6-го класса в этой чудесной загадочной игре и узнайте, кто нашел ангела, который был перемещен с верхушки дерева. Вопросы основаны на вопросах. Пригласите своих учеников принять участие в увлекательной и мотивирующей математической задаче с помощью нашего математического проекта для второго класса «Тайна пропавших пряничных домиков». Я собрал некоторые из моих самых продаваемых загадок 30 декабря 2019 г. · Тайна оценки дает детям возможность попрактиковаться и отточить свои навыки оценки, которые развивают чувство числа.Мы включили в эту математическую коллекцию MMs. Чтобы увидеть графические изображения с координатной сеткой, перейдите к Graph Art Mystery Pictures. Последняя возможность, оставшаяся в списке, если все сделано правильно, – это 13 октября 2020 г. · Бесплатные распечатываемые рабочие листы с математическими загадочными изображениями – Бесплатные печатные математические задания с изображениями-загадками могут помочь учителю или ученику понять и реализовать программу урока изнутри намного быстрее. Базовые таблицы фактов сложения, вычитания, умножения и деления. Но он также переплетается с другими математическими навыками и математической лексикой, обеспечивая дополнительный обзор навыков.Math Mysteries – это легкое подготовительное занятие, которое в то же время делает математику увлекательной! В каждом досье есть пять листов математических загадок, которые нужно разблокировать. Они будут использовать различные математические навыки, включая задачи со словами, логические навыки, сложение до 20, вычитание до 20, нечетные / четные числа и многое другое! Эта бесплатная математическая загадка «Дело о супер плохом супергерое» – отличный способ добавить азарта и веселья к вашим урокам математики! Студенты будут работать над различными математическими вопросами, чтобы выяснить, какой из супергероев стал супер плохим! Это будет похоже на игру, но с удобством простых в использовании листов Math Mystery Picture.Подписчики могут просматривать в полноэкранном режиме. Math Mysteries – это легкое подготовительное занятие, которое в то же время делает математику увлекательной. Египетская математическая загадочная история: используйте математику, чтобы победить мумию и получить сокровище! Это сделано не в типичном математическом стиле CSI, а в стиле математической истории / квеста. Загрузите полную книгу Math Murder Mystery Book или читайте онлайн в любое время в любом месте, доступном в PDF, ePub и Kindle. com. издает Math Blaster Mystery: The Great Brain Robbery для Windows 3. Эта математическая загадка «Дело о супер плохом супергерое» – отличный способ добавить азарта и веселья к вашим урокам математики! Студенты будут работать над различными математическими вопросами, чтобы выяснить, какой из супергероев стал плохим! Легкая подготовка! Просто распечатайте и решите! 23 мая 2017 г. · Бесплатные летние задания по математике Mystery Picture.Вашим второклассникам наверняка будет интересно поработать над математическими задачами из этого сборника, чтобы они могли решать загадочные картинки. Я надеюсь создать больше в будущем, так что следите за обновлениями! Назначение в Google Slides. Математика Тайна убийства. Скрытые изображения включают в себя изображения животных, детей, транспортных средств, фруктов, овощей, монстров и многое другое! Загадочное дополнение – продвинутый уровень. Позвольте своим детям практиковать и изучать другие математические навыки, посетив наш раздел забавных математических игр Здесь. Это загадочное математическое задание «Дело о супер плохом супергерое» – отличный способ добавить азарта и веселья к вашим урокам математики! Студенты будут работать над различными математическими вопросами, чтобы выяснить, какой из супергероев стал плохим! Легкая подготовка! Просто распечатайте & amp; Решать! Или откажитесь от бумаги с новой игрой G Mathematics Magic и Mystery Free Murderous Maths: The Magic of Maths, где есть множество невероятных трюков, которые поразят как детей, так и учителей.0 Оживите National Geographic в Лилли Ву и Тайне Терракоты и возглавьте судебно-медицинскую экспедицию, чтобы раскрыть тайну 2000-летней давности. Создайте бесплатную учетную запись, чтобы получить доступ к неограниченному количеству книг, быстрой загрузке и без рекламы! Мы не можем гарантировать, что книга Math Murder Mystery находится в библиотеке. Икс. Интерактивные математические игры: загадочные картинки, раскраски, открытие картинок, пиксель-арт, головоломки с картинками, настольные игры и игры с призами. 13 янв. 2021 г. · Раскраски с загадочной математикой | Раскраски – Бесплатная распечатка по математике | Бесплатные распечатанные математические листы с изображением загадок, исходное изображение: freeprintablehq.Эти математические загадки были разработаны Стивом Уайборни, который является мастером разработки значимых математических задач. По мере того, как вы щелкаете по каждой Esti-Mystery, будут появляться подсказки, которые позволят учащимся использовать математические концепции, чтобы сузить набор возможностей до небольшого набора чисел. & nbsp; В этот проект добавлено слово. В конце концов, ученикам нужно будет позвонить… 27 июля, 2016 · Задания-раскраски по математике для 2-х классов. Рабочие листы по математике; Лучшие ресурсы; Книга по математике на этой неделе; Математика; Добавление; Оценка; Кодирование для Детей Math Murder Mystery.& nbsp; Этот проект включает в себя добавление слова d для решения математических задач, созданное синди митчелл вибли, вычитание загадочных картинок с математическими рабочими листами, бесплатные двойные, загадочные листы с скрытыми картинками, бесплатные математические загадки, скачать pdf epub, загадочные картинки, математические, arubabooks com, пасхальные математические упражнения, кресты загадочные картинки печатные формы, математические загадочные изображения – решать бесплатные математические загадки PDF для второклассников. M M Математическая деятельность. 23 апр.2020 г. · Загрузите бесплатные упражнения с изображениями «Цифровая математика». Чтобы прочитать подробный пост о том, как назначить эти бесплатные цифровые математические PDF-файлы Free Math Mystery для третьеклассников.1. Нажмите здесь, чтобы найти распечатку, и отправляйтесь БЕСПЛАТНО разгадывать загадку математики! Это супер веселое и увлекательное занятие, в котором ученики полюбят математику! Студенты должны использовать свои математические навыки, чтобы выяснить, какой супергерой оказался супер ПЛОХОЙ! Загрузите БЕСПЛАТНЫЙ комплект оценок! НЕТ ПОДГОТОВКИ, просто распечатай и работай! Получите бесплатный комплект оценок K-6 здесь! 30 сентября 2021 г. · Mystery math. Загадочная картинка умножения. Математические загадочные картинки для 3-го класса – Сделайте математический класс чем-то таким, чего стоит ожидать с этим набором загадочных математических картинок для 3-го класса, раскрашенных по номерам.Детям больше понравится эта тема с помощью наших рождественских задач по математике (6 класс), которые по-новому знакомят детей с этой темой! Эта загадочная игра позволяет вашим ученикам использовать и применять свои навыки решения задач и рассуждения. Отлично подходит для использования во время летней школы, детского сада или дома со своими детьми, когда они 12 ноября 2018 г. · Представляем Esti-Mysteries! Каждое изображение предлагает учащимся задуматься, какое число представлено на изображении. Это бесплатный образец моей работы, включающий 2 рабочих листа, на каждом из которых приводится собственная цитата.Это загадочное задание по математике «Дело о супер плохом супергерое» – отличный способ добавить азарта и веселья к вашим урокам математики! Студенты будут работать над различными математическими вопросами, чтобы выяснить, какой из супергероев стал плохим! Легкая подготовка! Просто распечатайте & amp; Решать! Или откажитесь от бумаги с новым G FREE – МНОЖЕНИЕ ТАЙНЫ СКРЫТЫЕ РАБОЧИЕ СООБЩЕНИЯ. Сколько ракушек у каждого морского существа? Используйте подсказки, чтобы разгадать тайну. БЕСПЛАТНО – УМНОЖЕНИЕ ТАЙНЫ СКРЫТЫЕ РАБОЧИЕ СООБЩЕНИЯ.Рабочие листы по математике; Лучшие ресурсы; Книга по математике на этой неделе; Математика; Добавление; Оценка; Программирование для детей Загадка глубоководной математики | Математическая площадка. Отображение 8 лучших листов, найденных для – M M Math Activities. Вопросы основаны на 1994 г., Davidson & Associates, Inc. Правильные ответы отмечены зеленым цветом. Тайна убийства по математике. Бросьте вызов своим ученикам 6-го класса в этой чудесной загадочной игре и узнайте, кто нашел ангела, который был перемещен с вершины дерева. Наведите указатель мыши на изображение, чтобы увидеть, как выглядит PDF-файл.16 мая 2013 г. · Это наша первая математическая загадка, и Питер хотел поделиться ею с другими детьми. Случайное действие с M M S Вероятностные действия Математические данные Действия Вероятность Math M и M Math. Эти бесплатные задания по математике – такой увлекательный способ для детей практиковать различные математические навыки, весело проводя время и оставаясь вовлеченными. Рабочие листы по математике; Лучшие ресурсы; Книга по математике на этой неделе; Математика; Добавление; Оценка; Программирование для детей Эта бесплатная загрузка включает в себя четыре различных математических загадочных картинки, которые охватывают самые разные навыки для ваших элементарных детей! Включает: Таинственная картинка с сотнями диаграмм.Сохранить в избранное. Эльфийка Куки хочет вручить главный приз за чудесное рождественское изобретение. Найдите БЕСПЛАТНОЕ задание по математике по этой ссылке в моем магазине TPT: https: // www. Нажмите здесь или на показанное изображение, чтобы загрузить бесплатные цифровые математические загадочные картинки для базового умножения (с летней темой). Метры сантиметры и миллиметры. Я хотел, чтобы вы могли использовать их с детьми в младших классах (так как есть много esti Mystery Addition- Advanced. В онлайн-классах Cuemath Live математические головоломки помогают детям узнать и понять, почему ‘любых математических понятий в увлекательной игровой форме.Идеально подходит для использования бесплатных заданий по математическим головоломкам в формате pdf для печати, рабочих листов по математическим головоломкам для практики и улучшения различных математических навыков, сложения, вычитания, соотношений, дробей, деления, умножения для детского сада, 1, 2, 3, 4, 5 и 6 классов. Теперь у меня есть возможность написать блог о математических расследованиях убийств и о том, к чему они могут привести. Math Mystery Free Activity {4-го класса Math Math Spiral Review} – Super Bad Superhero. Разнообразные бесплатные рабочие листы по математике для школьников K-8. Поделиться в Google Classroom.23 мая 2017 г. · Бесплатные летние задания по математике Mystery Picture. Было бы здорово использовать в качестве разминки по математике. 2 октября 2020 г. · Бесплатные распечатываемые рабочие листы с изображениями Math Mystery – Бесплатные рабочие листы с изображениями Math Mystery могут помочь учителю или ученику понять и реализовать программу урока изнутри намного быстрее. бесплатная математика тайна

axg w8p bod 94x gwp osi ksm tpg ysg 4ow ais fqe tjc tg7 irv fgv jl4 1k4 wrv zu7

Вопросы физики делятся на несколько категорий.
  1. Вопросы о физических явлениях, зависящие от данных реального мира.
  2. Вопросы, требующие теории и математики. Обычно такого рода задачи в конце главы в учебнике. Иногда это идеализированные предположения.
  3. Вопросы, требующие хитрых, изворотливых, умных или проницательных подходов. Это более правильное название «головоломки».
Когда мы смотрим на историю наук, мы обнаруживаем, что многие достижения возникли в виде головоломок, часто решаемых с помощью беспорядочного процесса мозгового штурма, а иногда и бессмысленной удачи.После нахождения ответа опубликованные статьи полируются, обрабатываются и представляются «надлежащим» академическим языком и стилем, часто скрывая мыслительные процессы, которые на самом деле привели к результатам. В конечном итоге они попадают в учебники, чтобы служить для учащихся, казалось бы, недостижимыми образцами. Азарт от игры ушел.

Так было не всегда. В книгах и опубликованных статьях пионеров науки XVII и XVIII веков творческий процесс часто подробно описывался со всеми его фальстартами, тупиками и несостоятельными гипотезами.В наши дни это не считается правильным стилем.

Есть задачи по физике и головоломки по физике. Мне нравятся головоломки. Этот сборник побуждает искать простые, умные и проницательные методы, чтобы прийти к ответу. Особенно мне нравятся те, которые уступают только элементарной математике. Некоторые из них также можно решить с помощью утомительного формального математического анализа, но только в крайнем случае.

Некоторые из этих загадок довольно старые, их источники неизвестны. Некоторые из них – мое собственное изобретение.Я старался не включать головоломки, правильные ответы на которые можно легко найти в Интернете.

Темы.

Несколько категорий головоломок.
  1. Определите это! Головоломки, требующие осторожного и точного определения или сформулированные с неоднозначными определениями.
  2. Самостоятельная ссылка. Головоломок, на которые есть простые или концептуальные ответы, которые возможны только из-за особого способа постановки головоломки.
  3. Головоломки с трюками. Головоломки, описывающие особые ситуации, которые (а) нефизичны или (б) имеют особые условия, позволяющие легко разрешить беспорядочную и сложную ситуацию.
  4. Концептуальные путаницы. Головоломок, основанных на распространенных заблуждениях о природе, физике или математике.
  5. Парадоксы (очевидные). Головоломки, предназначенные для описания противоречивых или парадоксальных ситуаций или объяснений.
  6. Визуальный обман. Головоломки, в которых прилагаемая диаграмма вводит в заблуждение или вводит в заблуждение. (Картинки могут лгать.)
  7. Разве это не предел? Головоломок, требующих особой осторожности при поиске пределов нуля и бесконечности. Многие классические математические доказательства парадоксов (иногда называемые «пуфами») зависят от такого рода обмана, такого как скрытое деление на ноль. Подобно доказательству того, что 2 + 2 = 5 (ну, это так, если значение 2 достаточно велико).
Для тех, кто хочет ответов, мои ответы можно найти здесь: Puzzle Answers.Однако они приходят без каких-либо гарантий.

Пазлов.

  1. Вращение по центростремительным кругам. Две одинаковые гири (черные) соединены шнурами T 1 и T 2 одинаковой длины и повернуты вокруг «неподвижной» оси. Если дан T 1 , что будет T 2 ? Объясните свой ответ, не используя слова «центробежный». Сначала сделайте быстрое предположение. Тогда решите это правильно.

    Сборщики нитров могут заметить, что из-за силы тяжести это движение не будет лежать в одной плоскости, поскольку шары будут вращаться ниже, чем рука свингера.Не обращайте внимания на это отвлечение.

  2. Водный мост. В Европе есть несколько необычных мостов. Обычно мосты (с улицей или железной дорогой) пересекают реку. Но бывают исключения. Иногда канал проводят через мост, пересекающий шоссе, или даже через реку. Их называют водяными мостами.
    Магдебургский водный мост. Фотография Сандры Сандрок Дуглас.

    Проектировщики мостов учитывают максимальную нагрузку, которую мост может выдержать.Как это применимо к водяным мостам? Предположим, что уровень воды на таком мосту остается почти постоянным, когда корабли пересекают его, как и масса воды над мостом – она ​​может составлять 10 000 тонн. Корабль, плывущий по каналу, весит 1000 тонн. Какая дополнительная нагрузка на мостик, когда корабль находится посередине моста?

    Магдебургский водный мост – это судоходный акведук в Германии, который соединяет канал Эльба-Хафель с каналом Миттелланд, позволяя судам пересекать реку Эльба.Это самый длинный судоходный акведук в мире, длина которого составляет 918 метров.

    Водные мосты в США – редкость. Стэнтон де Риэль сообщает мне об одном.

    Канал D&R (Делавэр и Раритан) имеет то, что вы могли бы назвать водным мостом, через приток реки Миллстоун, к северу от Принстона, штат Нью-Джерси. Канал судоходен, хотя в настоящее время только на каноэ (коммерческое судоходство остановилось некоторое время назад; шлюзы больше не обслуживаются).

    Пазл предоставил Ганс-Петер Граматке.

  3. Могучий мускус. Скоростной поезд едет со скоростью 100 миль в час. Муха на встречных курсах движется со скоростью 5 миль в час, точно встречаясь с поездом. Муха ударяется о оконное стекло локомотива, застревает (RIP) и продолжает двигаться вместе с поездом – конечно, со скоростью 100 миль в час, со скоростью поезда. Когда муха изменила направление своего движения, должна была быть точка, в которой она имела нулевую скорость, хотя бы на мгновение. Если бы скорость мухи была равна нулю, то в этот момент скорость поезда тоже должна была быть нулевой.Объясните, как эта сильная муха могла остановить (даже на короткое время) поезд, движущийся со скоростью 100 миль в час.

    Пазл предоставил Ганс-Петер Граматке.

  4. Дрэг-рейсинг. Дик и Джейн проводят лабораторный эксперимент, измеряя трение по времени, когда деревянные блоки скользят по наклонной плоскости с постоянной скоростью. Размеры блоков 3 х 4 х 5 дюймов. Дик предлагает гонку и предсказывает, что если блок скользит по поверхности 3х4, он будет иметь меньшую площадь контакта и меньшее трение, чем если бы блок скользил по поверхности размером 4х5 дюймов, поэтому с меньшим сопротивлением трения он выиграет гонку, когда оба скользить по самолету.Джейн не соглашается, утверждая, что они все еще весят одинаково, поэтому гонка закончится вничью. Кто и почему прав?
    Скатывание по склону.
  5. От гонок. Стандартная физическая задача (и демонстрация) – это гонки цилиндров, катящихся по наклонной плоскости. Цилиндры имеют одинаковую массу и одинаковый внешний радиус, но один из них сделан из цельного дерева, а другой – из металла. Обруч, имеющий больший момент инерции, меньше ускоряется под действием силы тяжести и проигрывает гонку.

    Но что, если мы по-другому воспользуемся гандикапом этой гонки. Сделайте два твердых цилиндра одинаковой длины и радиуса, но из материалов очень разной плотности. У них будут очень неравные массы. Кто победит и почему?

    Теперь гоните два сферических шара одинакового радиуса, но разной массы, скажем, один из стали, другой из дерева. Что победит?

    Как обычно в этих головоломках, вы можете предположить идеальные материалы, незначительное трение и качение без скольжения (несмотря на отсутствие трения!).Эти головоломки можно решить без явного использования математики. Галилей мог их решить. Возможно, Архимед мог.

    Схватка.
  6. Забитая снасть. . Время от времени изобретательные мастера пытаются улучшить Архимеда. Вот умная вариация стандартных систем шкивов. Рассчитайте его механическое преимущество, принимая во внимание отсутствие трения, безмассовые шкивы и идеально гибкий канат с пренебрежимо малой массой.
  7. Мыльница Дерби. Ребенок строит безмоторный гоночный автомобиль для скоростного спуска. У него есть блестящая идея использовать вместо четырех колес только три, чтобы уменьшить трение в машине. Повысит ли эта модификация характеристики машины в скоростном спуске? Почему?
  8. Очень честная гонка . Несколько лет назад кому-то пришла в голову яркая идея провести гонку на воздушной подушке. Корабли на воздушной подушке поддерживаются над землей большим вентилятором, который нагнетает воздух вниз.Корабль движется так, как если бы он находился на поверхности с очень низким коэффициентом трения. У этих автомобилей также есть еще один вентилятор для приведения в движение. Это довольно большие машины, поэтому промоутеры спроектировали круговую трассу с наклоном, чтобы машины внутри трассы не имели преимущества. Судя по всему, ипподром так и не был построен, и такие гонки никогда не проводились. Почему?
    Вытаскивание йо-йо.
  9. Йо-йо. Игрушка йо-йо стоит ребром на ровном столе.
    1. Если веревка выходит за ось, что произойдет, если вы потянете веревку параллельно столу? Будет катиться влево или вправо?
    2. Если веревка выходит ниже оси, что произойдет, если вы потянете веревку параллельно столу?
    3. Если натянуть веревку прямо вверх, в какую сторону она катится?
    4. Под каким углом можно натянуть веревку, чтобы она скользила по столу без перекатывания? Коэффициент трения скольжения на границе со столом равен 0.5.

      Свои ответы подкрепите анализом прогнозов.

  10. Ударный. Две одинаковые круглые шайбы опираются на ровный стол без трения. Одна шайба направляется к неподвижной шайбе. Они сталкиваются, и столкновение происходит совершенно упруго. Нитерская шайба вращается до или после столкновения. Докажите, что после столкновения скорости двух шайб находятся под прямым углом, независимо от того, как произошло столкновение.
  11. Левитация. Почему этого не происходит?
    Утренняя левитация.
  12. Горящая свеча с обоих концов.
    Харпо Маркс горит свечу с обоих концов.
    Конские перья (1932).

    Свечу обрезают снизу так, чтобы были видны оба конца фитиля. Гвоздь или длинная игла вставляется в середину свечи и опирается на края двух стаканов. Затем свеча зажигается с обоих концов.Обычно свеча колеблется вокруг оси. Является ли результирующее движение простым гармоническим движением или просто периодическим? Постоянен ли его период?

  13. Парадокс давления. Не трогайте старомодную бутылку негомогенизированного молока. Сливки в молоке поднимаются вверх и занимают узкое горлышко у более узкого верха бутылки. Давление молока на дно бутылки теперь такое же, больше или меньше, чем раньше?

    Вы знаете, что эта загадка старая, потому что эти молочные бутылки сегодня редко можно увидеть.Не распространено и негомогенизированное молоко. Однако многие продукты питания выпускаются в похожих бутылках с узким горлышком. Для современной версии представьте себе взбитую бутылку заправки для салата с маслом и уксусом. Затем масло медленно отделяется и поднимается к узкому горлышку бутылки. Примечание. Многие материалы при смешивании занимают объем, отличный от их общего объема при разделении. Это вообще небольшой эффект. В этой задаче эта разница в громкости будет проигнорирована. Фактически, это будет иметь незначительный вклад в рассматриваемые здесь изменения давления.

  14. Маятник имеет ведро для боба. Он наполовину заполнен водой. Вода замерзает. Что происходит с периодом маятника?
  15. Тепловое давление. Твердый куб стоит на ровной поверхности. Куб сильно нагревается. Увеличивается ли давление куба на поверхность,
    1. ?
    2. остались прежними?
    3. уменьшение?

    Не обращайте внимания на релятивистские эффекты и придерживайтесь классической физики.Объясните свои рассуждения.

  16. У большого корабля, пришвартованного в доке, есть веревочная лестница, свисающая с борта и уходящая в воду. Расстояние между ступенями составляет 30 см, а над водой – 20 ступенек. Скорость прилива составляет 15 см / час. Через 6 часов, сколько ступенек над водой?
  17. Мяч упрямый.

    Гладкий шар упирается в стык пола и наклонной стены. Когда тела находятся в контакте, на границе раздела возникает сила, направленная по нормали к контактной поверхности.Мы показываем силу, создаваемую наклонной стеной (зеленый) в точке B, и силу, создаваемую полом (синий) в точке A. Синий вектор не имеет горизонтальной составляющей, поэтому он не вызывает качения мяча. Но у зеленого вектора есть горизонтальная составляющая. Почему эта сила не заставляет мяч отскакивать от стены?

  18. Простой маятник имеет небольшую массу (B), прикрепленную к веревке с незначительной массой, подвешенной на неподвижной опоре (F). Во время замаха натяжение струны непостоянно.Для маятника с длиной струны L = 30 см , качающегося по дуге θ = 10 ° по обе стороны от вертикали, сколько работы совершает сила натяжения струны, действующая на массу в течение одного периода маятника?
  19. Кузова жесткие. Говорят, что законы Ньютона универсальны, то есть они применимы везде и всегда, по крайней мере, для макроскопических (крупномасштабных) явлений. Почти в каждом учебнике механики есть глава, посвященная твердым телам .Это тела, которые сохраняют свою физическую форму именно во время взаимодействий. Покажите, что совершенно твердые тела не могут существовать, поскольку они нарушают законы Ньютона.
  20. Весомые дела. В учебниках вес тела часто определяется как сила тяжести, действующая на тело на поверхности земли. Но позже они говорят о ситуациях, когда тело полностью или частично погружено в жидкость, и говорят о «потере веса» тела, погруженного в жидкость. Затем при обсуждении орбитальных пилотируемых спутников Земли говорят о «невесомых космонавтах».Говорят, что физика – «точная» наука, но кажется, что язык, используемый в учебниках, далек от точного. Разрешите эту дилемму.
  21. Лох Леонардо 1. В записных книжках Леонардо да Винчи есть ряд ошибок.
    Источник: Леонардо да Винчи, Кодекс Арундела, лист 1030, чертеж № 68: Лондон, Британский музей

    На этом рисунке показан его военный танк с приводом от шестерен и кривошипов, как показано слева.Два человека внутри крутили кривошипы, приводящие в движение колеса. Зубчатая передача – обычная «фонарная передача» того времени. Игнорируя тривиальное наблюдение, что для этого потребуются два очень сильных человека , почему бы это не сработать? Нет никаких записей о том, что он когда-либо строился и использовался.

  22. Болван Ленардо 2. В записных книжках Ленардо да Винчи есть ряд ошибок. Вот один, показывающий потоки воды из отверстий на разной высоте в резервуаре для воды.

    Что не так с этой схемой. Как это должно выглядеть?

    Винт антенны Леонардо. Codex Atlanticus.
  23. Лох Леонардо 3. Леонардо да Винчи предложил несколько идей для летательных аппаратов с двигателем. Один из них, названный «воздушный винт», имел вращающийся аэродинамический профиль в форме винта, приводимый в движение двумя людьми на платформе внизу и вращающими рукоятки. Помимо тривиального наблюдения, что даже двое мужчин не могут обеспечить достаточно энергии, эта идея имеет серьезный физический недостаток, который не позволяет ей оставаться в воздухе.Что это? Очевидно, эта идея не взлетела.
  24. В учебниках часто говорится, что когда объект находится в фокальной плоскости собирающей линзы, свет от него, проходя через линзу, формирует реальное изображение «на бесконечности». Однако с равным успехом можно сказать, что он также формирует виртуальное изображение «на минус бесконечности», которое легко увидеть, глядя через линзу на источник света. Итак, один объектив дает два изображения. Как это может быть?

    Не играем ли мы здесь со словом «бесконечность» слишком быстро? В некоторых курсах математики учителя говорили: «параллельные прямые пересекаются в бесконечности».Кажется, более небрежный язык. Разрешите эту путаницу.

    Возникает еще один вопрос. Любой объектив может создавать реальные или виртуальные изображения, в зависимости от местоположения объекта, и задается уравнением линзы 1 / p + 1 / q = 1 / f . Но это все? Создает ли объектив какие-либо другие изображения?

  25. Изображение, которое вы видите, когда смотрите в зеркало, оказывается перевернутым влево / вправо, но не вверх / вниз. Если вы правша, ваше зеркальное отображение будет левым.Если вы дотронетесь до правого уха, изображение коснется его левого уха. Но ваш имидж не стоит с ног на голову. На первый взгляд это кажется парадоксальным, поскольку зеркало симметрично относительно нормали. Вы можете вращать зеркало вокруг его нормальной оси, и изображение не поворачивается. Так почему же изображение не симметрично относительно нормали? Разрешите эту путаницу простым аргументом. Вы должны быть осторожны и точны в использовании языка.

  26. Виртуальный поворот изображения. У призмы Dove есть интересное свойство: когда вы смотрите сквозь нее и поворачиваете ее, изображение поворачивается на угол, вдвое больший, чем была повернута призма. Объяснять.

    Если у вас нет такой призмы, используйте равностороннюю призму, глядя сквозь нее, как показано, чтобы свет имел внутреннее отражение с одной стороны призмы.

  27. Перископ вверх. Подводные лодки сыграли важную роль во Второй мировой войне. Вы видели те фильмы, в которых капитан ищет вражеские корабли через перископ – длинную узкую трубку, уходящую вверх, чуть выше поверхности воды.Это были дни до появления телевидения и волоконной оптики, поэтому в перископе использовались только линзы и отражающие призмы. Вы знаете, что, глядя через длинную узкую трубу, вы не можете видеть больше, чем очень узкое поле зрения, но перископы могут видеть гораздо большее поле. Эти перископы могут быть 30 футов в длину и шесть дюймов в диаметре. Просматривая такую ​​трубку, вы увидите поле всего в один градус. Тем не менее, перизокпес обычно имел поле 9 ° и более. Как это можно сделать, используя только оптическую систему со стеклянными линзами?
  28. Физика падения. В каждом вводном учебнике физики говорится, что при отсутствии сопротивления воздуха два тела разной массы падают с одинаковым ускорением, то есть они будут падать на равные расстояния за одинаковое время. Галилей обычно упоминается в этом контексте, хотя другие проводили эксперимент до него, и он, вероятно, никогда не проводил эксперимент со свободно падающими телами (конечно, не в Пизанской башне). Но у Галилея был простой логический аргумент, чтобы заключить, что масса падающего тела не имеет значения.Помните, что во времена Галилея алгебра еще не была изобретена, а исчисление появилось еще позже. Позже возникла концепция гравитации и закон Ньютона F = м a . Так как же Галилей пришел к такому важному результату, используя только простой логический аргумент?
    Взвешивание подвижной системы
    .
  29. Снижение веса? Нам часто говорят, что если мы будем продолжать двигаться, мы похудеем.Но зависит ли вес движущегося объекта от его движения? Классический лабораторный эксперимент по физике – это машина Этвуда: две неравные массы на конце струны, проходящей через шкив. Систему можно заставить ускоряться достаточно медленно, чтобы легко измерить ее ускорение, и с небольшой математикой определить значение ускорения свободного падения. Показанная машина Этвуда подвешена на пружинных весах. Масса на одной подвеске M , на другой – (M + m) . Предположим, что более тяжелая сторона (правая сторона) подвес крепится к крючку пружинных весов дополнительной резьбой, предотвращающей перемещение масс.Шкала показывает (2M + m) .

    Ограничивающая нить сжигается или обрезается, и система приводится в движение, левая сторона поднимается, а более тяжелая правая сторона опускается. Пока массы находятся в движении, пружинные весы показывают

    1. то же, что и раньше.
    2. больше, чем раньше.
    3. меньше, чем раньше.
    4. ноль.

    Объяснить, почему.

  30. Обсуждая кинетическую теорию, учебники часто моделируют идеальный газ как коробку с бесконечно массивными стенками, содержащую очень крошечные частицы, отскакивающие от стен.Часть аргумента касается одной такой частицы, отскакивающей от стены. Нам говорят, что столкновение является совершенно упругим, и частица отскакивает от стены с той же скоростью, что и до удара о стену. Это говорит нам о том, что мяч отскакивает с неизменной кинетической энергией, что студенты слишком охотно принимают некритически. Мы разумно делаем вывод, что в стене не было потерь энергии. Но как насчет импульса? Частица имела импульс mv до столкновения и импульс −mv после столкновения, поскольку импульс является вектором.Итак, есть изменение импульса частицы на −2mv , и из-за сохранения импульса должно было произойти изменение на & плюс; 2mv изменение импульса стенки.

    Так как же стена может набирать обороты, не получая при этом энергии? Учебники снова обманывают нас? Решите эту проблему с помощью расчета энергии и импульса.

  31. Определения эластичности. Учебники говорят нам, что идеально эластичное тело – это тело, которое при деформации возвращается к своей первоначальной форме без потери энергии.Они также говорят нам, что идеально упругое столкновение – это такое столкновение, в котором участвующие тела сохраняют как кинетическую энергию, так и импульс.

    Но рассмотрим колокол, сделанный из латуни, с латунной тарелкой. Колокола и их колокола сделаны из почти эластичных металлов, и оба сохраняют свою форму после многих столкновений. Совершенно упругое столкновение – это такое столкновение, при котором сохраняется механическая энергия без потерь на диссипативные процессы. Столкновение хлопушки и колокола не является абсолютно упругим, поскольку значительная часть энергии теряется в виде звука, исходящего от колокола.К тому же качающийся колокол и хлопушка вскоре останавливаются, так что вы знаете, что их энергия каким-то образом рассеивалась. Так как же упругие тела могут подвергаться неупругим столкновениям? Разрешите это кажущееся противоречие.

    Пустой вопрос: издаст ли звук колокол и колотушка, изготовленные из идеально эластичных материалов ?

  32. Эквивалентность? Учебные трактовки теории относительности иногда иллюстрируют «принцип эквивалентности» на примере человека в лифте.Трос лифта обрывается, и несчастный пассажир падает вместе с лифтом, испытывая «невесомость», в которой он свободно плавает в системе отсчета лифта, как если бы не действовали никакие внешние силы. В учебниках часто говорится, что человек внутри не сможет ни одним экспериментом определить, что в его лифте есть гравитационное поле. Этот пример, конечно, ошибочен, поскольку с помощью чувствительных инструментов человек в лифте может обнаружить гравитационное поле.Как?

  33. Эллипс или Парабола? В учебниках физики много места посвящено обсуждению траекторий полета снарядов в гравитационном поле Земли. Они подчиняются закону d = v o t + & half; г т 2 , что является уравнением параболы. Но Ньютон говорит нам, что путь пушечного ядра (при отсутствии сопротивления воздуха) – это часть эллипса с центром Земли в одном фокусе.Знаменитая картина «Гора Ньютона» иллюстрирует это.

    Итак, если бы вас спросили: «Каков путь снаряда, эллипса или параболы?», Какой ответ вы бы дали? Разрешите это кажущееся противоречие.

  34. Третий закон Ньютона гласит: Если тело A оказывает силу на тело B, то тело B оказывает равную и противоположно направленную силу на A. Другие законы Ньютона были бы бесполезны без этого важного закона. Говорят, что законы Ньютона универсальны и применяются везде и всегда.Но третий закон Ньютона не может быть верным во всех случаях, даже в классической физике. Покажите, почему, на простом примере.
  35. Закон всемирного тяготения Ньютона , F = GMm / R 2 без вопросов принимается первокурсником. Но небольшая мысль показывает, что это не может быть правдой во всех случаях. Когда R = 0, сила становится бесконечной, что является нефизическим результатом. Обоснуйте, почему это не является серьезной проблемой.

  36. Плавающая идея. Стакан с водой стоит на весах, используемых для измерения его веса. Шар, менее плотный, чем вода, обычно плавает по воде. Но он полностью погружен в воду с помощью веревки, прикрепленной ко дну стакана. Мяч окружен водой и не касается стенок стакана. Струна, очевидно, оказывает восходящее усилие на дно стакана. Струна рвется, и мяч поднимается на поверхность, плавая там. Струна больше не оказывает восходящее усилие на стакан.Шкала теперь читается больше, меньше или так же, как раньше? Подтвердите свои рассуждения схемой свободного тела.
  37. Дырочная физика. Проблемы физики часто представляют собой в высшей степени идеализированные ситуации. Вот такая классическая проблема. Если бы прямая дыра была просверлена на всем протяжении земли прямо через центр земли, и камень упал в дыру, сколько времени потребовалось бы, чтобы вернуться?

    Чтобы не усложнять задачу, игнорируйте тот факт, что дыру нельзя было просверлить в раскаленном материале в земле, и если бы это было так, она немедленно заполнилась бы магмой.Затем есть досадное осложнение вращения Земли, поэтому мы должны остановить это, потому что камень столкнется со стенкой дыры. Кстати, какая стена? Одним из способов избежать этой проблемы было бы бурение скважины вдоль оси вращения Земли с севера на юг.

    Чтобы завершить идеализацию, предположим, что плотность Земли однородна.

    И чтобы расширить проблему, после того как вы нашли предыдущий ответ, предположим, что был пробурен прямой туннель из Нью-Йорка в Сан-Франциско.Теперь проложите железнодорожный путь через туннель. Сколько времени займет поездка в вагоне без двигателя без толчка, без учета трения и т. Д.?

    Как обычно, мы ищем простейшее решение, желательно даже не требующее вычислений.

  38. Навсегда – это долго. Может ли идеальный цилиндр при первоначальном толчке катиться на бесконечной плоскости без трения вечно?
  39. Трение – это сопротивление. Студенты иногда предполагают, что трение всегда препятствует движению тела, уменьшая его скорость.Но есть много повседневных примеров, показывающих, что трение может быть необходимо для начала и поддержания движения. Приведите несколько примеров. Сформулируйте определение трения, чтобы его нельзя было неправильно истолковать.
  40. Гоночные фотоны. Рассмотрим свет, проходящий через собирающую линзу от точечного источника к точечному изображению. Световые лучи, проходящие через линзу возле ее края, должны проходить большее расстояние от источника до изображения, чем лучи, проходящие через центр линзы.Разве это не заставит лучи приходить в разное время и, возможно, вызвать деструктивные помехи на изображении? Объяснять.
  41. Расплетение спектра. Сэр Исаак Ньютон (1642-1727) известен своими экспериментами со светом и призмами. Он показал, что свет, проходящий через призму, разделяется (рассеивается) на цветной веер (спектр). Он также показал, что если этот цветной свет затем пропустить через другую призму, правильно расположенную, он может быть рекомбинирован в белый свет.Таким образом, утверждал он, цвета на самом деле в белом свете , а не , созданное призмой . Вот галерея примеров из Интернета, которые должны проиллюстрировать этот эксперимент.

    Учебники и веб-страницы часто иллюстрируют этот эксперимент такими красивыми картинками – и ошибаются! Призма Google рекомбинирует белый свет и просматривает изображения. Большинство изображений будут неправильными по одной или нескольким серьезным причинам. Это наглядный пример того, почему Интернет называют «магистралью дезинформации», поскольку он опасно скомпрометирован выбоинами.Если вы попытаетесь повторить этот эксперимент в лабораторной работе, следуя этим примерам, у вас наверняка ничего не получится. Определите ошибки в каждом из них. Как правильно разложить белый свет на цвета, а затем рекомбинировать его в белый свет? Есть несколько способов.

    Однажды у меня был студент, который хотел получить дополнительный балл по проекту, чтобы поднять свой не впечатляющий средний балл. Я предложил ему пойти в лабораторию и повторить этот эксперимент. Он копировал иллюстрации из учебников и каждый раз терпел неудачу.Он был разочарован. В конце концов я предложил ему выяснить, где находится библиотека колледжа, а затем найти «Оптикс» Ньютона. Там он нашел один способ добиться успеха.

  42. Банка соды. Вот загадка из коллекции Мартина Гарднера. Это старая проблема, но метод все же поучительный.

    Предположим, что у полной цилиндрической банки с газировкой центр тяжести находится в ее геометрическом центре, на полпути вверх и прямо посередине банки.По мере потребления газировки центр тяжести изначально понижается. Однако, когда банка пуста, центр тяжести снова оказывается в центре банки. Следовательно, должна быть точка, в которой центр тяжести находится ниже всего.

    Зная вес пустой банки и ее вес в наполненном состоянии, как определить, какой уровень соды в вертикальной банке переместит центр тяжести в самую низкую точку?

    Чтобы разработать точную задачу, предположим, что пустая банка весит 1.5 унций. Это идеальный цилиндр, и любая асимметрия, вызванная пробиванием отверстий в верхней части, не принимается во внимание. Банка вмещает 12 унций (42 грамма) соды, поэтому ее общий вес в заполненном виде составляет 13,5 унций (382 грамма).

  43. Обратный осмос. Корреспондент из Новой Зеландии присылает нам эту гениальную идею, которую он увидел в декабрьской колонке «Ученый-любитель» в журнале Scientific American. Дадим ему описать:
    Осмос – это процесс, при котором вода течет через полупроницаемую мембрану из менее концентрированного раствора в более концентрированный.Обратный осмос – это когда вода течет через мембрану из сильного раствора в слабый. Конечно, у вас должно быть давление за мембраной, чтобы она текла “в неправильном” направлении. Чтобы пресная вода вытекла из морской воды через мембрану, требуется давление около 20 атмосфер. Это основа опреснительных устройств, используемых на больших судах.

    В любом случае, вы берете очень длинную трубу с полупроницаемой пробкой на конце, заполняете ее пресной водой и опускаете через борт лодки так, чтобы один конец находился на несколько метров над поверхностью, а другой – с Пробка находится на дне в самой глубокой океанской впадине, которую вы можете найти на глубине 12 км или какой бы глубиной ни был океан.На этой глубине напор соленой воды в океане вокруг конца трубы составляет более 20 атмосфер, скажем, 21 атмосферу, поэтому пресная вода вытекает из соленой воды океана в трубу для пресной воды. Пресная вода поднимется примерно на 10 м (21-20 = 1 атмосфера) над поверхностью, что-то вроде артезианской скважины. Возможно, вам придется немного отрегулировать глубину в зависимости от плотности морской воды, но принцип кажется правдоподобным.

    Это устройство не только дает бесконечный поток пресной воды, но и может использоваться для запуска небольшого генератора.

    На рисунке показана трубка в океане, ее верхний конец изогнут, чтобы направлять воду к маленькому водяному колесу W. Вы должны любить предложения вечного двигателя, которые настолько просты, без движущихся частей и обещают решить нашу мировую энергетику. проблемы, а также проблемы с ресурсами пресной воды. То есть, если только мы сможем сразу насытить работой этих машин.

    Давление в океане линейно изменяется с глубиной, увеличиваясь примерно на 1 атмосферу на каждые 10 метров глубины.Таким образом, давление в океане на глубине около 200 метров (700 футов) на 20 атмосфер выше атмосферного. Этот факт может быть полезным, а может и нет.

    Кажется, это отличная идея. Но это не сработает. Почему нет? Ответ дан в апреле. 1972 г. в журнале Scientific American, но это немного косвенно. См. Также июньский выпуск 1971 года.

  44. Какое яйцо варят? Это очень старая проблема. На столе два яйца, одно свежее и одно сваренное вкрутую.Как определить, какая из них вареная, не разбивая их скорлупу?
  45. Какой полый? Две сферы имеют одинаковый диаметр, одинаковый вес и окрашены в один цвет. Один прочный, из легкого материала. Другой – полая оболочка из более плотного материала. Не повредив их, как определить, какая из них полая?
  46. Замечательная головоломка. Эту головоломку часто критикуют за кажущуюся двусмысленность. Вот версия, в которой большая часть двусмысленности удалена.

    Вам даются два стальных стержня, одинаковых, за исключением того, что один стержень намагничен, а другой не намагничен. Не используя ничего, кроме двух стержней и рук, как можно определить, какой из них является магнитом? Мы позволим гравитации действовать как обычно на вас и стержнях ».

    Неосторожно сформулированные версии этой задачи приводят к следующим ответам:

    1. Подвесьте один на нити, привязанной к его центру, и наблюдайте, стремится ли он указывать на север.
    2. Нагрейте один из прутков очень горячим и дайте ему остыть. Если стержни больше не притягиваются так сильно, значит, тот, который вы нагрели, был магнитом.
    3. Несколько раз уроните один на пол. Если притяжение между стержнями уменьшается, значит, вы уронили магнит.
    Но мы исключили это, специально потребовав, чтобы вы использовали только грифы и руки. Нельзя использовать ни веревку, ни проволоку, ни другой металл, ни что-либо для нагрева стержня. Вы даже не можете использовать магнитное поле земли.Так как же проще всего идентифицировать намагниченный стержень?

    Один хорошо известный ответ – это «Т-тест». Поместите стержни, соприкасающиеся в Т-образной конфигурации, так, чтобы конец одного находился в центре другого. Если они притягиваются, то тот, который стоит вертикально от буквы Т, является магнитом, поскольку у другого есть полюса на обоих концах, а в центре нет полюса.

    Но магниты из материалов с высокой проницаемостью могут быть изготовлены с множеством полюсов, например, с расположением одного полюса [N SS N].Такой магнит , а не будет направлен на север в подвешенном состоянии и может не пройти тест “Т”. Какой самый простой способ идентифицировать магнит, независимо от того, как расположены полюса этого магнита?

  47. Что длиннее? Подготовьте две металлические трубки. Шахты вырезаны из алюминиевых трубок диаметром 1 дюйм, которые продаются в строительном магазине. Одна трубка имеет длину 11 дюймов. Другой на 1/4 дюйма короче. Постарайтесь убедиться, что на трубках нет царапин или дефектов, которые можно отличить друг от друга.

    Возьмите их по одному в каждую руку и спросите, может ли кто-нибудь визуально увидеть, что один из них короче другого. Конечно, никто не может. Держите их рядом, соприкасаясь, и разница очевидна. Попросите кого-нибудь взять их, затем развернитесь, чтобы скрыть их от вашего взгляда, выберите один и затем верните его вам. Вы делаете вид, что оцениваете ее длину между руками, касаясь концов трубки только кончиками пальцев. Отложите его и попросите другого, сделав то же самое, затем объявите: «Этот (короче / длиннее, в зависимости от случая).”

    Вы могли бы сделать это с завязанными глазами, но это, вероятно, переборщить с демонстрацией физики. В чем твой секрет?

  48. Парадокс качения. В учебниках физики сила трения определяется как сила, касательная к двум поверхностям в точке их соприкосновения. Представьте, что шарик или цилиндр катятся без скольжения по идеально ровной и ровной поверхности. Мы ожидаем, что он замедлится. Мы наивно предполагаем, что трение – это причина того, что он замедляется и в конечном итоге останавливается.Конечно, трение противоположно скорости мяча и, следовательно, замедлит движение мяча по второму закону Ньютона. Но эта сила из-за трения имеет крутящий момент, и этот векторный крутящий момент вокруг центра масс мяча находится в том же направлении, что и вектор угловой скорости мяча. Это на увеличит угловую скорость мяча на , заставив его катиться все быстрее и быстрее. Разрешите это кажущееся противоречие.

    Когда изобретатели впервые предложили железнодорожный транспорт со стальными колесами на стальных рельсах, некоторые скептики сказали: «Колеса просто будут крутиться на месте, и устройство никуда не денется.”Может быть, они думали об этом парадоксе.

  49. Принцип бутстрапа. В небылицах барона Мюнхгаузена есть история его спасения из неприятной ситуации, когда он увяз в болоте. Находчивый барон наклонился и приподнялся, натянув ремни ботинок. Мы знаем, что это невозможно, но может ли человек, используя физику и систему шкивов, поднять себя, используя только свою силу?

    Рассмотрим показанную систему.Используется легкий стул с подвесным блоком. Это может сработать? Есть ли ограничения в этой системе? Покажите векторный анализ со схемами свободного тела.

  50. В состоянии покоя . Редко бывает книга по физике, в главах, посвященных статике, не говорится что-то вроде «Чистая сила, действующая на покоящееся тело, равна нулю». Кроме того, в нем говорится, что если результирующая сила равна нулю, ускорение тела равно нулю. Затем, в главах, посвященных динамике, мы можем увидеть: «Тело, брошенное вверх, на мгновение покоится в высшей точке своей траектории».Затем ученик логически заключает, что в этой точке результирующая сила, действующая на тело, равна нулю (по крайней мере, на мгновение), и, следовательно, его ускорение в этой точке равно нулю. Это ошибка «в состоянии покоя → ​​нулевая чистая сила → равновесие → нулевое ускорение». Можно ли обвинять студентов в том, что они ловят учебники на слове?

    Сможете ли вы разрешить это кажущееся противоречие?

  51. Потери энергии?
    Парадокс конденсатора.

    Этот парадокс конденсатора обсуждался в Интернете и в опубликованных статьях, но люди до сих пор спорят по этому поводу.

    Получите два одинаковых конденсатора. Зарядите одного из них. Затем соедините их вместе, чтобы заряд распределялся поровну на обоих. Простой расчет показывает, что энергия двух заряженных конденсаторов после этой операции составляет только половину энергии одного первоначально заряженного конденсатора. Что случилось с потерянной энергией?

    Конечно, сразу подозревается потеря энергии из-за нагрева соединительных проводов.Поэтому мы идеализируем проблему и используем безопорные соединительные провода. Тем не менее, мы должны учитывать энергию, излучаемую ускоряющими зарядами во время начального процесса включения переключателей и последующего ускорения электронов во время перераспределения заряда. Однако опубликованные статьи спорят о деталях этих процессов.

    Так что же происходит? Неправильны ли теория цепей и классическая теория электромагнитного поля? Можете ли вы решить это просто?

  52. Соломинки для захвата.

    1. Мы все проделали эту демонстрацию, используя трубочку для питья и стакан воды. Вставьте соломинку в воду (A), закройте верхнюю часть соломинки пальцем, затем поднимите соломинку, удерживая верхнюю часть закрытой. Это поднимает столб воды внутри соломинки (B), несмотря на открытый конец. Какая физика демонстрируется?

    2. Обычно мы не изучаем детали этой простой демонстрации, но как насчет нижнего конца соломинки? Там есть поверхность воды, открытая для воздуха.Какая у него форма?

    1. Выдвигается вниз.
    2. Вздымается вверх.
    3. Почти плоский.
    Подтвердите свое предположение веским физическим аргументом.

    3. А теперь сделаем поинтереснее. Проделайте отверстие в трубочке для питья примерно в двух дюймах от дна. Сделайте отверстие размером с концевые отверстия соломинки. Теперь погрузите соломинку в стакан с водой. Боковое отверстие должно быть ниже уровня воды. Теперь закройте пальцем верхний конец соломинки.Поднимите соломинку, пока она полностью не выйдет из воды (С). Что, по вашему мнению, произойдет? Подтвердите свой ответ аргументом, основанным на законах физики. Конкретно обсудите, что происходит у бокового отверстия. А теперь попробуй.

  53. Скользкий спуск. Если вы спускаетесь по скользкому склону в автомобиле, сохраните ли вы лучший контроль над рулем, если ваши передние или задние колеса заблокируются?
  54. Мощные магниты? Часто можно услышать сильные магниты, которые называют «мощными».Но являются ли они источником силы? Я часто слышу, как люди утверждают, что магниты должны быть неиссякаемым источником энергии. Они ссылаются на скромный магнит на холодильник, говоря: «Он поддерживает свой собственный вес на стенке холодильника навсегда или, по крайней мере, в течение многих лет. Так что магниты должны быть источником значительной энергии». Я часто слышу это от людей, которые думают, что могут изобрести вечное движение, разместив магниты во вращающемся механизме для извлечения накопленной энергии.

    Что не так в их аргументе?

  55. Повышение силы тяжести. Генри Кавендиш (17311810) измерил силу гравитационного притяжения между двумя свинцовыми шарами в лабораторных условиях. Он использовал чувствительную торсионную подвеску для измерения такой небольшой силы. Предположим, у нас есть жидкость в U-образной трубке в состоянии равновесия, а затем поместите тяжелый свинцовый шар (красный) прямо под левую сторону трубки. Как это повлияет на уровень жидкости в трубке?
  56. Отрицательная реакция? Обычно, когда мы тянем за что-то, оно движется к нам в направлении приложенной силы (если только не прибито гвоздями).Можете ли вы придумать или придумать простую систему, которая отодвигает от вас на , когда вы пытаетесь подтянуть ее к себе?
  57. Маятник Фуко.
    Маятник Фуко на Пантоне в Париже.

    Леон Фуко (1819–1868) установил большой маятник в 1851 году на Пантеоне в Париже, чтобы продемонстрировать вращение Земли. Его длина составляла 220 футов, а вес – 62 фунта. При установке качания он медленно прецессировал, потому что он сохранял свою первоначальную плоскость качания, пока земля вращалась под ним.Это было легко наблюдать в течение дня, так как его плоскость качания изменялась по отношению к полу под ним. Такие маятники есть в научных музеях по всему миру, и в некоторых университетских зданиях физики тоже.

    Но почему маятник продолжает движение в исходной плоскости? В конце концов, его подвесной трос прикреплен вверху, и, несомненно, вращение здания вызовет крутящий момент на трос. Разве это не приведет к тому, что маятник будет двигаться вслед за движением здания, в котором он находится? Требуются некоторые пояснения.

    Тогда есть вопрос о начальных условиях. Когда маятник оттягивается утром и отпускается, этот процесс выполняется в уже вращающейся системе отсчета – в самом здании. Разве это начальное движение не должно приводить к смещению маятника, чтобы сохранить это движение до конца дня, чтобы его плоскость движения вообще не изменилась по отношению к зданию? Следовательно, явной прецессии не будет.

    Когда я был студентом университета, мне однажды дали хороший совет по физике.”Вы понимаете все, что знаете об этом?” Эти простые вопросы, поставленные маятником Фуко, мучили меня за много лет до того, как я нашел ответы. Учебники и профессора избегают этого, редко задавая такие вопросы.

  58. Хожу по кругу. Человечество, иногда называемое «болезнью ползания по лицу земли», поражает землю разными способами. Но об одном эффекте человеческой деятельности упоминается редко. В большинстве стран автомобили едут по правой стороне дороги.Круги проезда проезжают против часовой стрелки. Большинство автомобилей и грузовиков возвращаются домой после поездки, поэтому они двигаются против часовой стрелки. В США карнавальные карусели (карусели) также вращаются против часовой стрелки, а гонки – люди, лошади, собаки и автомобили – проходят против часовой стрелки. Единственным исключением является Великобритания (и несколько других стран), где все это движется по часовой стрелке, включая автомобильное движение и перекрестки с круговым движением.

    Меняет ли это вращательное движение земной поверхности скорость вращения Земли, хотя бы чуть-чуть? Может ли это ускорить или замедлить вращение Земли? Стоит ли нам беспокоиться? И каков эффект от всех тех спутников Земли, которые мы вывели на орбиту, большинство из которых направлено на восток?

    Иллюстрирует центростремительную силу.
  59. Круговой аргумент. Шарик на конце веревки. Удерживая другой конец веревки, вы раскачиваете мяч по большому кругу. Учебники часто представляют это как проблему, прося вас связать угловую скорость мяча с натяжением струны, используя известную формулу для центростремительной силы: F = mω 2 R . Но действительно ли напряжение равно центростремительной силе?

    Из-за сопротивления воздуха мяч замедлится.Чтобы это продолжалось, что-то еще должно давать энергию в виде работы. Но если струна радиальная и мяч движется по касательной к его круговой траектории, сила и смещение перпендикулярны друг другу. Так как же веревка может работать с мячом, чтобы поддерживать его движение?

  60. Маятник недоумения. Каждый учебник физики говорит нам, что период простого маятника не зависит от массы боба. Но в этих книгах редко задается вопрос: «Почему период не зависит от массы?» Если вы проследите за выводом формулы периода, вы увидите, что масса выпадает из расчета.Но есть простой и проницательный способ доказать это, даже не занимаясь математикой. Не могли бы вы?

  61. Опирающийся мяч. Равномерная сфера массой м и радиусом r висит на веревке у гладкой вертикальной стены, линия струны проходит через центр шара. Трос прикрепляется на высоте h = √ (3r) над точкой касания мяча стены. Каково натяжение струны T и сила F , прилагаемая мячом к стене? Если шар шероховатый, с коэффициентом трения покоя μ s , как эти силы увеличиваются или уменьшаются?
  62. Действие и реакция. Учебники часто говорят нам, что закон Ньютона выглядит примерно так: «На всякое действие есть равное и противоположное противодействие». Конечно, это написано небрежно. Как любые две вещи могут быть равными и противоположными? Следует сказать: «На каждое действие есть противодействие равного размера и противоположного направления».

    Но каково определение «действия»? Кто-то может возразить, что «реакция» – это отрицательное «действие». Если да, то исходное утверждение может быть правильным, но оно все равно сбивает с толку. Серьезно, когда у вас есть пара действие / противодействие, как вы можете определить, какое действие, а какое – противодействие?

  63. Ставить телегу впереди лошади. К телеге прицепляют лошадь. Лошадь прикладывает силу F вперед к телеге, а телега оказывает такое же усилие назад на лошадь в соответствии с третьим законом Ньютона. Так что лошадь с телегой никуда не пойдут.

    В чем недостаток этого аргумента?

  64. Лунный аттракцион. Стоя на Земле, вы ближе к Солнцу в полдень во время новолуния или в полдень на полмесяца позже во время полнолуния? Почему?
  65. По кругу .Модель Солнечной системы Птолемея была геоцентрической (с центром на Земле) и полностью основывалась на кругах (которые считались идеальной фигурой). Чтобы соответствовать наблюдениям за положением планет, он стал чрезвычайно геометрически сложным, с кругами (циклами) и меньшими кругами (эпициллы), выступами и эквантами и другими уловками, чтобы согласовать его с наблюдениями.
    Упрощенная система Птолемея. Не в масштабе.
    Адаптировано из книги Ван Аллена, Джеймса А. 924 Элементарные задачи
    и ответы в астрономии солнечной системы.
    U. of Iowa Press, 1993.

    Коперник попытался упростить это, используя гелиоцентрическую (центрированную по Солнцу) модель. Но он по-прежнему настаивал на геометрии, основанной на кругах. Его системе по-прежнему нужны эпицилы, но, как он утверждал, их меньше. Менее важным, чем количество эпициклов, является свойство конкретных эпициклов, которые его система исключила. В системе Птолемея шесть заброшенных циклов и эпициклов имели одну важную общую черту.Что это было?

  66. Постоянная ошибка. Бесконечно растягивающаяся резинка соединяет дерево с задним бампером автомобиля. По мере того как автомобиль уезжает с постоянной скоростью, лента растягивается. Жук на ремешке медленно ползет к машине. Может ли ошибка когда-нибудь добраться до авто, если будет достаточно времени?
  67. Дырявый шар.
    Дырявый шар. L = 6 дюймов.
    Из мира математики.wolfram.com.

    Просматривая книги Мартина Гарднера, я наткнулся на эту дьявольскую загадку. Гарднер называет это «невероятной проблемой». Он также проследил это до Сэмюэля И. Джонса « Mathematical Nuts », 1932, стр. 86.

    Его можно увидеть в Интернете в различных формах, часто с двусмысленными формулировками, а также с бесконечными обсуждениями, которые часто ни к чему не приводят. Я попытался переформулировать это, чтобы устранить двусмысленность (что непросто).

    Отверстие полностью просверливается в сфере, прямо через центр сферы и центрируется в ней.Отверстие в сфере представляет собой цилиндр длиной 6 дюймов. Каков объем оставшейся части сферы (не включая просверленный материал).

    Можно подумать, что предоставлено недостаточно информации. Но есть. Решение не требует исчисления. Гарднер дает проницательное решение, которое требует всего двух предложений, включая только одно уравнение.

  68. Сплющенная земля . Из-за вращения Земля не имеет сферической формы. Это сплюснутый сфероид, выпуклый на экваторе.Радиус его кривизны больше на экваторе или на полюсах?
  69. Цепочка резисторов.
    Цепочка резисторов.

    Каждый резистор в этой цепи имеет сопротивление 1 Ом. К клеммам A и B подключается источник питания. Ток в двух крайних правых резисторах составляет 1 ампер. Какова разность потенциалов на входных клеммах A и B этой цепи? Какое сопротивление всей цепи измерено в точках A и B? Какой ток подает на эту схему источник питания?

    Это простая, но утомительная задача, поскольку в цепи всего четыре «звена».Это не достойно ярлыка «пазл». Но что, если бы в цепи было 500 звеньев? Дальнейшее удлинение цепочки не имеет практического смысла, но позволяет решить ее для бесконечного числа звеньев, поскольку по мере того, как вы разрабатываете ее, возникает удивительный паттерн.

    Подсказка 1. Иногда помогает решить головоломку, если подойти к ней с другого конца.
    Подсказка 2: Иногда это не так.
    Подсказка 3. Как это может иметь отношение к Фибоначчи?

  70. Снятие шкуры с контактной сети. Силовой кабель натянут между двумя опорами электросети. Конечно, он провисает в форме кривой, называемой цепной линией. На каждом конец кабеля, на котором он прикреплен к столбу, должен быть под углом 10 ° к горизонтали. Вес этой секции кабеля W. Каково натяжение кабеля в его самой нижней точке? Каково его напряжение на каждом из полюсов?
  71. Поиск центра.
    Рис. 1. Найдите центр масс
    .

    На рис. 1 показан плоский металлический лист L-образной формы одинаковой толщины и состава. Сможете ли вы найти его центр масс, используя только линейку без опознавательных знаков?

  72. Падающая обтяжка. Подвесьте пружину Slinky ™ за один конец. Если отпустить этот конец, как будет весна?
    1. Вся пружина опускается, сохраняя свою растянутую длину до тех пор, пока нижний конец не коснется пола, затем остальная часть пружины опускается, сжимаясь при движении.
    2. Вся пружина падает, сжимаясь при движении.
    3. Нижний конец поднимается, чтобы встретиться с верхним концом, затем пружина опускается в сжатом состоянии.
    4. Нижний конец сохраняет свое положение до тех пор, пока остальная часть пружины не сожмется, затем пружина перейдет в сжатое состояние.

    Последующий вопрос: каково начальное ускорение верхнего конца пружины при падении?

    1. Ускорение свободного падения, г
    2. Ускорение более g .
    3. Ускорение менее g .

    И еще один вопрос: если бы к нижней части подвешенного ремня был прикреплен груз, как это повлияло бы на наши предыдущие ответы?

    О, еще одна вещь: если бы жесткость пружины или скорость импульса сжатия в пружине были другими, мог бы нижний конец ненадолго подняться сразу после того, как верхний конец пружины будет отпущен?

    Как всегда, объясните свои ответы.

  73. Snap! . Все мы знаем, как «щелкнуть» пальцами, что легче сделать, чем описать словами. Сильно сожмите большой и средний пальцы вместе, позволяя пальцу внезапно соскользнуть с большого пальца, и вы услышите щелкающий звук. Не делая этого, объясните, откуда именно исходит звук.
    Перетягивание каната.
  74. Буксир войны. Два одинаковых груза, W, расположены, как показано на рисунке. Старомодные пружинные весы подключаются в середине горизонтального шнура и поддерживаются таким образом, чтобы не провисать шнуры.(Возможно, используйте невесомые пружинные весы.) Каковы приблизительные показания пружинных весов?
    1. 3 Вт
    2. 2 Вт
    3. Вт
    4. ноль
    5. Вт / 2
  75. 15 головоломок магических квадратов.
    Деревянный пазл 15.

    Классическую «15 головоломку» до сих пор можно найти в магазинах игрушек. Он состоит из квадратных плиток, пронумерованных от 1 до 15, в корпусе размером 4 × 4 плитки.При заполнении остается одно пустое место, позволяющее перемещать плитки в разном порядке. Обычно каждый пытается расположить плитки в числовом порядке влево-вправо, сдвигая их, никогда не вынимая из коробки.

    Мастер головоломок Сэм Лойд утверждал, что изобрел эту игрушку в 1891 году, но идея пришла не ему первому. Нойес Чепмен подал на него патент в марте 1880 года. Лойд описал шутку, с которой можно поиграть: просто поменяйте местами две плитки, чтобы ее нельзя было решить в числовом порядке слева направо.Он назвал это загадкой 14-15, потому что он поменял эти две плитки местами, но замена любых двух плиток дала бы тот же результат. Загадку все еще можно было решить хитрыми методами.

     1 5 9 13
     2 6 10 14
     3 7 11 15
     4 8 12
     
    Или переверните пазл боком:
     4 8 12
     3 7 11 15
     2 6 10 14
     1 5 9 13
     
    Но возможностей для головоломок больше. Можете ли вы перетасовать плитки стандартной головоломки с 15 числами, чтобы получился «магический квадрат», в котором сумма плиток в каждой строке, столбце и диагонали равна 30?
  76. Быстрое затмение. Если смотреть сверху на северный полюс Земли, Земля вращается вокруг Солнца против часовой стрелки. Луна вращается вокруг Земли против часовой стрелки. Земля вращается вокруг своей оси против часовой стрелки. Тогда почему область полного солнечного затмения перемещается по Земле с запада на восток? Например, во время солнечного затмения в США в 2017 году область совокупности прибыла на Западное побережье в Орегоне примерно через 1,5 часа после того, как была замечена на Восточном побережье в Чарльстоне, Южная Каролина. Проверьте свой ответ, рассчитав время, которое потребовалось области совокупности, чтобы пересечь U.S.A.
    Скольжение или скольжение?
  77. Скольжение или скольжение? Представьте, что с помощью нового процесса можно производить твердые материалы без трения. Сплошной цилиндр помещается наверху наклонной плоскости, причем оба они сделаны из этого материала. Цилиндр отпускают, стараясь не толкать его или не вращать. Будет ли цилиндр катиться по плоскости без проскальзывания или по плоскости, не вращаясь? Или он будет и скользить, и скользить?
  78. Сезонная головоломка. В Северном полушарии лето теплее зимы. Почему? Обычный поверхностный ответ: «Потому что земная ось имеет фиксированное направление в пространстве, и летом она наклоняется к Солнцу, а зимой – от Солнца». Это правда, но это не полный ответ. Не упоминаются два важных процесса. Вы можете объяснить, почему наклон влияет на сезонные температуры?
  79. Любопытный аттракцион. Эти визитки с магнитной оборотной стороной представляют собой интересную головоломку.Их черные спины имеют чередующиеся полосы северных и южных магнитных полюсов, расположенных на расстоянии 1 мм друг от друга. Вы можете проверить это с двумя одинаковыми картами. Поместите их лицевой стороной вверх, сложите, переместите их по длине, и они будут двигаться плавно. Переместите их перпендикулярно их длине, и движение “подпрыгнет” при переходе от N к S намагниченных полос. Поверните их так, чтобы их черные стороны соприкасались, наблюдаются аналогичные результаты. Но … Поверните одну по отношению к другой (черные поверхности все еще соприкасаются), и при углах от 10 ° до 90 ° движение будет плавным и будет оставаться привлекательным даже с картами под углом 90 °.Почему это должно быть, если при 90 ° должно быть столько же притяжения, сколько отталкивания, и вы ожидаете, что карты не будут притягиваться?

    Эти магниты называются магнитами на молнии. Магнитные элементы в них расположены в виде массива Хальбаха. Вот несколько веб-ссылок, которые могут помочь вам визуализировать их поведение, но они не дают ответа на эту загадку.

    Центр физики. Магниты на молнии.

    EngineerDog.com. Почему магниты на холодильник прилипают только к одной стороне?

    Еще никто не дал ответа.

  80. Армагедон. Если мы обнаружим (достаточно скоро) большой космический камень, направляющийся на встречу с Землей, и мы захотим предотвратить это, что будет лучшей стратегией?
    1. Отправьте большую ракету, чтобы толкнуть камень со скоростью, противоположной ее скорости, чтобы замедлить ее.
    2. Отправьте большую ракету с ядерной боеголовкой, чтобы она приземлилась на скале и поразила ее.
    3. Послать большую ракету с ядерной боеголовкой, чтобы она приземлилась на скале и взорвалась, чтобы сбить скалу с курса.
    4. Поднимите большую ракету, чтобы толкнуть камень в сторону (под прямым углом к ​​скорости камня), чтобы отклонить его курс.

    Почему это сработает? Как его шансы на успех будут зависеть от массы и относительной скорости камня?

  81. Облачная тайна. Мы все видели самые разные облака, тонкие облака, туманные облака и пушистые кумучные облака. Кумулятивные принимают разные формы, но сохраняют индивидуальность, если вы наблюдаете за ними с момента их образования до тех пор, пока они не исчезнут.Почему они сохраняются так долго? Они состоят из капель воды, которые тяжелее воздуха. Почему они просто не растекаются в однородную дымку, покрывающую все небо? Что так долго удерживает облака водяного пара в определенных (хотя и медленно меняющихся) формах? Как они вообще возникают «из воздуха»? Какая у этого физика?

    В поисках ответа вы можете сравнить клубы пара от чайника или старомодного паровоза. Или дым из трубы.

  82. Горячий воздух. Где-то в своем ответе на предыдущую задачу вы, возможно, утверждали, что «поднимается горячий воздух». Это поднимает фундаментальный вопрос: «Почему поднимается горячий воздух?» Упрощенные объяснения из учебников утверждают, что воздух возле пламени расширяется и, следовательно, становится менее плотным, а более легкие предметы поднимаются в более плотных жидкостях. Это действительно объясняет процесс? Почему нагретый воздух ведет себя как объем с меньшей плотностью, чем окружающий воздух?

    Представьте себе горячий предмет, например зажженную спичку или свечу.От него сразу поднимается дым. Но почему молекулы воздуха, нагретые около пламени, просто не двигаются во всех направлениях вверх, вниз и в стороны одинаково? Что заставляет молекулы двигаться преимущественно вверх?

    Конечно, нагретые молекулы в среднем движутся быстрее и вскоре взаимодействуют с другими молекулами, обмениваясь энергией и импульсом. Что в этом процессе побуждает их двигаться вверх? Любое объяснение на молекулярном уровне должно включать гравитацию.


© 2017, 2019 Дональд Э.Симанек.


Вернитесь на главную страницу Дональда Симанека с полным меню,
Вернитесь к головоломкам с ответами. Ответы на головоломки на этой странице.
Вернуться на главную страницу физических головоломок ..

Man Camera Bottle | Ответ на головоломку с подробным объяснением

Головоломка с бутылкой с фотоаппаратом «Человек с камерой» – это вирусная математическая головоломка. В головоломке используются фотографии Человека, Камера и Бутылка, и указана их ценность. Нужно найти значение головоломки 45 24 12.Решая данные уравнения, можно найти ответ на загадку Man Camera Bottle. Головоломка Man Camera Bottle или головоломка 45 24 12 – одна из самых популярных головоломок WhatsApp во время нынешней блокировки. Вот подробное описание ответов на головоломки Man Camera Bottle Puzzle и Man Camera Bottle Puzzle.

Также читают | Салман в роли Халка, Ритик в роли Тора мог бы быть идеальным, если бы у «Мстителей: Финал» была «дези» версия

Пазл Man Camera Bottle

Вот загадка Man Camera Bottle.Посмотрите, сможете ли вы решить эту загадку, прежде чем идти вниз, чтобы найти правильный ответ.

Также читают | Пародия на песню Kiwi Woman “Sound Of Music” на фоне пандемии COVID-19 собрала огромное количество просмотров на YouTube

Ответ на загадку Man Camera Bottle

Точный ответ на загадку Man Camera Bottle – 38

Также читают | Лили Рейнхарт рассказывает о слухах о разрыве с Коулом и осуждает «токсичных» пользователей социальных сетей

Подробное объяснение головоломки Man Camera Bottle или ответ на головоломку 45 24 12

Давайте сначала узнаем ценность камеры из головоломки “Человек с камерой”.Чтобы найти правильное значение камеры, нам нужно разделить 24 на 4, поскольку в уравнении 4 камеры. Таким образом, стоимость одной камеры становится 6.

Также читают | Джеймс Ганн раскрыл секреты фанатов Мстителей как «Стражей Галактики 2»

Чтобы узнать стоимость бутылки,
3 бутылки + 1 бутылка + 2 бутылки = 12
Следовательно, стоимость одной бутылки будет
. 12/6 = 2,

Теперь, чтобы понять ценность Человека, нам нужно внимательно посмотреть на картинку.В первом уравнении трое мужчин с фотоаппаратом в руках и трое с тремя бутылками каждый. У каждого человека в первом уравнении есть 3 бутылки (одна в руке, одна в кармане брюк и одна в кармане рубашки).

Следовательно, стоимость мужчины = {45 – (шампунь 2 x 9) + (камера 6 x 3} ÷ 3
= (45 – 36) ÷ 3 = 3
Таким образом, стоимость человека составляет 3

.

Теперь, наконец, (Бутылка = 2) + (Человек = 3 (без камеры и шампуня) x (2 x камера = 6 × 2 = 12)

Окончательное уравнение принимает следующий вид:
2+ (3 х 12) = 38.

Итак, правильный ответ на загадку Man Camera Bottle – 38.

математических головоломок для печати и головоломок для детей младшего и старшего возраста для повышения IQ и математических навыков.

Бесплатные распечатываемые математические и числовые головоломки подходят для всех детей, родителей, учителей и изучающих математику. Выберите свои любимые математические или логические головоломки:


Объяснение наших математических и числовых головоломок:
Печатные головоломки Проверьте свой IQ и логические навыки и распечатайте наши головоломки на разных уровнях.

Математические кроссворды Наслаждайтесь и осваивайте наши распечатанные математические кроссворды для детей, которые помогут вам освоить или научить базовым математические навыки. Печатные математические кроссворды с числами используются учащимися для отработки и проверки математических навыков, полученных в школе.

Пазлы Судоку для детей У нас есть пазлы судоку для печати в более мелкой сетке 4 на 4 и 6 на 6, которые идеально подходят для начинающих и детей. Начните с более простых головоломок судоку, а затем переходите к нашим распечатываемым головоломкам 9 на 9.

Пазлы для печати Numbrix может улучшить ваши логические навыки и умственные способности. Наши числовые головоломки Numbrix можно распечатать, они созданы как для детей, так и для взрослых, и их можно решить дома или во время урока математики. Соедините все числа на сетке, не отрывая карандаш от бумаги. Сложные логические головоломки для школьников.

KenKen или MathDoku являются идеальными головоломками для печати для изучающих математику, поскольку они сочетают в себе логику и математические навыки. Заполните сетку числами, используя математические операторы в ячейках.Печатные математические головоломки, которые улучшат математические навыки студента.

Математические поиски слов отлично подходят в качестве математических упражнений. Подсчитайте слова, которые нужно найти, и найдите их в поиске слов. Наши математические головоломки для печати входят умножение, сложение, вычитание и другие математические темы.

Алгебра форм (1) – это головоломки с фигурами для печати по математике. студенты с формами. Используйте логические и математические навыки, чтобы определить ценность различных форм.

Алгебра форм (2) математические головоломки фигур.Тренируйте свою логику и числовые навыки и найти ценность различных фигур в коробках с учетом их общего количества.

Пазлы Какуро – это математические головоломки, основанные на сложении. для всех уровней. Пазлы Какуро можно использовать в классе или дома в качестве «домашнего задания». Дети любят разгадывать складывающие головоломки и лучше разбираются в математике, решая их.

Головоломки Prinatble Hidato – это фантастические ускорители мозга и логические головоломки. Пазлы придуманы и созданы доктором Дьёрой Бенедек.Соединяйте числа, как числовая змея, и решайте эти распечатываемые головоломки, предназначенные для детей и начинающих.

Головоломки с числами Фубуки отлично подходят для студентов-математиков. Заполните числа От 1 до 9, так что они складываются в числа подсказки. Практикуйте свои навыки сложения, используя наши распечатываемые математические головоломки для юных учеников.

Головоломки в области Сикаку – это числовые и логические головоломки, в которых вам нужно разделить ячейки сетки, каждая из которых представляет определенную область. Отличные печатные математические и логические головоломки.

Судоку-убийца – это комбинация судоку и Пазлы сложения Какуро. Наши распечатанные головоломки-убийцы-судоку отлично подходят для студентов-математиков.

Волшебные сотни математических головоломок для школьников. Суммируйте каждую строку и столбец до 100, добавляя цифры к числам в ячейках. Математические головоломки Magic Hundred могут быть довольно сложными.

Пазлы со спичками для печати – фантастические возможности для тестирования и повышения вашего логические способности и навыки. Отлично подходит как согреватель или награда для студентов-математиков в классе.

Квадраты сложения для печати – известные математические и складывающие головоломки. которые идеально подходят для студентов-математиков. Математические головоломки и обучение идут рука об руку с этими головоломками с суммированием чисел.

Операционные головоломки CrossMath – это печатные математические головоломки для студенты-математики всех уровней способностей. Заполните сетки, используя указанные операторы. Идеально подходит для уроков математики и репетиторства.

Сетки Теннера – это очень сложные математические сеточные головоломки, основанные на логика и навыки сложения.Начните с более легких уровней и постепенно продвигайтесь вверх.

Матрица операций и математические головоломки для студентов-математиков. Наша печатная Математические головоломки отлично подходят для обучения в математической школе или для репетиторства по математике.

Fillomino – фантастическая логическая головоломка для печати на разные уровни сложности. Пазлы Prinatble Fillomino пришли из Японии и отлично подходят для математических целей.

Более или менее головоломки – это головоломки, похожие на судоку, с более или менее состояние. Прекрасно подходит для студентов-математиков и пригоден для печати.

Building Bridges – это распечатываемые логические головоломки, которые отлично подходят для студенты-математики.

Логические головоломки Masyu Circles, которые можно распечатать, отлично подойдут для развития вашего ума.

Наши математические пазлы для детей веселые и полезные для студентов-математиков!
Решая наши математические числа, логические головоломки и головоломки, учащиеся активно развивают аналитические навыки и решения проблем. навыки и умения. Головоломки – это не только весело, но и полезно для обучения студентов.Задачи по математике можно использовать в качестве разогревающих предметов, домашних заданий или лечебных учебных и учебных материалов. После каждой главы или темы пазлы можно использовать для оценки обучения и способностей учащихся. Таким образом, мы можем использовать математические головоломки как инструменты в обучении учащихся.

Головоломки на этом сайте основаны на следующих математических темах:
– кратные и множители
– четные и нечетные числа
– сложение и вычитание
– числовые комбинации
– умножение
– простые числа
– экспоненты

Are математические головоломки содержат ответы и могут быть очень сложными.Если у вас есть предложения, свяжитесь с нами.

Наши новые материалы? Подписывайтесь на нас.

Сколько из этих 25 головоломок вы сможете решить?

1. СЛОТ ДЛЯ ВОСХОДА НА ПОЛЮС

Днем скользкий ленивец поднимается на шесть футов вверх по опоре, а ночью скатывается вниз на пять футов. Если высота шеста 30 футов, а ленивец стартует с земли (ноль футов), сколько дней требуется ленивцу, чтобы достичь вершины шеста?

Ответ : 25 дней.Математика здесь сводится к чистому приросту на один фут в день вместе с порогом (24 фута в начале дня), который должен быть достигнут, чтобы ленивец мог добраться до 30-футовой отметки в течение данного дня. Через 24 дня и 24 ночи ленивец поднимается на 24 фута. В этот 25-й день ленивец карабкается на шесть футов вверх, достигая 30-футовой вершины шеста. Читателю остается в первую очередь побуждение ленивца совершить этот подвиг. Может, на шесте есть что-нибудь вкусненькое?

(По материалам головоломки Карла Пруджана.)

2. ПИРАТСКАЯ ЗАГАДКА

Группа из пяти пиратов должна разделить свою награду в 100 монет, как описано в видео ниже. Капитан предлагает план распределения, и все пятеро пиратов голосуют «ярр» или «нет» по этому предложению. Если большинство голосов «нет», капитан идет по доске. Пираты располагаются по порядку и голосуют в таком порядке: капитан, Барт, Шарлотта, Даниэль и Элиза. Если большинство голосов «против», и капитан идет по доске, капитанская шляпа переходит к Барту, и процесс повторяется в дальнейшем с серией предложений, голосованием и другим принятием или ходьбой по доске.

Как капитану остаться в живых, получив как можно больше золота? (Другими словами, какое оптимальное количество золота капитан должен предложить каждому пирату, включая его самого, в своем предложении?) Все правила смотрите в видео ниже.

Ответ : Капитан должен предложить ему оставить 98 монет, раздать по одной монете Шарлотте и Элизе и ничего не предлагать Барту и Дэниелу. Барт и Дэниел проголосуют против, но Шарлотта и Элиза посчитали и проголосовали за них, зная, что альтернативный вариант принесет им еще меньше добычи.

3. ДИЛЕММА ПУТЕШЕСТВЕННИКА

Путешественник наткнулся на перекресток, на котором пересекаются три дороги. Он ищет указатель, указывающий направление в город назначения. Он обнаруживает, что столб с тремя названиями городов и стрелками, указывающими на них, упал. Он поднимает его, рассматривает и ставит на место, указывая правильное направление для пункта назначения. Как он это сделал?

Ответ : Он знал, из какого города он только что приехал. Он указал стрелкой обратно на исходную точку, которая правильно сориентировала указатели на пункт назначения и третий город.

(По материалам головоломки Яна Уивера)

4. ЗАГАДКА ПАССОДА

В видео ниже изложены правила этой загадки. Вот отрывок: трое членов команды заключены в тюрьму, и одному предоставляется возможность сбежать, столкнувшись с проблемой. Имея совершенные логические навыки, как могут оставшиеся два члена команды подслушивать, что делает выбранный член команды, и вывести трехзначный код доступа, чтобы вытащить их?

Ответ : пароль 2-2-9 для коридора 13.

5. ПОДСЧЕТ СЧЕТОВ

У меня в кармане была пачка денег. Я отдал половину, а из того, что осталось, я потратил половину. Затем я потерял пять долларов. У меня осталось всего пять баксов. С какой суммы я начал?

Ответ : 40 долларов.

(По материалам головоломки Чарльза Бут-Джонса)

6. ЗАГАДКА ТОПЛИВА САМОЛЕТА

Профессор Фукано планирует совершить кругосветное путешествие на своем новом самолете, как показано на видео ниже.Но в топливном баке самолета не хватает на поездку – на самом деле, его хватает только на половину пути. У Фукано есть два идентичных самолета поддержки, пилотируемых его помощниками Фугори и Ороканой. Самолеты могут перекачивать топливо в воздухе, и все они должны взлетать и приземляться в одном аэропорту на экваторе.

Как эти трое могут сотрудничать и делиться топливом, чтобы Фукано облетел весь мир и никто не разбился? (Смотрите видео для более подробной информации.)

Ответ : Все три самолета вылетели в полдень на запад, полностью загруженные топливом (по 180 килолитров каждый).На 12:45 у каждого самолета осталось 135 км. Орокана дает по 45 киллограмм каждому из двух других самолетов, затем возвращается в аэропорт. В 14:15 Фугори дает профессору еще 45 килограммов и возвращается в аэропорт. В 15:00 Орокана летит на восток , по сути, к профессору по всему миру. Ровно в 16:30 Орокана дает ему 45 кл и переворачивается, теперь летя рядом с профессором. Тем временем Фугори взлетает и направляется к паре. Он встречает их в 17:15 и переводит на каждый самолет по 45 кл.Все три самолета теперь имеют 45 кл и возвращаются в аэропорт.

7. ПРОБЛЕМА HAYSTACK

У фермера есть поле с шестью стогами сена в одном углу, на треть больше в другом углу, вдвое больше в третьем углу и пятью в четвертом углу. Складывая сено в центр поля, фермер позволил ветру разлететь одну из куч по всему полю. Сколько стогов сена осталось у фермера?

Ответ : Всего один.Фермер сложил их все посередине, помнишь?

(По материалам головоломки Яна Уивера)

8. ЗАГАДКА ТРЕХ ЧЕЛОВЕК

В этой видео-загадке вы потерпели крушение и приземлились на планете с тремя инопланетными повелителями по имени Ти, Эфф и Арр. На планете также есть три артефакта, каждый из которых соответствует одному инопланетянину. Чтобы успокоить инопланетян, вам нужно сопоставить артефакты с инопланетянами, но вы не знаете, какой из пришельцев есть какой.

Вы можете задать три вопроса типа «да или нет», каждый из которых адресован любому инопланетянину.Вы можете задать одному и тому же инопланетянину несколько вопросов, но это не обязательно.

Однако ситуация становится более сложной, и эту безумно хитрую загадку лучше всего объяснить (как ее проблему, так и ее решение), посмотрев видео выше.

9. ВОЛЯ ФЕРМЕРА

Однажды фермер решил заняться планированием своего поместья. Он стремился разделить свои поля между тремя дочерьми. У него были дочери-близнецы, а также младшая дочь. Его земля составляла 9 акров. Он хотел, чтобы старшие дочери получили участки земли одинакового размера, а младшая дочь – поменьше.Как он может поделить землю для достижения этой цели?

Ответ : Выше показаны три возможных решения. В каждом из них прямоугольник с меткой 1 представляет собой идеальный квадрат для одного близнеца, а две секции с меткой 2 вместе образуют квадрат того же размера для второго близнеца. Область, отмеченная цифрой 3, представляет собой небольшой идеальный квадрат для самого младшего ребенка.

(По материалам головоломки Яна Уивера)

10. МОНЕТЫ

В руке у меня две американские монеты, которые чеканились на данный момент.Вместе они составляют 55 центов. Один не пятак. Какие монеты?

Ответ : Монета и монета 50 центов. (В последнее время на монете за 50 центов США изображен Джон Ф. Кеннеди.)

(По материалам головоломки Яна Уивера)

11. ЗАГАДКА МОСТА

Студент, лаборант, дворник и старик должны перейти мост, чтобы их не съели зомби, как показано на видео ниже. Студент может перейти мост за одну минуту, лаборант – две минуты, дворник – пять минут, профессор – 10 минут.У группы всего один фонарь, который нужно носить с собой в любое путешествие. Зомби прибывают за 17 минут, а мост может одновременно удерживать только двух человек. Как вы можете перебраться через отведенное время, чтобы перерезать веревочный мост и не дать зомби наступить на мост и / или съесть ваши мозги? (Подробнее смотрите в видео!)

Ответ : Студент и лаборант идут вместе первыми, а студент возвращается, выставляя на часах три минуты.Затем профессор и дворник берут фонарь и вместе пересекают его, занимая 10 минут, выставив общее время на 13 минут. Лаборант хватает фонарь, пересекает крест через две минуты, затем студент и лаборант пересекаются как раз вовремя – всего 17 минут.

12. МАЛЕНЬКАЯ НЭНСИ ЭТТИКОАТ

Вот загадка для детских стихов:

Маленькая Нэнси Эттикот
В белой юбке
С красным носом –
Чем дольше она стоит
Чем короче вырастает

Учитывая эту рифму, что такое «она»?

Ответ : Свеча.

(По материалам головоломки Дж. Майкла Шеннона)

13. Логическая головоломка с зелеными глазами

В этой зеленоглазой логической головоломке есть остров из 100 вполне логичных заключенных с зелеными глазами, но они этого не знают. Они заперты на острове с рождения, никогда не видели зеркала и никогда не обсуждали цвет своих глаз.

На острове зеленоглазым людям разрешено уйти, но только если они пойдут ночью одни к будке охранника, где охранник проверит цвет глаз и либо отпустит человека (зеленые глаза), либо бросит его внутрь. вулкан (не зеленые глаза).Люди не знают своего цвета глаз; они никогда не могут обсудить или узнать свой цвет глаз; они могут уйти только ночью; и им дают только единственную подсказку, когда кто-то со стороны посещает остров. Какая тяжелая жизнь!

Однажды на остров приходит гость. Посетитель говорит заключенным: «По крайней мере, у одного из вас зеленые глаза». На сотое утро после этого все заключенные ушли, все попросили уйти накануне вечером. Как они это выяснили?

Посмотрите видео, чтобы увидеть наглядное объяснение загадки и ее решения.

Ответ : Каждый человек не может быть уверен, зеленые ли у него глаза. Они могут сделать вывод об этом, только наблюдая за поведением других членов группы. Если каждый человек смотрит на группу и видит 99 других с зелеными глазами, то, логически говоря, они должны ждать 100 ночей, чтобы дать остальным возможность остаться или уйти (и для каждого сделать этот расчет независимо). К сотой ночи, используя индуктивное рассуждение, вся группа предложила каждому члену группы возможность уйти и пришла к выводу, что уходить безопасно.

14. НОМЕРНЫЙ РЯД

Цифры от 1 до 10, указанные ниже, перечислены в порядке. Какое правило заставляет их располагаться в таком порядке?

8 5 4 9 1 7 6 10 3 2

Ответ : Номера упорядочены в алфавитном порядке на основе их английского написания: восемь, пять, четыре, девять, один, семь, шесть, десять, три, два.

(По материалам головоломки Карла Пруджана)

15. ЗАГАДКА С ПОДДЕЛЬНЫМИ МОНЕТАМИ

На видео ниже вы должны найти одну поддельную монету среди дюжины кандидатов.Вам разрешено использовать маркер (чтобы делать пометки на монетах, которые не меняют их вес) и всего три использования весов. Как найти в наборе одну подделку, которая немного легче или тяжелее обычных монет?

Ответ : Сначала разделите монеты на три равные стопки по четыре. Положите по стопке с каждой стороны весов. Если стороны уравновешены (назовем этот случай 1), все восемь из этих монет настоящие, а фальшивка должна быть в другой стопке из четырех.Отметьте законные монеты нулем (кружком) с помощью маркера, возьмите три из них и взвесьте с тремя оставшимися немаркированными монетами. Если они уравняются, оставшаяся немаркированная монета считается фальшивой. Если они этого не делают, сделайте другую отметку (на видео выше предлагается знак плюса для более тяжелого, минус для более легкого) на трех новых монетах на шкале. Протестируйте две из этих монет на шкале (по одной с каждой стороны) – если на них есть отметки «плюс», то более тяжелая из протестированных будет подделкой. Если на них стоит минус, то зажигалка – подделка.(Если они уравняются, то непроверенная монета – подделка.) Для случая 2 посмотрите видео.

16. ЭСКАЛАТОР RUNNER

Каждая ступенька эскалатора на 8 дюймов выше предыдущей. Общая высота эскалатора по вертикали составляет 20 футов. Эскалатор движется вверх на полшага в секунду. Если я наступаю на самую нижнюю ступеньку в тот момент, когда она находится на уровне нижнего этажа, и поднимаюсь со скоростью один шаг в секунду, сколько шагов я сделаю, чтобы добраться до верхнего этажа? (Примечание: не включайте шаги, которые вы предприняли, чтобы подняться на эскалатор и спуститься с него.)

Ответ : 20 шагов. Чтобы понять математику, потребуется две секунды. В течение этих двух секунд я самостоятельно взбегаю на две ступеньки, и эскалатор поднимает меня на высоту дополнительной ступени, в общей сложности три ступени – это также можно выразить как 3 раза по 8 дюймов или два фута. Поэтому за 20 секунд дохожу до верхнего этажа, сделав 20 шагов.

(По материалам головоломки Карла Пруджана)

17. ЗАГАДКА ПЕРЕСЕЧЕНИЯ РЕКИ

В видео-загадке ниже три льва и три антилопа гну застряли на восточном берегу реки и должны добраться до западного.Доступен плот, на котором одновременно могут перевозиться не более двух животных, и для того, чтобы переплыть его, требуется как минимум одно животное на борту. Если львы когда-либо будут превосходить гну по обе стороны реки (включая животных в лодке, если она на той стороне), львы съедят гну.

Учитывая эти правила, как все животные могут пересечь границу и выжить?

Ответ : Есть два оптимальных решения. Давайте сначала возьмем одно решение. При первом переходе каждое животное движется с востока на запад.Во втором переходе один гну возвращается с запада на восток. Затем на третьем переходе два льва переходят с востока на запад. Один лев возвращается (с запада на восток). Пройдя пять, два антилопа гну пересекают дорогу с востока на запад. При переходе через шесть один лев и один антилоп гну возвращаются с запада на восток. Пересекая седьмой, два антилопа гну идут с востока на запад. Теперь все три антилопа гну находятся на западном берегу, а единственный лев на западном берегу плывет обратно на восток. Оттуда (переход с восьмого по одиннадцатый) львы просто переправляются туда-сюда, пока все животные не доберутся до них.

Чтобы узнать о другом решении, посмотрите видео.

18. ТРИ ЧАСА

Я застрял на острове с тремя часами, все из которых были установлены на правильное время, прежде чем я застрял здесь. Одни часы сломаны и вообще не работают. Бегает медленно, теряя по одной минуте каждый день. Последние часы идут быстро, каждый день набирая одну минуту.

После того, как я был на мгновение заброшен, я начал беспокоиться о хронометрии. Какие часы с наибольшей вероятностью покажут правильное время , если я взгляну на часы в любой конкретный момент? Какой из наименьший покажет правильное время?

Ответ : Мы знаем, что секундомер должен показывать правильное время дважды в день – каждые 12 часов.Часы, которые теряют одну минуту в день, не будут показывать правильное время до 720 дней в своем цикле потери времени (60 минут в час, умноженные на 12 часов), когда они на мгновение будут отставать от графика ровно на 12 часов. Точно так же часы, которые выигрывают одну минуту в день, также будут неправильными до тех пор, пока не пройдут 720 дней после того, как они пошли в неправильное русло, когда они будут на 12 часов раньше запланированного срока. Из-за этого часы, которые вообще не работают, скорее всего, будут показывать правильное время. Два других с одинаковой вероятностью ошибочны.

(По материалам головоломки Карла Пруджана)

19. ЗАГАДКА ЭЙНШТЕЙНА

В этой загадке, ошибочно приписываемой Альберту Эйнштейну, вам представлен ряд фактов, и вы должны вывести один факт, который не представлен. В случае с видео ниже похищена рыба. В ряду пять одинаковых домов (пронумерованных от одного до пяти), и в одном из них находится рыба.

Посмотрите видео, чтобы получить различную информацию о жителях каждого дома, правилах получения новой информации и выяснить, где прячется эта рыба! (Примечание: вам действительно нужно посмотреть видео, чтобы понять это, и список подсказок тоже полезен.)

Ответ : Рыба находится в доме 4, где живет немец.

20. МАТЕМАТИЧЕСКАЯ ОБЕЗЬЯНА

Трое потерпевших кораблекрушение и обезьяна оказались вместе на тропическом острове. Они проводят день, собирая большую кучу бананов, насчитывающую от 50 до 100. Брошенные люди соглашаются, что на следующее утро они втроем поделят бананы поровну между собой.

Ночью просыпается один из потерпевших кораблекрушение. Он боится, что другие могут обмануть его, поэтому берет свою треть и скрывает ее.Поскольку на один банан больше, чем количество, которое можно разделить поровну на трети, он дает обезьяне лишний банан и снова засыпает.

Позже ночью просыпается второй потерпевший кораблекрушение и повторяет то же поведение, терзаемый тем же страхом. И снова он берет одну треть бананов в кучу, и снова количество на единицу больше, чем можно было бы разделить на три части, поэтому он передает лишний банан обезьяне и прячет свою долю.

Еще позже последний потерпевший крушение встает и повторяет ту же самую процедуру, не подозревая, что двое других уже сделали это.И снова он берет треть бананов и получает еще один, который дает обезьяне. Обезьяна очень довольна.

Когда потерпевшие кораблекрушение собираются утром, чтобы разделить банановую добычу, все они видят, что куча значительно уменьшилась, но ничего не говорят – каждый боится признаться в своем ночном воровстве бананов. Они делят оставшиеся бананы на три части, и в итоге получается один лишний для обезьяны.

Учитывая все это, сколько бананов было в исходной куче? (Примечание: в этой задаче нет дробных бананов.Мы всегда имеем дело с целыми бананами.)

Ответ : 79. Обратите внимание, что если бы куча была больше, следующее возможное число, которое соответствовало бы указанным выше критериям, было бы 160, но это выходит за рамки, указанные во втором предложении («от 50 до 100») головоломки. .

(По материалам головоломки Карла Пруджана)

21. ЗАГАДКА ВИРУСА

На видео ниже вирус проник в лабораторию. Лаборатория представляет собой одноэтажное здание, построенное в виде сетки комнат 4х4, всего 16 комнат, 15 из которых загрязнены.(Входная комната все еще в безопасности.) Есть вход в северо-западном углу и выход в юго-восточном углу. Снаружи связаны только входная и выходная комнаты. Каждая комната соединена с соседними комнатами шлюзами. Как только вы войдете в зараженную комнату, вы должны нажать выключатель самоуничтожения, который уничтожит комнату и вирус в ней, как только вы перейдете в следующую комнату. Вы не можете повторно войти в комнату после того, как ее переключатель был активирован.

Если вы войдете через комнату входа и выйдете через комнату выхода, как вы можете обеззаразить всю лабораторию? Какой маршрут вы можете выбрать? Посмотрите видео с отличным визуальным объяснением проблемы и решения.

Ответ : Ключ находится в комнате входа, которая не загрязнена, и поэтому вы можете повторно войти, выйдя из нее. Если вы войдете в эту комнату, переместите одну комнату на восток (или на юг) и обеззаразите ее, затем снова войдите в комнату входа и уничтожьте ее по пути в следующую комнату. Оттуда ваш путь становится ясным – на самом деле у вас есть четыре варианта завершения пути, которые показаны на видео выше. (Нарисовать это на бумаге – это простой способ увидеть маршруты.)

22. ЗАКОННАЯ ЗАГАДКА

По словам автора сборника головоломок Карла Пруджана, этот был фаворитом писателя Льюиса Кэрролла.

Премьер-министр планирует званый обед, но он хочет, чтобы он был небольшим. Он не любит толпы. Он планирует пригласить зятя своего отца, тестя своего брата, брата своего тестя и отца своего зятя.

Если бы отношения в семье премьер-министра сложились наиболее оптимальным образом, каково было бы минимально возможное количество гостей на вечеринке? Обратите внимание, что мы должны предполагать, что браки с кузенами разрешены.

Ответ : Один. Через несколько сложных путей в семье премьер-министра можно сократить список гостей до одного человека. Вот что должно быть правдой: у матери премьер-министра есть два брата. Назовем их братом 1 и братом 2. У премьер-министра также есть брат, который женился на дочери двоюродного брата 1. У премьер-министра также есть сестра, которая вышла замуж за сына брата 1. Сам ведущий женат на дочери брата 2. Из-за всего этого брат 1 является зятем отца премьер-министра, тесть брата премьер-министра. law, брат свекра премьер-министра и отец зятя премьер-министра.Брат 1 – единственный гость на вечеринке.

(По материалам головоломки Карла Пруджана)

23. ЗАГАДКА ЗАКЛЮЧЕННЫХ ЯЩИКОВ

На видео десять участников группы случайно поместили свои музыкальные инструменты в коробки, помеченные изображениями музыкальных инструментов. Эти изображения могут совпадать, а могут и не совпадать с содержанием.

Каждый участник получает пять выстрелов по открытию ящиков, пытаясь найти свой собственный инструмент. Затем они должны закрыть ящики. Им не разрешается рассказывать о том, что они находят.Если вся группа не найдет свои инструменты, все будут уволены. Вероятность того, что они случайно выберут этот путь, составляет 1 к 1024. Но у барабанщика есть идея, которая радикально увеличит их шансы на успех до более чем 35 процентов. Какая у него идея?

Ответ : Барабанщик сказал всем сначала открыть коробку с изображением их инструмента. Если их инструмент внутри, они готовы. Если нет, участник группы наблюдает, какой инструмент был найден, затем открывает коробку с изображением этого инструмента и так далее.Посмотрите видео, чтобы узнать, почему это работает математически.

24. S-N-O-W-I-N-G

Однажды снежным утром Джейн проснулась и обнаружила, что окно ее спальни запотело от конденсата. Она нарисовала на нем пальцем слово «СНЕЖНЫЙ». Затем она вычеркнула букву N, превратив ее в другое английское слово: «СЕЯ». Она продолжала так, удаляя по одной букве за раз, пока не осталась только одна буква, которая сама по себе является словом. Какие слова произнесла Джейн и в каком порядке?

Ответ : Снег, посев, задолженность, крыло, победа, в, I.

(По материалам головоломки Мартина Гарднера)

25. ТАЙНА ПЕЧАТИ

Находясь в отпуске на острове Бима, я зашел на почту, чтобы отправить домой несколько пакетов. Валюта на Бима называется пим, и почтмейстер сказал мне, что у него есть только марки пяти разных номиналов, хотя эти значения не напечатаны на марках. Вместо этого на марках есть цвета.

Марки были черного, красного, зеленого, фиолетового и желтого цветов в порядке убывания стоимости.(Таким образом, черные марки имели самый высокий номинал, а желтые – самый низкий.)

Одна посылка требовала марок на 100 пимов, и почтмейстер вручил мне девять марок: пять черных, одну зеленую и три фиолетовых.

Для двух других пакетов потребовалось по 50 пимов каждая; для них почтмейстер вручил мне два разных комплекта из девяти марок. Один набор состоял из одной черной марки и двух других цветов. Другой набор состоял из пяти зеленых марок и по одной каждого другого цвета.

Какое наименьшее количество марок необходимо для отправки пакета из 50 пимов и какого цвета они будут?

Ответ : Две черные печати, одна красная печать, одна зеленая печать и одна желтая печать. (Может помочь выписать приведенные выше формулы штампа с использованием различных b, r, g, v и y. Поскольку мы знаем, что b> r> g> v> y, и у нас есть три описанных случая, мы можем сделать некоторая алгебра, чтобы найти значения для каждой марки: черные марки стоят 18 пим, красные – 9, зеленые – 4, фиолетовые – 2, а желтые – 1.)

(По материалам головоломки Виктора Брайанта и Рональда Постилла)

Источники: Brain Teasers Яна Уивера; “Головоломки и головоломки” Чарльза Бут-Джонса; Загадки и другие загадки Дж. Майкла Шеннона; Изобилие головоломок: головоломки, викторины и кроссворды из журнала Science World под редакцией Карла Пруджана; «Стрела-книга головоломок» Мартина Гарднера; «Санди Таймс», Книга «Головоломки» , под редакцией Виктора Брайанта и Рональда Постилла.

.

Добавить комментарий

Ваш адрес email не будет опубликован. Обязательные поля помечены *